Ophthalmology Clinics for Postgraduates Namrata Sharma, Atul Kumar, Prafulla Kumar Maharana
INDEX
Page numbers followed by f refer to figure and t refer to table
A
Aberrant regeneration 360
Acanthamoeba 90, 91, 93, 93f
cysts of 99
growth of 99
keratitis 90, 95
species, cysts of 99
trophozoites 99
Accommodative esotropia, classification of 391t
Acetazolamide 306
Acetylcholine receptor antibody 405
test 22
Acute angle closure attack, management of 182
Acute spastic entropion, management of 63
Adenocarcinoma, primary 47
Adenoid cystic carcinoma 4, 44, 45f, 47, 50, 50t, 51
Adnexa 338
Adnexal diseases 18
Adult vitelliform macular dystrophy 276
Afferent papillary defect 246
Age-related eye disease 281, 282
study 281
Age-related macular degeneration 182, 208, 214, 272, 275, 279, 282, 347
classification of 281t
clinical classification of 280t
Aicardi syndrome 322
Alcohol intake 230
Allergic conjunctivitis, signs of 107
Allergic reactions 52
Allergy 75
Allograft rejection 138
episodes of 137
Alport's syndrome 423, 428
Amblyopia 52, 82, 88, 164, 166, 320, 421
management of 67, 166
treatment 393
Amelanotic melanoma 305
Aminoaciduria 422
Amniotic membrane
graft 104, 152
transplantation 150
Amorphous corneal dystrophy, posterior 123
Amphotericin 102
Amsler grid testing 226
Amsler-Krumeich classification 110t, 116
Amyloidosis 211
primary 205
Anemia 237, 240
Aneurysm 357, 375
Angioid streaks 277
Angiomatous proliferation 276
Angioneurotic edema 107
Angiotensin-converting enzyme 149
Angle anatomy, distortion of 198
Angle closure glaucoma
classification of 182t, 184
management of 182t
primary 178, 181, 182
signs of 209
Angle closure, mechanisms of 183
Angle recession glaucoma, treatment of 198
Anisometropic amblyopia 164
Ankyrin repeat proteins 283
Anophthalmia 18
Anophthalmos 164
Anterior chamber paracentesis 97, 100
Anterior segment optical coherence tomography 131, 137, 165, 182, 334
Anti VEGF injections, role of 235
Antiangiogenic drugs 194
Antibiotics 408
Antibody against lingo1 383
Antifungal agents 101
Antifungal drugs, penetration of 103
Antiglaucoma drugs 178, 194
Anti-glaucoma medications, timing of 174
Anti-inflammatory drugs 194
Antioxidant supplementation 281
Anti-thyrotropin receptor antibodies 43
Antiviral agents 77
Apex of orbit 39
Aphakic keratopathy 130
Aponeurotic 22
ptosis 23
Aqueous misdirection 184
Aquired keratoglobus 144
Areflexia 422
Arruga intracapsular forceps 449, 449f
Arruga needle holder 445f
Arsenic exposure 10
Arteriovenous malformations 85
Artery forceps 442
Aspiration cannula 448f
Aspiration of hyphema 448
Aspirin 307
role of 242
Assorted retinal diseases 195
Asteroid hyalosis 211
Astigmatism, type of 420
Astrocytic hamartoma 411
Asymmetric orbital size 9
Asymmetric palpebral fissure 9
Atopy-bronchial asthma 107
Atresia of external auditory meatus 86
Atrophy of
fundus 247
orbits 245
Attack of hydrops 145
Atypical mycobacteria 90
Atypical optic neuritis 381, 381t
Autoantibody 226
Autofluorescence 347
Autosomal dominant transmission 346
Avellino corneal dystrophy, signs of 119
Axenfeld-Rieger syndrome 158
B
Baclofen 407
Bacterial corneal ulcers, treatment of 101
Bacterial keratitis, management of 100
Bacterial ulcer 91
Bagolini striated glasses 359, 388
Band shaped keratopathy 125
Bandage contact lens 103, 132, 152
Band-shaped keratopathy 148, 333
Bardet-Biedl syndrome 247
Bared circumciliary vessels 177
Barkan's membrane 189
Barraquer needle holder 444, 444f
Barron's artificial anterior chamber 453
Basal cell carcinoma 9, 12, 13, 69
Basal hole diameter 254
Basal laminar deposits 284
Bassen-Kornzweig syndrome 247
Beam radiation therapy, external 50
Beaten-bronze 347
Bell's palsy, treatment for 77
Bell's phenomenon 4, 18, 61, 73
loss of 37
Berke's criteria 54t
Berke's method 17
Berke's ptosis clamp 455, 456f
Bevacizumab 283
Bielschowsky's head tilt 373
Bifoveal fixation 388
Binocular single vision 400
Biocompatible 30
Biogenic implants 30
Biointegrated implants 30
Bionic eye 250, 284
Biopsy 45
Bladder cells 434f
Bleeding
disorder 209
from abnormal vessels 214
risk of 220
Blepharitis 69
Blepharochalasis 54
Blepharoconjunctivitis 69
Blepharokymographic analysis 74
Blepharophimosis 63, 66f
epicanthus inversus syndrome 64f
syndrome 18, 54, 55
management of 67
Blepharophimosis-ptosis-epicanthus inversus syndrome 66
Blindness, family history of 241
Blink reflex, loss of 54
Blood 405
glucose 102
investigations 73
removal of 459
sugar 73, 211
transfusions 224
vessels, lack of 79
Blood-retina barrier 242
Blowout fracture 39
etiopathogenesis of 34
of orbit 30
Bluish colored swelling 63, 86
Bluish discoloration of sclera 188
Blunt trauma 210, 353, 435
manifestations of 198
Blurred vision, symptoms of 157
Blurring 141, 346
Bone mineral density, measurement of 65
Bony orbital walls 8
Bony spicules 250
Boston ocular surface prosthesis 147
Botulinum toxin 361, 394, 401, 407
role of 63
Bowman lacrimal probe 456, 456f
Bowman layer dystrophies 123
Bowman's layer 145
loss of 162
Brainstem auditory evoked response 74
Bruch's membrane 292
Bruckner test 385
Bulbar conjunctiva 11
Bull's eye
appearance 425f
maculopathy 347
diagnoses of 348
Bullae, rupture of 129
Buphthalmos 18, 188, 415
bilateral 188f
diagnosis of 189
reversible 190
Burian's classification 398
Busin glide 454
Byron smith classification 27
C
Calcified cyst 330
Canal of petit 210
Canalicular stenosis 12
Canaliculi 29
Canaloplasty 176
Cancer-associated retinopathy 247
Capillary hemangioma 82, 83f, 84
course of 83
Capillary nonperfusion 240, 310
Capsular bag distention syndrome 433
Capsular tension ring 418, 429
Capsule on retroillumination, posterior 426f
Capsule tension segments 418
Capsulorhexis 426
anterior 412
Capsulotomy, anterior 447
Cardiac disease 160
Cardiovascular abnormalities 86
Cardiovascular disease 163, 172, 300
Cardiovascular system 230
Carotid angiography 8
Carotid cavernous fistula 9
Carotid Doppler scan 219
Carotid-cavernous fistula 39, 173
Carpal tunnel syndrome 118
Castroviejo caliper 445, 445f
Castroviejo corneoscleral scissors 446, 446f
Castroviejo needle holder 444, 444f
Cat's paw lacrimal wound retractor 444
Catabolism of bloodlike 210
Cataract 12, 162, 290, 321, 429, 438
cause of 335
detection of 130
in silicone oil-filled eyes 332
in trauma, pathophysiology of 439
presence of 130, 160
surgery 206t, 440, 447
effects of 237
history of 130, 192
small incision 342, 446
type of 238, 335
Cataracta nodiformis 436
Cat-scratch diseases 290
Causative organism, type of 90
Causes, variety of 74
Cavernous hemangioma 83, 84, 84t
management of 83
Cavernous sinus 8
Cell precipitates 432
Central areolar choroidal dystrophy 347
Central cloudy dystrophy of francois 123
Central corneal thickness
increased 136, 159
variation of 338
Central Eales disease 211f
Central nervous system 185, 329
anomalies 160
Central retinal
artery occlusion 193, 195, 300, 301f, 302
vein occlusion 193, 195, 216, 380
types of 217t
Central scotoma 346
Central serous chorioretinopathy 254, 303, 305, 307t
Central suppression scotoma 388
Central vein occlusion 222
Cerebrospinal fluid 368
Cerebrovascular accident 233
Ceroid lipofuscinosis 347
Chalazion 69
clamp 455, 455f
scoop 455, 455f
Chandler syndrome 157, 158
Charge syndrome 322
Charles flute
cannula 460
needle 459, 459f
Charleaux sign 109
Chemical injury 29
Chemotherapy 12, 357
Cherry-red spot 300, 301
diagnosis of 302t
Chest wall deformities 415
Childhood onset retinal dystrophy, severe 248
Chloroquine retinal toxicity 347
Choriodal neovascularization 321
Chorioretinal venous anastomosis 221
Chorioretinopathy 306
Choriostoma 81
Choroid 247
Choroidal coloboma 322
pathophysiology of 321
Choroidal hemorrhage 269, 352
Choroidal melanoma 291, 293t, 294, 297, 298
Choroidal neovascular membrane 214, 215, 254, 304, 316, 347
Choroidal neovascularization 255, 273, 275, 281, 282
Choroidal nevus 294
Choroidal rupture 352
Choroidal vessels 249
Choroideremia 247
Chromosomal abnormalities 190
Chronic blepharoconjunctivitis 68
Chronic infection 29
Chronic macular hole 347
Chronic sun exposure, signs of 151
Ciancia syndrome 393
Cicatricial ectropion 57, 58f, 61, 62
causes of 62
management of 58
Cicatricial skin changes 57
Cigarette smoking, avoidance of 348
Cilia
epilation of 442
removal of 442
Ciliary body 198f, 285
Clear hexagonal margins, loss of 159
Climatic droplet keratopathy 150
Coagulase negative staphylococcus 90
Coat's disease 195, 214, 240, 411
Cobweb opacity 436
Cochet-Bonnet esthesiometer 94
Cogan's twitch sign 22
Cogan-Reese syndrome 157159
Cold intolerance 12
Colibri forceps 441, 441f
Colibri-style polack double corneal forceps 453
Collaborative ocular melanoma study 298
Collagen vascular disorders 225
Collection of corneal material 98
Coloboma 86, 88, 318, 321, 322, 341
detachments in 323
histopathology of 322
incomplete 321
lasered, types of 324
of eyelid 86
types of 321
Colobomata, diagnosis of 322
Color vision 248
Combination trephines 452
Comitant esotropia, types of 391t
Commotio retinae 352
Complete blood count 7, 88, 166, 331
Complete coloboma 321
Complete palsy 368
Compressive neuropathy 82
Compressive optic neuropathy 380
Cone-rod dystrophy 107, 248, 347
Cones, types of 108
Congenital abnormality 39
Congenital atrophy of skin 424
Congenital cataracts, etiology of 413
Congenital corneal
opacity 161t
causes of 161t
Congenital cranial dysinnervation disorders 400
Congenital dyskeratosis 424
Congenital ectropion 57, 59
uvea 190
Congenital entropion 61, 62, 62t
Congenital epiblepharon 62, 62t
Congenital eyelid colobomas, family history of 64, 86
Congenital glaucoma 143, 154, 161, 187, 187t, 188f, 190t
classification of 190
diagnosis of 189
loops of blood vessels 189
management of 190
Congenital hereditary
endothelial dystrophy 123, 130, 131, 153 155, 156, 161
stromal dystrophy 155
Congenital Horner syndrome 23
Congenital ichthyosis 424
Congenital keratoglobus 144
Congenital myogenic ptosis 23
Congenital neurogenic ptosis 23
Congenital ptosis 18, 19t, 21, 52
causes of 54
family history of 52
pathology in 55
severe 16f, 52f
treatment of 20
type of 21
Congenital rubella 154, 247
Congenital simple ptosis 65
Congenital stromal corneal dystrophy 123
Congenital torticollis 376t
Congestive heart failure 233
Conjuctival dermoid cyst 79
Conjunctiva 56, 65, 76, 90, 107, 309, 318, 325, 359, 366, 372, 378, 410, 416, 422, 424, 428, 436
edema of 40
retraction of 444
Conjunctival cicatrizing diseases 29
Conjunctival dermoid cyst 79, 80
Conjunctival dissection 447
Conjunctival flaps 103
Conjunctival fornix 12
Conjunctival keratinization 12
Conjunctival lymphoma 72
Conjunctival swab 97
Conjunctival, removal of 440
Connective tissue disorder 91, 92, 200, 342
Consciousness, loss of 351
Constant exotropia 396
Contact lens 91, 111, 144
types of 144
wear 15, 106
Contracted socket 25
causes of 28
classification of 27t
Contralateral enophthalmos 9
Contrast sensitivity 248
Control cardiovascular risk in diabetes 237
Cornea 56, 65, 90, 92, 94, 309, 319, 325, 359, 366, 372, 378, 410, 416, 422, 424, 428, 432, 436
anterior 119
appearance of 154
center of 92, 106
central 111
flaps of 441
in peters’ anomaly 162
normal 116
palpebral adhesions 87
perforation of 103
slices of 111
transparency of 115
Corneal biopsy 100, 104
Corneal choristoma classification
Mann's scheme 167
Stargardt scheme 167
Corneal clouding 153
Corneal decompensation 184, 205
Corneal diameter, normal 142f
Corneal dystrophy
classification of 122
signs of 94
Corneal edema 94, 132, 157f, 159
blepharophimosis, presence of 153
Corneal endothelial
punches 453
reflex 108
Corneal endothelium 343
Corneal epithelial
defect, presence of 129
edema, stage of 129
Corneal erosion, recurrent 117, 124, 230, 238
Corneal exposure 74
Corneal foreign body 440
Corneal graft 104
rejection, acute 137t, 139
Corneal granular dystrophy 107
Corneal guttae, causes of 133
Corneal haze 100
Corneal hydrops 109f
Corneal hypoesthesia 238
Corneal inflammation 94
Corneal nerves, increased visibility of 108
Corneal nodules 152f
Corneal opacity 87, 118, 162
treatment of 162
Corneal pachymetry 131
Corneal perforation 94
risk of 101
Corneal protrusion 145
Corneal scar 97, 108, 108f, 164, 167
infection 164
presence of 132
trauma 164
Corneal sensations 121
Corneal sensitivity, loss of 54
Corneal stromal
dystrophy 117, 122t
edema, stage of 129
Corneal surface, irregularity of 130
Corneal thickness 111, 121
central 107, 110, 121, 173
increased 154f
measurement of 111
Corneal thinning 94, 145
development of 94
Corneal tissue firmly 443
Corneal topography 143, 146
Corneal transplantation 112
Corneal trauma 151
Corneal trephines 451
Corneal ulcer 90, 91, 91t, 94, 94f, 95, 96, 97t, 103, 104
central 92f
diagnosis of 97, 104
grading of 96, 97t, 104
perforation of 91
size of 93
trial 101
Corneal vascularization 121
Corneolenticular touch 429
Corona sign 276
Correct osmolarity 256
Cortical cataract 421
Corticosteroid 42, 172, 200
intake, history of 174
use of 101
Corynebacterium diphtheria 91, 92
Cosmetic deformity 163, 166
Cosmetically unacceptable enophthalmos 35
Costenbader 399
Cox's postulates 354
Cranial nerve 237, 371
Crawford's fascia lata stripper 456
Crescent knife 446f
Crescentic lamellar keratoplasty 147
Crouzon's syndrome 415
Cryosurgery 12
contraindications of 12
Cryotherapy 268, 316, 327
application of 12
Cryptophthalmos 86, 89
Cuff of subretinal fluid 253
Cutaneous lesion, site of 186
Cuticular drusen 277
Cycloablative procedures 189
Cyclodestructive procedure 195, 339
Cyclophotocoagulation 186
Cycloplegic refraction 164
Cyclosporin A 138
Cyst
evidence of 330
recurrent 82
rupture of 78, 82
Cysticercosis, life cycle of 331
Cysticercus cellulosae 329
Cystoid 311
macular edema 245, 246, 285, 286, 288, 289f, 290, 315
retinal spaces 280f
Cystotome needle 448, 449f
Cytokines, release of 106
Cytomegalovirus 410, 411
D
Dace procedure 269
Dacryocystorhinostomy 450
surgery 444
Dahan's formula 412t
Dalrymple's sign 37
Dark adaptation, loss of 233
Dastoor iris repositor 448, 448f
Deafness 160
Deep anterior lamellar keratoplasty 112, 118, 124
Deep dermoid cyst 81
Deep seated ocular pain 197
Deep stromal lesions 124
Deeply pigmented skin 12
Deficient amounts of skin 64
Degeneration 125
Delineate fovea 241
Dellen formation 164, 166
Dense premacular hemorrhage 213
Deoxyribonucleic acid 233
Dermatochalasis 18
Dermoid
cyst 4, 77, 79
anterior 81
diagnosis of 164
epidemiology of 80, 167
grading of 167
recurrent 164, 167
Dermolipoma 164
Descemet stripping automated endothelial keratoplasty 156f
Descemet's folds 145
Descemet's membrane 115, 122, 123, 133, 454
compression of 108
endothelial keratoplasty 132
Descemet's stripping
automated endothelial keratoplasty 132, 155, 446
endothelial keratoplasty 132
Descemetocele 93f
Desmarre lamellar dissector 453
Desmarre retractor 443, 444f
Developmental disorders 181
Dewecker scissors 446, 446f
Diabetes 208, 414
control 237
mellitus, diagnosis of 236
duration of 236
family history of 209
mellitus 16, 36, 200, 229, 236, 252, 308
Diabetic macular edema 230, 233, 308, 309f
laser procedures for 312t
Diabetic papillopathy 240, 243
Diabetic retinopathy 195, 219, 236, 238t, 252, 309, 310, 312, 314, 316
study, early treatment 309, 233, 237
vitrectomy study 234
Diabetic subjects 234
Diamond knife 453
Diamond-dusted membrane scraper 460, 460f
Diamond-knife assisted DALK 114, 127
Dietary factors 273
Diffuse concussion cataract 437
Diffuse diabetic macular edema 310f
Diffuse retinal pigment epitheliopathy 304
Diffusion tensor imaging 383
Dilated blood vessels 10
Dilated retinal vessels 276
Diode laser cyclophotocoagulation 195
Diplopia 356, 365, 371, 395, 404
charting 32, 359, 366, 370, 388
management of 369
treat persistent 35
Disc coloboma 322
Disc diameter 256
Disciform scar 280f
Disease discrete, early stages of 120
Disease progresses 109, 128
Disease, early stage of 211
Disorganized epithelium 162
Disproportionate proptosis 37f
Disseminated subepithelial opacity 436
Distant direct ophthalmoscopy examination 417
Distended veins 378
Docosahexaenoic acid 249
Dot acuity test 410
Double corneal forceps 453
Double eversion of eyelid 443
Double-freeze thaw technique 290
Down syndrome 107
Doyne honeycomb retinal dystrophy 276
Droopy eyelid, compression of 52
Dropped lens, management of 345
Drusen 273, 275, 284
Dry eye 54, 86, 163
syndrome 18
Dry socket, treatment of 28
Drying of cornea 151
DSAEK busin forceps 454
DSAEK spatula 454
Dual bore cannula 460
Duane retraction syndrome 24, 400
Duane's classification 398
Duane's syndrome, types of 17
Duane's retraction syndrome 86, 164
Dubowitz syndrome 65
Duke elder classification 425
Dumb-bell 82
dermoids 82
Dysostosis multiplex 300
Dysplastic eyelids 65
structure of 65
Dystrophic calcification 274
Dystrophy 125
with keratoconus 127
E
Eale's disease 195, 211, 213, 415
Ear
abnormalities 64
alterations of 89
anomalies 169f
Earliest sign of keratoconus 107
Early age-related macular degeneration 280
Early rosette cataract 436, 438f
Early surgery, reasons for 215
Echinocandins 101
Echocardiography 166
Ectatic protrusion of cornea 108f
Ectodermal dysplasia 424
Ectopia lentis 341, 414
causes of 345
et pupillae 415
Ectopic lacrimal gland 164
Ectropion 12, 56
basic cause of 58
grade of 58
of upper lip 88
uveae, presence of 192
Eculizumab 283
Edema, type of 227
Edrophonium
chloride test 22
test 405, 406
Ehler-Danlos syndrome 415, 416
Electrodiagnostic testing 74
Electrodynamic cutters 458
Electromyography 406
Electroneurography 74
Electronic magnifiers 285
Electrooculogram 350
Electrophysiology 347, 383
Electroretinogram 220, 241, 247
Elschnig's operation 59
Elschnig's pearls 433, 434
Encephalocele 78
End-grasping forceps 458f
Endocrine 7
Endogeneous
disease 174
release of corticosteroids 172
Endophthalmitis 101, 257, 269, 352, 411
Endoscopic approach 33
Endothelial cells 160
Endothelial dysfunction, first indicator of 136
Endothelial dystrophies 123
Endothelial keratoplasty 132, 140, 189
Endothelial rejection 134, 137
Enophthalmos 18
Entropion 12, 60
causing keratopathy 60f
types of 61
Enucleation scissors 447, 447f
Enucleation socket 63
Enzymatic vitreolysis 317
Enzyme-linked immunosorbent assay 211
Epibulbar dermoid 164
Epicanthus inversus 63, 65
syndrome 63, 66f
Epidermal dermoid cyst 77, 79
Epikeratophakia 114
Epilation forceps 442, 442f
Epinucleus removal 427
Epiphora 63
triad of 188
Epiretinal membrance 227, 245, 246, 259, 288, 290, 314, 317, 460
Episcleral osteoma 164
Episcleral plaque brachytherapy 297
Epithelial defect 167
size of 93
Epithelial dystrophy 122
Epithelial erosion 148
Epithelial recurrent erosion dystrophy 122
Epithelial scraping 124
Epithelioid cell melanoma 298
Equatorial cells 435
Erythrocyte sedimentation rate 7, 211, 219, 288
Erythropoietin 383
Escherichia coli 99
Esodeviation 384
Esophoria 391
Esotropia 391, 395, 397
of right eye 365f
Essential infantile esotropia 393
Ethmoidal mucocele 1
Evisceration curette 457
Exacerbations, acute 382
Excision biopsy 8, 11
Exodeviation 395
Exophoria 399
Exophthalmometer, type of 6
Exophthalmometry 6
Exotropia 395
alternating constant 398t
epidemiology of 397
unilateral constant 398t
Exposure keratitis 88
Exposure keratopathy, signs of 7
Extensive disease 12
Extracapsular cataract extraction 446
Extrafoveal 281
Extraocular diseases 195
Extraocular movement 400
Extraocular muscle 7
abnormalities 9
Extravasated blood, presence of 208
Extreme ectropion 58
Exudative retinal detachment 260
Exudative vasculopathy 249
Eye
buphthalmos, right 188f
complete coverage of 88
deviation of 52, 395
disorders, allergic 91
lid margins 10
movement
limitation of 45
restriction of 10
rubbing 106
Eyeball 1, 3, 309, 318, 325, 338, 357, 378, 400, 410, 416, 422, 424, 428, 436
absence of 63, 86
during surgery 441
size of 153
small size of 63, 86
Eyebrow hair, loss of 86
Eyelash 173, 179
loss of 10, 12, 68
rubbing cornea 61f
Eyelid 56, 69, 88, 309, 410, 424, 436, 428
abnormal 63
adnexal structures of 9
anatomy of 76
architecture, disruption of 68
carcinoma of 11
coloboma 89
surgical 87f
treatment of 88
crutches 407
defects 13
drooping of 52, 63
edema of 40
fullness of 40
margin, distortion of 10
mechanical ptosis, right upper 19f
notching 12
repair, scar of 197
structures 88
swab 97
Eylea 283
F
Fabry's syndrome 414
Face 357
turn 400
Facial angiomatosis, bilateral 185f
Facial asymmetry 64, 163, 357, 372
presence of 6
Facial nerve 76
anatomy of 76
lesion 39
Facial palsy, signs of 57
Facial symmetry 16
Familial exudative vitreoretino-pathy 210, 211
Familial vitreoretinal disorders 262
Fasanella Servat operation 20
Fasting blood glucose 411
Fatigability 22
Feeling pulsation 2
Fellow eye 211
examination of 121
role of 215
Femtosecond laser 114, 127
Fibrinoid syndrome 213
Fibrovascular disciform scar 276
Fibrovascular membrane 196, 268
Fibrovascular tissue, proliferation of 192
Filtration surgery 189, 195, 199, 339
trabeculectomy 176
Fine toothed tissue holding forceps 442
Fine-needle aspiration biopsy 8
Fine-needle aspiration cytology 11
Fish
mouthing 269
strike sign 256
Fishman classification 348, 349t
Fixed hypopyon 96f
Fleck corneal dystrophy 123
Fleck lesions 349f
Flieringa ring 450f
Floaters, history of 230, 236
Floppy eyelid syndrome 73, 107
Fluid occurs, egress of 459
Fluorescein angiogram 248, 288, 310
Fluorescein angiography 226, 255, 277, 295, 310f, 316
Fluorescein staining 61
Fluorinated pyrimidines 101
Fluoroquinolones 101
Focal cataract 184
Force duction test 32, 367
Force generation test 32, 368
Force injury 190
Forced duction test 360, 368, 374, 390, 405, 443
Forceps injury 154, 155
Forehead wrinkles, loss of 57
Foreign body 93
forceps 459
granuloma 164
irritation 163
sensation 60, 72, 117, 148, 163
Fornices, depth of 26
Four Δ prism test 388
Fourth cranial nerve
function in third palsy 374
palsy 371
Fourth nerve
course of 375
palsy 376
Foveal avascular zone 226, 241, 310
Foveal drusen 254
Foville syndrome 370
Fraser syndrome 87, 89
Freer periosteal elevator 456
Frequent change of glasses 106, 172
Frequent eye infections 60
Frisby Davis distance test 389
Frontalis overaction 16
Fuch's endothelial corneal dystrophy 131
Fuch's heterochromic iridocyclitis 290
Fuchs’ endothelial
dystrophy 158
corneal 123, 128
Fundal coloboma 318, 319f, 322
Fundus 32, 65, 78, 314, 326, 359, 366, 374, 410, 422, 425, 429, 432, 437
autofluorescence 280, 295
coloboma 323t
examination 109, 181, 263, 319, 417
flavimaculatus 348
fluorescein
angiograms 275
angiography 217, 219, 240, 305, 310, 347
ophthalmoscopy 374
photography 174, 240, 374
tessellated appearance 249
Fungal corneal ulcer 93
Fungal keratitis 103
management of 101
Fungal ulcer 93
Furrow degeneration 145
Fusarium species 91
Fusional amplitudes 387
G
Gabapentin 407
Ganglion cell layer 257
Gas permeable contact lenses 111
Gass clinical grading system 316
Gass retinal detachment hook 460
Gastrointestinal ulcer disease 43
Gaucher's disease 301, 302
Gelatinous drop-like corneal dystrophy 123
Gelatinous thickening of limbus 107
Genetics 428
of stromal dystrophies 127
Genital anomalies 89
Genitourinary malformations 64
Geographic atrophy 274
Ghost cell glaucoma 213
Giant retinal
dialysis 326
tear 265, 325, 326, 328, 329, 353
Gill's lamellar dissector 453
Glaucoma 104, 171, 421, 174, 176, 177, 179, 187, 192, 431
angle closure 250
angle recession 174, 197, 198
cause of 186
closure 179
damage, severity of 177
development of 339
diagnosis of 173, 174
unilateral 198
diagnostic tools of 177
drainage device 176, 186, 339
surgery 195
family history of 337
in angle recession 199
increased risk of 172
management of 429
mechanism of 250
medications 287
primary open angle 171, 200, 205
progression of 172
refractory 162
risk of 202
stage, angle-closure 196
surgery 440, 450
treatment of 162
Glaucomatocyclytic crisis 200
Glaucomatous cyclitis 268
Glaucomatous optic
damage, severity of 177
neuropathy, severity of 195
Glaukomflecken 180f
Glioma of optic nerve 3
Globe fixation
forceps 441, 441f
rings 450
Globe injury
closed 355
presence of open 354
Globe perforation 12
Globular corneal protrusion 141f, 142f
Globus type of cone 108
Glucose tolerance test 240
Gold weight implantation 75
Goldenhar syndrome 87, 89, 168, 168t, 169f, 402
Gonioscopy 161, 164, 189, 326, 436
of both eyes 180
Goniotomy 186, 189
Gopal Krishna classification 26
Graft edema
development of 134
presence of severe 137
Graft host junction 139
Graft rejection 134, 140
acute 134, 135, 136f, 137
episode of 137
Granular corneal dystrophy 117, 123
Granulation tissue, mass of 72
Granuloma 330
Graves’ disease 1
signs of 3
Graves’ orbitopathy 41
Grayson-Wilbrandt corneal dystrophy 123
Greater visual field loss 174
Gross examination of cyst 330
Gross mass of tumor plus 11
Group of non-inflammatory 117
Guillain-Barré syndrome 24, 73
Guillotine type cutters 458
Gunderson's flap 103
Gyrate atrophy of retina 247
H
Haab's striae injury 190
Hallmark of
graft rejection 135
keratoconus 110
Harada's disease 305
Harboyan syndrome 156
Hard exudates 275
Hassall-Henle bodies 130
Head posture, abnormal 63, 357, 365, 372
Headache 329
Healing keratitis 94f
sign of 93
Healing, signs of 104
Hearing loss 64, 163
Heavy silicone oils 340
Hemangioma 84
triad of 84
Hemangiopericytoma 84
Hematological disorder, discover signs of 209
Hematuria 423
Hemodynamic crisis 172
Hemoglobin A1C 240
Hemoglobinopathies 240
Hemorrhage
resolution of 220
types of 210
Hemosiderin pigments 210
Hemostatic forceps 442
Hereditary systemic disorders 262
Herpes keratopathy 151
Herpes simplex 410
virus 411
Herpetic disciform keratitis, central 130
Herpetic eye disease 105t
Herpetic footprints 130
Herpetic keratitis 94
Herpetic keratouveitis, recurrent 184
Hertel exophthalmometer 6, 32
Hessburg-Barron trephine 451, 452f
Hexagonal cells, percentage of 131, 230
High myopia 200, 209, 267, 415, 429
Hirschberg test 385
Histochemical stains, types of 125
Histopathological classification 308
Holt-Oram syndrome 402
Homocystinuria 415, 428, 417t
disease 420
Hormone replacement therapy 67
Horner syndrome 21, 53, 54
Hoskin's anatomic 191
Human papilloma virus 10
Human T-lymphotropic virus type 1 290
Hundred-day glaucoma 196
Hyalitis 285
Hyaloid membrane, posterior 317
Hybrid contact lenses 112
Hydraulic theory 34
Hydrodelamination 114, 127
Hydrodelineation 426
Hydrodissection 426
Hydrogel junction 112
Hydrops, acute 141, 144
Hyperglycemia, severity of 236
Hyperlipidemia 237
Hyperlysinemia 415
Hypermature cataract 415
Hyperopic correction 304
Hyperostosis 7
Hypertension 25, 208, 237
absence of 226
family history of 209
Hypertensive retinopathy 240
Hyperthyroidism, signs of 36
Hypertrophic scar 12
Hypertropia 18
Hyphema 184, 192, 218
Hypoparathyroidism 414
Hypoplastic disc 164
Hypopyon 96f
height of 95
role of 104
size of 95
Hypotonia 422
Hypotony 269
I
Iatrogenic break 269
Ice cell 159
Ice pack test 22, 405
Idiopathic juxtafoveal telangiectasis 240
Idiopathic lesions 375
Idiopathic polypoidal
choroidal vasculopathy 213, 215
choroidopathy 275, 276
Idiopathic retinal vasculitis 213
Idiopathic senile flaccidity 39
Immunohistochemistry 47
Immunologic disorders 95
Immunosuppressive treatment, side effects of 43
Impending macular holes 253
Implantable miniature telescope 284
In vivo confocal microscopy 100
Incision biopsy 8, 11
Incomitant esotropia types 391t
Indocyanine green 275
angiography 275
Infection
plus, signs of 100
recurrent 64
Infectious keratitis, diagnosis of 97
Inferior dystopia 37f
Inferior lid retractor laxity 56
Inferior rectus muscle 39
Infiltration, resolution of 101
Inflammation
degree of 195
iris, presence of severe 96
presence of severe 92
signs of 7
Inflammatory disease 316
Inflammatory orbitopathy 39
Infliximab 283
Infusion cannula 458, 458f
Inkblot leak 305
Inner retina, dimpling of 257
Inoperable disease 12
Insulin-dependent
diabetes mellitus 234
diabetic 241
Intact red blood cells 210
Intellectual disability 422
Intercalary membrane 322
Intermediate uveitis 285, 288, 290
causes of 286, 290
Intermittent esotropia 391, 398
classification 398t
Intermittent proteinuria 118
Intermittent pupillary block 429
Internal limiting membrane 234, 256, 257, 317, 460
peeling 256
International Thyroid Eye Disease Society 41
International vitreomacular traction study 316
Interpalpebral calcification 149f
Interpupillary distance 64, 388
Intracameral injection of bevacizumab 194
Intracapsular cataract extraction 446
Intracorneal ring segment insert 112, 114
Intracranial tension 365
Intraepithelial spread of tumor 13
Intralenticular lens aspiration 418, 419f
Intralesional corticosteroids 72
Intraocular cysticercosis 332
Intraocular foreign body 355
removal of 262, 459
Intraocular lens 442
dislocation 344
basis of 344
implantation 429
primary 412
power calculation 412
removal
holding forceps 449
removal 344
Intraocular malignancy 45
Intraocular pressure 7, 96, 136, 171, 182, 184, 192, 193, 202, 209, 263, 265267, 287, 309, 319, 326, 342, 359, 366, 372, 410, 417, 422, 424, 428, 436
measurement 338
range of 176
reduction of 132
symptoms of raised 197
Intraocular surgery 78, 163, 262, 341
Intraocular tumor 7
Intraoperative choroidal 186
Intraoperative complications 336t
Intraretinal edema 253
Intraretinal hemorrhage 330
Intraretinal microvascular abnormalities 242, 310, 311
Intrauterine infections, serology for 411
Intravascular papillary endothelial hyperplasia 72
Intravenous immunoglobulin 383, 406
Intravitreal injection 445
Intravitreal pharmacotherapy 220
Intravitreal ranibizumab, efficacy of 227
Intravitreal steroid 200
Intravitreal triamcinolone acetonide 200
Intrinsic lesions of optic nerve 1
Inverse Bell's phenomenon 18
Inverse glaucoma 429
IOL-holding forceps 449f
Ionizing radiation 10
Iridectomy 441
Irido fundal coloboma 164
Iridocorneal endothelial 157
dystrophy 131, 155
syndrome 154, 157, 157t, 181, 194
Iridophacodonesis 333
Iridoschisis 198
Iris 32, 65, 96, 359, 366, 372, 410, 422, 424, 428, 432, 436
atrophy 157, 158f
progressive 158
segmental 180f
bleeding 184
coloboma 322
diaphragm 199
forceps 441
melanosis 159
nevus syndrome 159
pigment, disruption of 203
root of 194
retrodisplacement of 198f
sphincter tears 198
stop platform 442
vessels
normal 194, 195
sizes of 194
Irregular astigmatism 166
Irrigating wire vectis 448f
Ischemic maculopathy 240
Ischemic mononeuropathy 361
Ischemic optic neuropathy 380
anterior 240
Isolated coloboma 87
Isolated ectopia lentis 415
J
Jaeger lid spatula 455, 455f
Jaw-winking
phenomenon 15
ptosis 54
Jones criteria 100, 104
Juvenile rheumatoid arthritis 290
Juvenile xanthogranuloma 164
Juxtafoveal 281
K
Kaposi's sarcoma 71
Kasabach-Merritt syndrome 84
Kawasaki disease. 290
Kearns-Sayre syndrome 15, 54, 247
Kelley's punch 450, 450f
Keratic precipitates 130, 135, 287
Keratin, result of 79
Keratitis 12, 18, 104, 115
active infectious 103
specific 95
Keratoacanthoma 70
Keratoconus 106, 107, 111, 115, 147, 249
cause of 106
complications of 115
development of 248
diagnosis of 110t
moderate cases of 114
posterior 115
progression 106
severe variants of 144
signs of 116
treatment of 111
Keratoglobus
diagnosis of 143
treatment of 144
Keratome 445, 445f
Keratometry, disadvantages of 110
Keratopathy 61f
causes of exposure 37
exposure 87
Keratoplasty 440, 450
result of 104
Kerrison bone punch 456, 457f
Kimura's spatula 97
King's clamp 453, 453f
Klinefelter's syndrome 428
Knapp's procedure 55
Koloboma 318
Krause-Kivlin syndrome 162
Krimsky test 385
L
Lacrimal fossa
mass, diagnosis of 47, 51
tumors 4
Lacrimal gland 45f
enlargement 40
mass 78
diagnosis of 47t
tumor 3, 6, 44, 46, 48t
Lacrimal sac 29
surgery 456
tumors 3
Lacrimal system 87
patency assessment 61
Lactosyl ceramidosis 301
Lagophthalmos
causes of 76t
degree of 73
etiology of 76
various, severe 75
Lamella reconstruction
anterior 13
posterior 14
Lamellar keratectomy, anterior 103
Lamellar keratoplasty 124, 143, 147, 152, 166
advantage of 126
anterior 150
Lamellar laceration 355
Lamellar macular hole 254, 257
Lamellar therapeutic keratoplasty, anterior 152
Lang's pencil gross streopsis test 389
Larynx, alterations of 89
Laser capsulotomy 262
Laser epithelial keratomileusis 115
Laser peripheral iridotomy 182, 183
complications of 184
Laser photocoagulation 213, 306, 316
Laser trabeculoplasty 176, 198, 201
selective 339
Lash loss 12
Late rosette cataract 437, 438f
Lateral anterior dermoid 78
Lateral canthal
laxity test 61
tendon laxity 57
Lateral tarsal strip procedure 58f
Lattice corneal dystrophy 123, 125, 151
Lattice dystrophy 127
Laxity of facial skin 118
Leber's congenital
amaurosis 248, 251
hereditary optic neuropathy 107
Leber's hereditary optic neuropathy 380
Left exotropia 399f
Left inferior rectus 373
Left lower eyelid 10f
Left upper eyelid 10f
Lens 65, 96, 115, 309, 319, 326, 409, 410, 416, 422, 425, 428, 432, 436
absorptive 285
aspiration 412
coloboma 321
drop 343
epithelial cells 434
hook 443f
posterior
dislocated 341
displacement of 199
posteriorly dislocated 352
size of 344
spatula 447, 447f
technique of removal of 344
type of 112
Lensectomy 412
role of 327
Lenticonus 421
anterior 423t
posterior 422, 423t, 425
Lenticular refractive surgery 112
Leopard skin retinopathy 247
Lesion
anterior 81
diameter of 12
of anterior ethmoidal sinus 3
Less severe occlusion 216
Leukemia 209, 214, 240, 305, 357
Leukemic vitreous infiltration 211
Leukocoria 409
Levator action 53
Levator function 53
measurement of 53
Levator palpebrae superioris 15, 23, 356
function 17
Level of Bowman's membrane 148
Lhermitte sign 377
Lid 318, 325, 338, 358, 365, 372, 378, 401, 422
clamp 455, 455f
coloboma 88
acquired 87
drooping of upper 15
examination of 4
excursion 17
fold, presence of 53
laxity 5860
amount of 56
lower, components of 58
malignancy, signs of 13
margins, ulceration of 10
opening, normal 72f
reconstruction 13, 14t
resection procedures, disadvantages of 59
retraction 37f
signs in proptosis 38t
speculum 440
surgery 455
tumor 9, 10
Life-threatening injuries 31
Light reflex tests 385
Limbal dermoid 87, 163, 163f, 164f, 169f
treatment of 166
Limbic keratoconjunctivitis, superior 69
Limbus-to-limbus corneal thinning 141, 142f
Limited anterior vitrectomy 412
Lingam gopal classification 323t
Lipodermoid 78
Lisch epithelial corneal dystrophy 123
Locus minoris resistentiae 322
Loss of vision, sudden painless 216
Low potency steroid 202
Low vision
aids 285, 348
cause of 226
in coloboma, causes of 323
Lowe's syndrome 190, 414, 421
Lower eyelid
coloboma 87, 89, 164
retractor reinsertion 62
tightening 75
Lower lid 39
coloboma 89t
retraction 39
Luedde's exophthalmometer 6
Lumbar puncture 379
Lyme's disease 74, 290
Lymph node
enlarged 69
involvement 11
Lymphatic malformation 84
Lymphoma 3, 164, 211
M
Macroaneurysm 213
Macula 379
center of 254
Macular atrophy 254
Macular colobomas 320
Macular corneal dystrophy 117, 123
Macular degeneration, signs of age related 152
Macular dystrophy 117, 127, 347f, 348, 348t
Macular edema 211f, 218f, 219, 220, 239, 243, 310
causes of 240
development, pathophysiology of 242
first line of treatment for 229
management of 229
Macular hemorrhage 254, 302
Macular hole 252, 257, 352
formation 260
index 255
management of 256t
surgery for 255
Macular ischemia 219, 228, 310f
Macular swelling, causes of 240
Maddox rod 387
Maddox wing 387
Male graves’ disease 36
Malignant contracted socket 27
Malignant melanoma, manifestation of 14
Malignant mixed tumor 49, 50
Malignant tumor 46, 47
Malpositioning of eyelid 52
Managing macular ischemia 313
Mandibulofacial dysostosis 428
Mannosidosis 414
Map biopsy 13, 69
Marcus Gunn jaw
grading of 21
winking 19, 21
syndrome 23, 54
Marcus Gunn ptosis 55
Marcus Gunn syndrome 15
Marden-Walker syndrome 65
Marfan's disease 417f, 417t, 420
etiopathogenesis of 419
Marfan's syndrome 107, 415417, 420, 428
Margin crease distance 17
Margin limbal distance 17
Margin-reflex distance 17
Margins of ulcer 93
Marked ectropion 58
Mass lesion, reducibility of 4
Mass, reducibility of 4
Mature cataract 415
Maxillary sinus
carcinoma of 3
tumor 3
Mc Donald criteria 382
McNeill-Goldman ring 451
McPherson's forceps 442, 443f
Mechanical debridement 149
Mechanical ectropion 58
management of 59
Mechanical ptosis 24
Media haze, causes of 290
Media opacity, causes of 210, 211
Medial anterior dermoid 78
Medial canthal
laxity test 61
tendon laxity 56, 59
Medial orbital
lymphangioma 3f
wall, fracture of 1
Medial rectus, bilateral 394
Meesmann corneal dystrophy 123
Megalocornea 142, 415
Membrane, type of 263
Membranectomy 434
Membranoproliferative glomerulonephritis 277
Membranous conjunctivitis, severe 62
Menstrual history 64
Mental retardation 417
Mestinon 407
Metabolic disorders 190
Metallic eye speculum 440f
Metastasis 70
Metastatic disease 305
Metastatic tumor 3, 50
Microaneurysm development, pathophysiology of 241
Microphthalmia 18
unilateral 29f
Microphthalmos 164
Micropsia 395
Micropulse 243
Microspherophakia 428
etiology of 430t
Micro-spherophakia, pathogenesis of 431
Microvascular disease 361
Microvascular lesions 375
Mild congenital ectropion 59
Mild contracted socket 27
Mild ptosis 53
Millard-Gubler syndrome 370
Miniature toy test 410
Misdirected third nerve ptosis 19, 55
Mitomycin C 69, 183, 194, 195
use of 166
Mitral valve prolapse 107, 415
Mixed types melanoma 298
Moebius syndrome 402
Mohs’ micrographic
surgery 11
technique 69
Moist socket, treatment of 28
Moll gland of eyelid 67
Monocular diplopia 415
Monster vessels 189
Mooren's ulcer 93, 145
Moraxella 90
Morning glory syndrome 402
Motility 318
deficits 81
Mucocele 78
Mucopolysaccharidoses 154, 161
Muir-Torre syndrome 70
Müller's muscles 75
Multicenter uveitis steroid treatment 289
Multifocal electroretinography 311
Multiple follicular cysts 70
Munson's sign 115
Muscle
biopsy 406
of retraction 76
tone, reduced 64
Musculoskeletal system 330
Musk protein antibodies 405
Mustard's technique 88
Myasthenia gravis 39, 55, 403, 408
Myasthenia, tests to confirm 22
Mycobacteria species 105
Mycobacterium tuberculosis 92
Mycophenolate mofetil 138
Myoepithelioma 47
Myogenic ptosis 23
acquired 23
Myogenic theory 402
Myopia, signs of 267
Myopic crescent 429
Myopic glasses, use of 252
Myotonic dystrophy 54, 414
N
Naffziger's method 3
Nasolacrimal duct stenosis 12
Nasopharyngeal tumor 3
Naugle exophthalmometer 6
Nausea, complaint of 337
Necrotic material 103
Neovascualar glaucoma 181, 192, 198, 216, 217, 219, 222, 230, 238
causes of 195
stages of 193t
Neovascularisation iris 181, 192, 193, 193f, 209, 230, 238, 263, 309
Nephropathy 237
Nerve fiber layer 173
Nettleship's punctum dilator 456f
Neurocysticercosis 331
Neurogenic ptosis 23
acquired 24
Neurologic disorders 39
Neuromyelitis optica 382
Neuroretinal rim 174
Neurotrophic keratitis 230, 238
Neurotrophic ulcer 93
New iris vessels 195
Night blindness 248
N-methyl d-aspartate 176
Nocardia species 90, 105
Nonantigenic 29
Noncontact trephines 452
Nonhealing corneal ulcer 104
Non-healing of epithelial defect 101
Non-insulin-dependent diabetes mellitus 234
Nonpenetrating deep sclerectomy 176
Nonpenetrating glaucoma surgery 176
Nonproliferative diabetic retinopathy 229, 236
Nonrefractive accommodative 392
Nonresolving vitreous hemorrhage 221
Nonspecific orbital inflammatory disease 39
Noonan syndrome 65
Norrie's disease 195
Nose, alterations of 89
Nuclear sclerotic cataracts 257
Nucleus
drop 342
emulsification 426
expression of 447
Nyctalopia 233
Nystagmoid movements 404
Nystagmus 52
surgery for 407
O
Oblique muscles directly 4
Oblique palsy
bilateral superior 376t
unilateral superior 376t
Occasional intermediate uveitis 249
Occlusion therapy 166, 396
Ocular
adnexa 225
associations 107
balance 46
bobbing 404
coherence tomography 315, 316f
cysts 329
dermoids 168
developmental disorders 190
disease 190
severe ischemic 25
epibulbar dermoids 168
examination 3, 188, 273, 286, 292, 325, 351, 357, 365, 372, 378, 400, 428, 431
flutter 404
function 12
hypertension 174
hypotony 269
inflammation, postoperative 205
inflammatory diseases 163, 195
injury, classification of 355
investigation 288
ischemic syndrome 219, 240
lymphangiectasia 72
manifestations of Bell's palsy 76t
motility 31, 365
abnormal 53
limitation of 31
movements 267
myasthenia 404
gravis 403, 406
neoplasms 195
neovascularization 220
pemphigus 62
perforation 167
predisposition 209
response analyzer 111
side effects 291
signs of myasthenia gravis 404t
surface
diseases 134
irregularity 60
surgery 64, 86, 107, 134, 172, 195
symptoms and signs of Bell's palsy 75
torticollis 376t
trauma 30, 60, 172, 174
Oculocerebrorenal syndrome 422
Oculodigital sign 248
Oculomotor abnormalities 18, 55
Oculoplasty 1
procedures 446
Oil tamponade, duration of 340
Oil-filled eyes 335, 340
Okihiro's syndrome 402
Old retinal detachment 247
Olivopontocerebellar atrophy 347
Open-angle glaucoma 171, 309
Ophthalmic artery occlusion 251
Ophthalmic division of trigeminal nerve 4
Ophthalmic examination 36, 300, 309
Ophthalmic instruments 440
Ophthalmic viscoelastic device 427
Ophthalmology, epidemiology of 184
Ophthalmopathy, treatment of 42
Ophthalmoplegia, external 65
Ophthalmoscopic classification 380
Ophthalmoscopy, indirect 301
Optic atrophy 219
Optic canal 8
Optic disc 173, 378
cup, enlargement of 173, 173f
hemorrhage 177
pit 305
types of 323t
Optic foramina, enlargement of 7
Optic nerve
damage, extent of 192
dysfunction, signs of 381
glioma 3f
head
changes 174
examination of 188
hypoplasia 193
Optic neuritis 377, 380
classification of 380
options for 381
treatment trial 377, 381
Optical coherence tomography 279, 282
uses of 240
Oral
hypoglycemic agents 237
pentoxifylline, role of 221
prednisone, short course of 32
Orbicularis muscle weakness 57
Orbicularis muscular tone 61
Orbicularis oculi muscle 76
Orbit excision of tumor 50
Orbit syndrome, expanded 34
Orbital apex, region of 8
Orbital cavity, enlargement of 7
Orbital cellulitis 78, 82
Orbital contents, entrapment of 31
Orbital decompression 43
Orbital extension 12
Orbital fascial connective tissue, restriction of 4
Orbital fat 76
Orbital floor
boundaries of 35
fracture of 30, 31f
repair 34t
Orbital fracture, type of 30
Orbital hemangioma 82
classify 84
Orbital hemorrhage 1
recurrent 2
Orbital implants, types of 30
Orbital mass 8
diagnosis of 8
Orbital myositis 39
Orbital pseudotumor 78
Orbital rim, fracture of 30
Orbital septum 76
Orbital tumors 39
Orbital venography 8
Orbital wall fractures 52
Orbito-lid apposition, grade of 58
Orbitotomy, anterior 49
Orthophoria 394
Ovarian failure, management of 67
Ovarian function, early loss of 65
Ovarian insufficiency, primary 65
Oversized prosthesis, avoidance of 29
P
Pagetoid spread, evidence of 69
Pain proptosis, presence of 2
Pain, symptoms of 129
Palpebral aperture 17
Palpebral conjunctiva 11
Palpebral fissure distance 53
Palpebral preauricular lymph nodes 10
Panretinal photocoagulation 194, 220, 232
Papillae, presence of 107
Papilledema 219
Papillophlebitis 380
Paracentral scotomata 236
Paracentral Staphlococcus aureus 92
Paralytic ectropion 57f
management of 59
Parapapillary atrophy 177
Parathyroid hormone 149
Paresis 368
Parinaud's syndrome 39
Parks-Bielschowsky three-step tests 376
Pars plana lensectomy 312, 418
Pars plana vitrectomy 256, 418
Pars planitis 290
Partial palsy 368
Past ocular disorders 224
Patch graft, types of 104
Patchy iris stromal atrophy 180f
Pathological myopia 267
Paton spatula 452f
Pattern-scanning retinal laser 235
Pediatric cataracts 413
Peek sign 405
Pegaptanib 283
Pellucid marginal
corneal degeneration, diagnosis of 146t
degeneration 110, 144, 146t
Penetrating keratoplasty 112, 118, 132, 143, 147, 152, 189
Penetrating trauma 435
Pentacam 111, 433
Perfluorocarbon liquid 327
role of 327, 344
Perforated corneal ulcer 94f, 100
Peribulbar dermoid, triad of 86
Perifoveal yellowish 301
Periocular tumors 2
Periodic orbital edema 2
Periorbital inflammation 4
Periorbital tumor 54
Peripapillary atrophy 173f
Peripheral anterior synechia 157, 158, 193, 198
Peripheral blood smear 211
Peripheral iridectomy 172, 182, 339
Peripheral retinal
breaks 257
fold 352f
Peripheral ulcerative keratitis 218
Peripheral vision, loss of 250
Peritomy, closure of 268
Permanent loss of vision 72
Permanent prism 361
Permanent tarsorrhaphy 75
Persistent epithelial defect 230
Persistent fetal vasculature 413
Persistent hyperplastic primary vitreous 190, 195, 411
Peters’ anomaly 154, 155, 160, 162
Peters’ plus syndrome 162
Phaces syndrome 84
Phaco needle tip 450f
Phacoemulsification 258
Phacovitrectomy 258
Phenylephrine 231
test 21
Phosphaturia 422
Photocoagulation, adequacy of 212
Photodynamic therapy 12, 283, 307
Photophobia 72, 91, 148, 153, 188, 346, 395, 415
Photopsia 261
Photorefractive keratectomy 115
Phototherapeutic keratectomy 115, 124, 149, 152
complications of 150
Phototherapeutic kerectomy 118
Pierce-Hoskin forceps 442, 443f
Piggyback systems 112
Pigment dispersion syndrome 207
Pigment epithelial
abnormalities 273
detachment 275, 279, 304
Pigment epithelial-derived growth factor 283
Pigment, loss of 246
Pigmentary glaucoma 174, 205
Pigmentation 93
Pigmented basal cell carcinoma 10f
Pigmented choroidal melanoma 294f
Pinch test 56, 60
Placement of bandage contact lens 150
Placental growth factor 233, 283
Plain forceps 441f
Plasma
exchange 383
homocysteine level 219, 226
Plateau iris syndrome 181
Platelet rich plasma, complications of 233
Pleomorphic adenocarcinoma 49
Pleomorphic adenoma 4547, 49, 50, 50t, 51
Plexiform neurofibroma 78
Plural-colobomata 318
Pneumatic cutters 458
Pneumatic retinopexy 269
role of 327
Pneumococcus species 90
Pneumoretinopexy, principles of 269
Polar cataract
classify posterior 427
posterior 424, 425f, 427
Polymerase chain reaction 100
Polymorphous corneal dystrophy, posterior 123, 130, 131, 155
Polymorphous dystrophy, posterior 158
Polypoidal choroidal vasculopathy 305
Poor bell's phenomena 54
Poor cosmetic appearance 25
Poor dilation of pupil 205
Poor functional status of vision 351
Poor orbicularis muscle function 54
Poor tumor differentiation 12
Poor vision 31
management of 284
Poor visual acuity 357
Port-wine stain 187
Position of lacrimal puncta 57
Posner-Schlossman syndrome 290
Post-enucleation socket syndrome 30
Posterior capsular defect, management of 427
Post-laser medication 434
Postpenetrating keratoplasty herpetic keratitis 105
Postperforating injury 101
Postsurgical ptosis 22
Post-traumatic atrophy of lens 437
Post-traumatic impairment 39
Post-vitrectomized eyes 335, 340
Preauricular lymph nodes 4
Pre-Descemet's corneal dystrophy 123
Prednisolone, dose of 43
Preferential hyperacuity perimeter 279
Pre-glaucoma stage 196
Preretinal blood, removal of 459
Pre-retinal hemorrhage 233
Previtelliform 349
Prisms 396
Progressive external ophthalmoplegia, chronic 15, 39
Proliferative diabetic retinopathy 193, 209, 213, 229, 236, 313
Proliferative vitreoretinopathy 261, 265, 270, 352
signs of 352
Prominent corneal nerves 108
Prominent scleral spur 198
Prophylactic peripheral iridectomy 339
Prophylactic treatment 194
Prophylaxis, methods of 272
Propionibacterium acnes 290
Proptosis 1, 3, 8, 18, 36, 82, 83
bilateral 9
cause of 2t, 9
bilateral 2, 2t
consequence of 37
direction of 3
early cases of 3
severe bilateral 37f
subacute course of 45
unilateral 2, 9
Proteinuria 423
Protrusion of eye 3
Provision of low vision aids 282
Proximal tubular acidosis 422
Pseudoepithelial hyperplasia 12
Pseudoesotropia, causes of 399
Pseudoexfoliation 415
glaucoma 174, 198, 203
syndrome 203
Pseudohypopyon 349
Pseudomacular holes 254
Pseudomonas 91, 92, 94
aeruginosa 90
Pseudophakic bullous keratopathy 130
Pseudoproptosis 8
Pseudopterygium, formation of 166, 167
Pseudoptosis 55
causes of 18
Pseudotumor of orbit 55
Psoralen plus ultraviolet A 10
Psychosomatic disorders 424
Pterygium
atypical 164
surgery 446
Ptosis 15, 20, 53, 55, 63, 65, 66f, 404
acquired 19, 19t, 21
amount of 15
classification of 19t, 21
evaluation of 18
measurement of 17
mechanical causes of 16
progression of 15
recurrent episodes of 15
repair 20, 54
section of 52
severe 52
severity of 53, 64
to jaw movement, amount of 52
Pulfrich phenomenon 377
Pulsatile proptosis 8
causes of 9
Pulse steroid therapy 32
Punctal malposition 59
Pupil 73, 96, 319, 333, 359, 366, 372, 378, 410, 416, 422, 424, 428, 432, 436
examination of 7
reaction 263
sparing nerve palsy 363
Pupillary margin 204f
Pupillary reaction 16, 352
Purified protein derivative skin test 288
Putterman's method 17
PXF syndrome 203
Pyogenic granuloma 71, 71f
R
Radial keratotomy 107
Radial optic neurotomy 221
Radiation retinopathy 219, 240
Radiation therapy 42
Radiotherapy, complications of 12
Raised intraocular pressure 176
Randot test 389
Ranibizumab 283
Rare syndromes 87
Real vital dye 256
Recalcitrant cases, management of 28
Recalcitrant macular edema 229
Rectus muscles 7
Rectus-holding forceps, superior 441, 441f
Red eyes 60
Red-free imaging 241
Refraction 429
Refractive accommodative 392
Refractive error 393, 395
Refractive index 336
of vitreous 336
Refractive lens exchange 115
Refractive surgery 107, 115
Refsum disease 247
Regional lymph nodes 4
Regular astigmatism 166
Rehabilitate aphakic patient 345
Reis-Bücklers corneal dystrophy 123
Relative afferent
pathway defect 192, 436
pupillary defect 32, 246, 263, 352, 378, 432
Renal function
measurements 149
test 88, 166, 224, 240, 423
Repetitive nerve stimulation test 406
Residual tumor, locations of 12
Resolution of ulcer, early sign of 94
Respiratory system 300
Retained intraocular foreign body 327, 352, 355
Reticular pseudodrusen 272
Retina 208
attached 212
thinning of 257
Retinal aplasia 251
Retinal artery occlusion, branch 195
Retinal break 265
types of 353
Retinal capillary nonperfusion 241
Retinal change in diabetes, sign of 242
Retinal cryopexy, anterior 194
Retinal cryotherapy, anterior 213
Retinal cyst 263
Retinal detachment 195, 198, 260, 262, 262t, 270, 286, 290, 314, 318, 320, 330, 352, 354, 411, 413, 415, 429, 436
diagnosing 263
in myopes 267
management of 324
presence of 210
surgery for 265t
types of 261t
Retinal dialysis 198, 353
Retinal disorders 262
family history of 351
Retinal folds 269
Retinal ganglion cell 383
Retinal hemorrhages 239f
Retinal hypoxia 196
Retinal incarceration 269
Retinal ischemia
cause of 192
evidence of 193
Retinal laser 237
Retinal mass 263
Retinal necrosis, acute 290
Retinal nerve
fiber layer 174, 177, 383, 384
thickness 174
Retinal pigment
epithelial tear 276
epithelium 260, 272, 294, 306
Retinal surgery 316
Retinal tears 314, 352
Retinal vascular
anomalies 214
occlusive diseases 195, 314
Retinal vasculitis 210
Retinal vein occlusion 216, 240, 252
branch 195, 223
Retinal vessel
distortion of 315f
rupture of normal 215
Retinitis pigmentosa 244, 247, 415
inversus 247
severe 251
sine pigmento 247
stages of 244
Retinitis punctata albescens 247
Retinitis sclopetaria 352
Retinoblastoma 265, 411
Retinochoroidal coloboma 323, 324
Retinopathy 242
decreases, progression of 237
in diabetes 234
of prematurity 190, 195, 210, 262, 411
severity of 237
to anemia 219
Retinoschisis 263
Retinoscopy 422
Retinovascular disease 240
Retroillumination, presence of 184
Retropulsion test 4
Retropulsion theory 34
Rhegmatogenous retinal detachment 257, 260, 265, 325
sign of 342
Ribonucleic acid 233
Rigid gas permeable 107, 111
lenses 111, 144
Ring sctoma, presence of 248
Ring shaped ulcer 93
Rizzuti phenomena 109
Rizzuti's sign 115
Roof of orbit, development of 9
Rosette cataract 436
Rothmund disease 424
Rubella 410, 411
Rubeosis iridis 196
presence of 194
Rupture 355
S
Sabouraud's dextrose agar 98
Saddle-shaped instrument 443
Salmon patch 46
Salzmann nodular degeneration 125
Sandhoff's disease 303
Scanning slit combined 110
Schirmer's test 18, 46, 56, 61, 73
Schnyder corneal dystrophy 123
Schnyder's crystalline dystrophy 117
Schocket scleral depressor 461, 461f
Schroeder classification 426
Schwartz-Jampel syndrome 65
Schwartz-Matsuo syndrome 266
Scissoring of red reflex 107
Sclera 65, 95, 359, 366, 372, 410, 416, 422, 424, 428, 432, 436, 441
Scleral buckling 268, 327
surgery, steps in 268
Scleral involvement 101
Scleral rupture 352
Scleral wounds, signs of old repaired 351
Scleritis 263
posterior 305
Scleroderma 424
Sealed capsular irrigation 435
Sebaceous adenomas 70
Sebaceous carcinoma 69, 70
in systemic disease 71
Sebaceous cell carcinoma 11, 68, 69
Sebaceous epitheliomas 70
Sebaceous gland carcinoma 9, 10f, 13, 67, 68f
Secondary glaucoma 429
Secondary tumor 3
Segment coloboma, posterior 321
Segment cysticercosis, posterior 329
Self-retaining barraquer eye speculum 440
Semi-integrated implants 30
Senile cataract occurs 238
Senile ectropion 57f, 62
Sensory detachment 311
Sentinel lymph node 13
biopsy 13
Serum
angiotensin converting enzyme 219
calcium 149
galactokinase 411
lipids, management of 237
phosphorus 149
uric acid 149
Seven rings of trauma, discuss 439
Severe complicated ptosis, right-sided 24f
Severe contracted socket, management 28
Severe visual loss, harbinger of 229
Shaffer's sign 210
Shallow anterior chamber 179f
Sheridan-Gardiner test 410
Sickle cell disease 209, 214
Silicone brush tip cannula 459
Silicone oil 336
disadvantages of 270
filled eyes 336, 340
induced secondary glaucoma 337
removal 339
role of 258, 327
Singh classification 425
Single-fiber
electromyogram 22
nerve electromyography 406
Sinusotomy 176
Sixth cranial nerve palsy 364
Sixth nerve palsy 368t
causes of 369t
diagnosis of 367t
right 365f, 366f
Sixth nerve, course of 369
Sjögren syndrome 46
Skeletal anomalies 89
Skewed radial axes 116
Skin 76
atrophy 12
biopsy punches 452
Sleep test 405
Sling surgery, complications of 55
Slit-lamp
biomicroscopy 164
examination 61
Small graft 139
Smear examination, result of 97
Smith-Lemli-Opitz syndrome 65
Smokestack leak 305
Smoking 230, 237
history of 36
Snailtrack degeneration 266
Snap-back test 56, 60
Snellen's entropion clamp 455, 455f
Soemmering's ring 432f, 433, 434, 434f
Soft silicone tip cannula 460, 460f
Soft tissue, herniation of 32
Solar retinopathy 254
Spastic entropion
acute 61
causes of 63
Spectacle 144
Specular biomicroscopy 205
Spheroidal degeneration 150, 151f, 153
Spindle cell melanoma 298
Spongiform 311
Squamous cell carcinoma 9, 11, 12, 69, 70
of eyelid 10f
Squamous layer of epidermis 11
Squamous papilloma 72
Squint 351
examination for 372, 401
tests for 359
Staphylococcus aureus 9092
Staphyloma, posterior 429
Stargardt disease 346348, 349t
pathophysiology of 348, 349
Stereopsis 389
lack of 263
Steroid 36, 101, 137
implants, discuss 291
induced glaucoma 200202
prevention of 201
side effects of 291
therapy, discontinuation of 200
type of 202
Steroid-sparing immunosuppressive drugs 42
Steven tenotomy scissors 446, 446f
Steven-Johnson syndrome 29, 60, 62, 134
Stiles-Crawford effect 307
Still's syndrome 150
Stimulation deprivation amblyopia 164
Stool examination 331
Strabismic amblyopia 164
Strabismus 16, 82, 88, 164, 166, 356, 401, 409, 421
Striational antibodies 405
Stromal dystrophy 123
types of 118
Stromal scars 145
Sturge-Weber syndrome 173, 185, 187t, 190
Subcapsular cataract, posterior 246, 249, 335, 425
Subcapsular opacities 436
Subconjunctival hemorrhage 32, 197
Subconjunctival mass 7
Subcutaneous emphysema 31
Subcutaneous tissue 76
Subepithelial dystrophy 122
Subepithelial fibrillary lines 109
Subepithelial mucinous corneal dystrophy 123
Subhyaloid hemorrhage 184
Subluxated lens 414
etiology of 415t
Subluxation of lens 87
Subretinal bleed 269, 352
Subretinal choroidal neovascularization 305
Subretinal cysts 330
Subretinal exudates 263
Subretinal fluid 254, 306f
internal drainage of 459
Subretinal hemorrhages 198
Sulcus defect, superior 18
Sulfite oxide deficiency 415
Sunglasses 42
preferred, type of 251
Superficial foreign body 355
Superficial keratectomy 124, 149
Superior rectus, right 373
Superotemporal displacement 416f
Surgery
complications of 167, 328, 402
for ectropion, selection of 58
performed, type of 25
type of 195
Swimming goggles 407
Swiss cheese 46, 51
Sympathomimetic drugs 39
Syndromic retinitis pigmentosa 245t
Syndromic variants 87
Synechiae
anterior 135
posterior 184
Synkinetic ptosis 19
Synoptophore 359
test 389
Systemic antibiotic 101
Systemic anticoagulants, role of 221
Systemic antimicrobial agents, use of 95
Systemic chemotherapy 297
Systemic disease 91
Systemic disorders 190
Systemic inflammatory disease 2
Systemic medications 172, 179
Systemic myasthenia 404
Systemic steroid 137
therapy 79
T
Taenia solium 329, 331
Tarsoconjunctiva 58
combined, excision of 58
scarring of 58
Tarsorrhaphy, limitation of 75
Tay-Sachs disease 303
Tears, artificial 42
Tectonic patch graft 104
Teflon block 452
Telangiectasia's, small 68
Tenon's patch graft 104
Tensilon test 22, 405, 406
Tension glaucoma, normal 200
Teratoma 78, 81
Terrien's marginal degeneration 146
Terson's syndrome 210, 212, 214
Therapeutic keratectomy microkeratome system 454f
Therapeutic penetrating keratoplasty 104
Thermal injury 29
Thiazide diuretics 68
Thick hyaloid 263
Thick mature epiretinal membrane 315f
Thiel-Behnke corneal dystrophy 123
Third cranial nerve
course of 363
palsy 53, 356, 362
Third nerve palsy, causes of 364
Thudichum nasal speculum 456, 457f
Thymectomy 407
Thyroid
associated ophthalmopathy 35
disorder 2
eye disease 37f
function test 7, 39, 73, 406, 411
gland dysfunction, treatment of 42
involved 408
ophthalmopathy 7
treatment of 42
related ophthalmopathy 3, 35
Thyroxine 44
Tight medial rectus muscle 394
Tissue
adhesives 103
diagnosis 296
Titmus test 389
Tobacco
chewing 230
dust 263, 267
smoking 43
Tonometry 78, 164
Topical mitomycin C application 12
Topical steroids 137, 182
Torsion, evaluation of 373
Total detachment of retina 251
Toxocariasis 411
Toxoplasmosis 410, 411
Trabecular meshwork 171
Trabeculectomy 186, 195, 201
Trabeculotomy 186, 189
Trachoma 62
Traction on retina 458
Tractional retinal detachment 211, 219, 230, 231, 260, 262
Transient ischemic attacks 233
Transmitted cerebral pulsations 2
Transpupillary thermotherapy 283, 307
Trans-scleral
cyclophotocoagulation 195
panretinal cryotherapy 194
Trauma
effects of 32
history of 52, 91, 208, 333, 341, 351
seven rings of 354
signs of 197, 198, 254, 263, 351
Traumatic hyphema 199
Traumatic ptosis 23
Traumatic retinal detachment 350, 354
Treacher-Collins syndrome 89
Trephines, types of 451
Treponemal serology 219
Triamcinolone acetonide 256
Trocar and cannula 457
Tube surgery 189
Tumor 208
diameter 12
extent of 50
recurrent 12
type of 4
Tunica vasculosa lentis 427
Tunnel vision 249
diagnosis of 250
Typical optic neuritis 381t
U
Uhthoff phenomenon 377
Ulcer
base of 93
edge of 98
influence, depth of 90
progressive 93
Umbilical hernia 89
Universal metallic eye speculum 440
Upper eyelid
coloboma 89, 164
fornix 87
retraction 18, 43, 75, 444
Upper lid
coloboma 88, 89t
retraction 39
Urine examination 411, 423
Usher syndrome 247
Utrata capsulorhexis forceps 442, 442f
Uveal inflammation 96
Uveitis 212
anterior 184
chronic 247
posterior 263, 316
signs of 205, 225
V
van Herick's method 183
Vannas scissors 447f
Variable intraocular pressure 37
Vascular disease 316
Vascular endothelial growth factor 233, 234, 307
Vascular occlusion 209
type of 216
Vascular sclerosis, gross signs of 219
Vascularization of host cornea 139
Vasculitis 240, 301
Vaso-inhibitory factors 197
Vasospastic diseases 172
Vein occlusion, branch 227
Venous beading 239f
Venous malformations 85
Venous stasis retinopathy 219
Vernal keratoconjunctivitis 106
Viral keratitis 103t, 104
management of 102
Virectomy, indications of 213
Viscocanalostomy 176
Viscosity silicone oils, use of low 340
Vision 350
achieved, quality of 114
acute loss of 230
better quality of 112
blurring of 73, 77, 86, 117, 118, 144, 148, 153, 163, 233, 285
deterioration of 10
diminution of 15, 63, 77, 86, 90, 163, 329, 346
dimness of 192
gradual loss of 230
gradual painless loss of 337
in eye, gradual painless loss of 272
loss 1, 91, 314, 341
causes of 290, 333, 234
progression of 172
preceding pain, dimness of 192
progressive loss of 118
sudden diminution of 300
sudden loss of 144, 272
Visual acuity 3, 16, 52, 253, 286, 304, 318, 325, 357, 365, 372, 378, 385, 410, 416, 431, 436
best corrected 106, 135, 230, 237, 341, 436
best spectacle corrected 431
decreased 91
gain of 228
loss of 346
significant loss of 91
uncorrected 230, 341
Visual axis 151, 166
opacification 413
Visual complaints, progress of 223
Visual correction, type of 111, 144
Visual evoked
potential 379
response 353, 379, 411
Visual field 220
defect 257, 261
loss of 233
loss, advanced 172
normal 174
testing 21, 347
Visual function 251
Visual loss 118, 172, 179, 261
significant 124
Visual potential 195
assessment 437
Visual rehabilitation 249, 348
postoperative 412
Visual requirements 317
Visual symptoms 184
Visualization of gadolinium-enhanced lesions 8
Vitamin
A
palmitate 249
supplements, avoidance of high-dose 348
C 282
E 282
Vitelliform 349
Vitelliruptive 349
Vitrectomy 268, 324
benefits of early 234
cutter 458, 458f
in diabetic patients 234
role of 221
sequential 258
with scleral buckle, role of 328
year of 333
Vitreomacular adhesion 225, 309
Vitreomacular tension, symptoms of 252
Vitreomacular traction 240, 315
syndrome 254
Vitreoretinal pathology 337
Vitreoretinal procedure 333
Vitreoretinal scissors 460, 460f
Vitreoretinal surgery 333, 440, 457
Vitreoretinal traction 268
Vitreous 314, 378, 410, 422, 425, 428, 432, 437
anterior 210, 285, 326
base 264
avulsion 352
body 309
degeneration 211
detachment, posterior 210, 212, 214, 231, 261, 263, 314, 326
gel, syneresis of 210
haze 290
hemorrhage 198, 208, 210, 219, 230, 263, 290, 314, 352, 411
causes of 215t
resorption of 233
incarceration 327
loss 205, 262
membranes 263
pigments 210, 342
prolapse 269
severe 233
spooling 458
substitute 270t
function of 258
role of 258
syneresis 263
Vitritis 211
Vogt Kayanagi Harada's disease 195
Vogt's striae 108
Volkmann's ischemic contracture 31
Voltage-gated calcium channel antibodies 406
Vomiting 329
von Graefe knife 445
von Graefe retractor 443, 443f
W
Waardenburg syndrome 422
Warburg syndrome 322
Warthin tumor 47
Watzke-Allen test 254
Wavy vision 346
Weill-Marchesani syndrome 415t, 428
Westcott scissors 447, 447f
White blood cells 95
White eyed blowout fracture 34
Wildervanck syndrome 402
Wilson disease 414
Wire vectis 448f
Worm's eye 3
Worsen diabetic retinopathy 234
Worsening of ptosis 15
Wound closure 412
X
Xanthelasma 237
Y
Yellow-gold spherules 151f
Z
Zeaxanthin 282
Zellweger's syndrome 190
Zimmerman's hypothesis 299
Zonular cataract 409, 410f
retroillumination 410f
Zonular coloboma inferiorly 319f
Zonular dialysis 205
Zonular opacity 436
Zonular weakness, cause of 206
Zonules 416
Zygomatic complex fracture 31
×
Chapter Notes

Save Clear


OculoplastyCHAPTER 1

Long Cases
 
PROPTOSIS
Varsha Varshney, Amar Pujari, Mandeep S Bajaj
 
INTRODUCTION
Proptosis is defined as an abnormal forward protrusion of one or both eyeballs with respect to the orbit. Among adults, the usual distance from the lateral orbital rim to the corneal apex is approximately 16–21 mm.1,2 Proptosis is said to be present when following criteria are present.
  1. Protrusion more than 22 mm beyond the orbital rim.
  2. An asymmetry of more than 2 mm between the eyes.
Proptosis is one of the common topics given as a long case in exams.
 
HISTORY
 
Chief Complaint
A case of proptosis usually presents with following complaints:
  • Protrusion of one or both the eyes.
  • Loss of vision: It indicates optic nerve compression/involvement [by intrinsic lesions of optic nerve such as meningiomas or optic nerve gliomas or external compression, e.g. tumors located in the orbital apex, such as a hemangioma and Graves’ disease] or induced astigmatism due to globe compression or exposure keratopathy.
  • Rare presentation includes diplopia, restricted ocular motility, redness/pain/discharge due to associated exposure keratopathy.
 
History of Present Illness
Following points must be noted while examining a case of proptosis.
  • Age of onset: Age of onset can point towards the probable diagnosis as shown in Table 1.
  • Nature of onset
    • Sudden (hours to days): It suggests inflammatory and infective process, trauma (orbital emphysema, fracture of the medial orbital wall, orbital hemorrhage) or rupture of ethmoidal mucocele.
    • Gradual (over many months to years): It suggests tumors, lymphoma—proliferative disorders.
  • Progression
    • Slow continuous: It suggests tumor, however, gradual progression with sudden increase in proptosis can harbor malignant transformation2
      Table 1   Causes of proptosis based on the age of onset
      Newborn
      Children
      Young adults
      Middle age
      Senile
      • Orbital cellulitis
      • Orbital neoplasm
      • Rhabdomyosarcoma
      • Hemangioma
      • Dermoid cyst
      • Orbital cellulitis
      • Optic nerve glioma
      • Craniosynostosis lymphomas
      • Thyroid ophthalmopathy
      • Pseudotumor
      • Orbital cellulitis
      • Osteomas
      • Infiltrative tumors
      • Pseudotumor
      • Endocrine
      • Malignant lymphomas/leukemias
      • Optic nerve sheath meningiomas
      • Mucocele
      • Malignant and metastatic tumor of orbit
      • Pseudotumor
      • Leukemia
      • Lymphomas
      • Sarcomas
    • Rapid progression: It indicates infection/inflammation/hemorrhage/malignant transformation
    • Intermittent: It intermittent proptosis can be due to following causes
      • Periodic orbital edema
      • Recurrent orbital hemorrhage/chocolate cyst and highly vascular tumors
      • Increases during attacks of common cold/upper respiratory infections– lymphangioma
      • Postural (associated with bending forward) or with Valsalva suggests orbital varices.
      • Increases on crying capillary hemangiomas in young children
  • Pain: Depending upon presence of pain proptosis can be
    • Painful: Infective, acute inflammations, chocolate cyst, orbital hemorrhages
    • Painless: Tumor, endocrinopathy (Pain can be there in some malignant tumors that show perineural spread, such as adenoid cystic carcinoma of the lacrimal gland).
  • Laterality
    Unilateral: Unilateral proptosis is seen in tumors, cysts, and vascular anomalies
    Bilateral: The different causes of bilateral proptosis are summarized in Table 2.
  • Special characteristics: Rarely, patient can give a history of feeling the pulsation within the orbit or periorbital area. An example of such cases include; AV malformation, carotico- cavernous fistula and saccular aneurysm of ophthalmic artery or due to transmitted cerebral pulsations in conditions associated with deficient orbital roof such as congenital meningocele or meningoencephalocele, and traumatic or operative hiatus.
Table 2   Common causes of bilateral proptosis
Pathology
Etiologies
Inflammations
  • Thyroid orbitopathy
  • Wegener's granulomatosis
  • Idiopathic inflammatory pseudotumor
  • Myositis
  • Sarcoidosis
  • Sjögren's syndrome
Neoplasia
  • Lymphoma
  • Leukemia
  • Metastatic carcinoma
  • Optic nerve glioma
Vascular lesions
Arteriovenous shunts
Varix
 
History of Past Illness
A careful past history can point towards the provisional diagnosis. Following points must be asked in history of past illness:
  • Systemic inflammatory disease such as thyroid disorder, sarcoidosis
  • Malignancy—Lungs, breasts, prostate
  • Trauma
  • Periocular tumors
 
Past Surgical History
Prior periorbital surgery or history of surgery for intraocular malignancy such as malignant melanoma might point to the possibility of orbital extension or metastasis.3
 
EXAMINATION
 
General Examination/Specific Systemic Examination
Look for signs of Graves’ disease/Sarcoidosis/Any malignancy/Any infective foci.
 
Ocular Examination
Following points must be noted in ocular examination:
 
Visual Acuity
In general, visual acuity is not affected with orbital diseases except in cases with optic nerve compression, refractive changes due to pressure on back of the eyeball or exposure keratopathy.
 
Eyeball
A case of proptosis should be examined under following headings:
 
A. Inspection
  • Head posture, facial asymmetry, shape of the skull
  • Protrusion of the eye: In unilateral cases, there will be obvious disparity between the two eyes. In bilateral cases, some difficulty may be there in early cases of proptosis. Following two methods are useful in detection of proptosis:
    1. Naffziger's method: Relative protrusion can be observed by simply standing behind a seated patient and gazing downward (tangentially) toward the chin from the forehead to assess protrusion of the eye beyond the orbital rim.
    2. Worm's eye view: It is similar to Naffziger's method but the difference is that the examiner examines up from below with the patient's head-tilted back.
  • The direction of proptosis: The direction of proptosis can indicate the probable etiology. The direction can be following:
    • Axial: Thyroid related ophthalmopathy, Glioma of optic nerve (Fig. 1), Optic nerve sheath, meningioma, cavernous hemangioma.
    • Nonaxial/Eccentric:
      • Down and out: Dermoid, dermolipoma, frontal and ethmoidal mucocele, meningocele
      • Down and in: Lacrimal gland tumor, Dermoid
      • Upwards: Carcinoma of maxillary sinus, lacrimal sac tumors, Lymphoma, maxillary sinus tumor, metastatic tumors
      • Outwards: Lesion of anterior ethmoidal sinus, nasopharyngeal tumor, lymphangioma (Fig. 2), lethal midline granuloma, Metastatic tumors, secondary tumor
        zoom view
        Fig. 1: Axial proptosis due to optic nerve glioma
        zoom view
        Fig. 2: Abaxial proptosis due to medial orbital lymphangioma
      • 4Medial displacement: Dermoid cyst, lacrimal fossa tumors, and cysts, sphenoid wing meningioma
  • Laterality: Unilateral or bilateral
  • Ocular motility: Ocular motility disturbance can be due to:
    • Involvement of the rectus or oblique muscles directly
    • Affecting nerve supply of rectus or oblique muscles
    • From restriction of the orbital fascial connective tissue septae
  • Eyelids: Lids can be affected by direct involvement of the levator muscle or the third cranial nerve, or because of associated proptosis. Examination of lid is especially important in thyroid-associated eye disease. The various lid signs and their terminology has been described in Table 3.
  • Periorbital inflammation: Look for inflammatory signs (erythema, edema, chemosis, dilated vessels) of the periorbital structures that can be associated with infections and acute inflammatory diseases.
  • Nose/roof of mouth (sinus disease or when intranasal source is suspected) and neck (for goiter)
  • Valsalva: Orbital varix or highly vascular lesions such as capillary hemangiomas will enlarge with increases in arterial pressure. This can be elicited with the Valsalva maneuver or by asking the patient to bend forward.
  • Lagophthalmos, Bell's phenomena, and exposure keratopathy
 
B. Palpation
It should be carried out to confirm the findings of inspection and also for following:
  • Retropulsion: In a retropulsion test, thumb/two fingers/palm is used to gently push on the globe through the upper eyelid to know compressibility/resistance of the tumor. While hemangioma, lymphangioma, orbital varices give a soft consistency; optic nerve glioma may give a firm consistency. Resistance to retropulsion suggests retrobulbar tumor or thyroid ophthalmopathy.
  • Orbital thrill/Pulsation: Place your index and middle finger over the orbit (with closed lids) and observe following
    • True pulsation: Finger will raise and separated
    • Transmitted pulsation—finger will raise only
  • Swelling/mass around the eyeball: If present its size, site, shape, consistency, fixity (skin/bone/muscle), signs of inflammation (Rubor—redness; Color—raised temperature, check with your back surface of hand; Dolor—pain/tenderness; Tumor—swelling; Functionless— loss of function), and overlying skin changes must be noted carefully.
  • Regional lymph nodes: Palpate the submandibular and preauricular lymph nodes.
    • Medial and central lower lid (along the facial vein), medial upper lid, central upper lid and medial canthus drains into the submandibular lymph nodes
    • Lateral upper lid and lateral lower lid drain into the preauricular parotid lymph nodes.
  • Orbital rim/margin with index finger to look for any mass, or erosion (malignancy), irregularity (previous trauma)
  • Reducibility of the mass: Check for reducibility of the mass lesion. It can provide a clue about the probable etiology, e.g. vascular tumors, schwannoma are reducible masses but lacrimal gland tumor, pseudotumor are not reducible.
  • Infraorbital/supraorbital anesthesia: Perineural invasion by a tumor can result in pain, numbness, paresthesia in infraorbital or supraorbital area. Adenoid cystic carcinoma (ACC) of the lacrimal glands often presents with orbital pain and paresthesia, since this type of tumor is frequently associated with perineural spreading (Remember, the lacrimal nerve is the smallest of the three branches of the ophthalmic division of the trigeminal nerve. It passes through the lacrimal gland then pierces the upper lid and supplies the supraorbital area. It provides sensory innervations for the lacrimal gland, conjunctiva, and the lateral upper eyelids and adjacent supraorbital area, and partly the zygomaticotemporal area just adjacent to the orbital rim. Hence, in case ACC, these areas must be checked for sensory abnormalities).5
Table 3   Lid signs in proptosis
Sign
Description
Abadie's sign
Elevator muscle of upper eyelid is spastic
Ballett's sign
Paralysis of one or more extraocular muscle (EOM)
Beck's sign
Abnormal intense pulsation of retina's arteries
Boston's sign
Jerky movements of upper lid on lower gaze
Cowen's sign
Extensive hippus of consensual pupillary reflex
Dalrymple's sign
Upper eyelid retraction
Enroth's sign
Edema especially of the upper eyelid
Gifford's sign
Difficulty in eversion of upper lid
Goldzieher's sign
Deep injection of conjunctiva, especially temporal
Griffith's sign
Lower lid lag on upward gaze
Hertoghe's sign
Loss of eyebrows laterally
Jellinek's sign
Superior eyelid folds is hyperpigmented
Joffroy's sign
Absent creases in the forehead on upward gaze
Jendrassik's sign
Abduction and rotation of eyeball is limited also
Knies’ sign
Uneven pupillary dilatation in dim light
Kocher's sign
Spasmodic retraction of upper lid on fixation
Loewi's sign
Quick mydriasis after instillation of 1:1000 adrenaline
Mann's sign
Eyes seem to be situated at different levels because of tanned skin
Means’ sign
Increased scleral show on upgaze (globe lag)
Moebius's sign
Lack of convergence
Payne/Trousseau sign
Dislocation of globe
Pochin's sign
Reduced amplitude of blinking
Rieseman's sign
Bruit over the eyelid
Movement's cap phenomenon
Eyeball movements are performed difficultly, abruptly and incompletely
Rosenbach's sign
Eyelids are animated by thin tremors when closed
Saiton's sign
Frontalis contraction after cessation of levator activity
Snellen-Rieseman's sign
When placing the stethoscope's capsule over closed eyelids’ a systolic murmur could be heard
Stellwag's sign
Incomplete and infrequent blinking
Suker's sign
Inability to maintain fixation on extreme lateral gaze
Tellas's sign
Inferior eyelid might be hyperpigmented
Topolanski's sign
Around insertion areas of the four rectus muscles of the eyeball a vascular band network is noticed and this network joins the four insertion points.
von Graefe's sign
Upper lid lag on downgaze
Wilder's sign
Jerking of the eye on movement from abduction to adduction
 
6C. Auscultation
Auscultation is done with the help of a stethoscope, over eyeball and temporal area, to look for bruit. A bruit can be seen in cases of AV malformation.
 
D. Transillumination
It is helpful in evaluating anterior orbital lesions. It is usually performed using a penlight. Interpretation is based on the opacity of the mass relative to surrounding tissues as follows:
  • Bright: The area lights up more than the surrounding tissues, e.g. cyst filled with clear liquid, Dacryops
  • Equal: The area lights up to the same degree as surrounding tissues, e.g. lipoma.
  • Indeterminate: The area seems darker than surrounding tissues to a variable degree, e.g. inclusion cyst, dermoid cyst
  • Dark: The area is clearly opaque, creating a shadow effect. Aneurysm, e.g. solid tumors (lacrimal gland tumors), osteoma.
  • Remember some clinicians interpret transillumination test as positive or negative; in that case first two categories can be considered positive while the last two as negative transillumination test
 
E. Exophthalmometry
The most common type exophthalmometer used in clinical practice is Hertel's exophthalmometer. Other Exophthalmometers used in clinical practice are Luedde scale, Naugle exophthalmometer, and Gormaz exophthalmometer. However, in the absence of any of these an ordinary transparent ruler can be used to measure proptosis. The measurement, in Hertel's exophthalmometer, is done from the lateral orbital rim to the anterior corneal surface. A difference of greater than 2 mm between an individual patient's eyes suggests proptosis. Procedure—following points must be considered while using Hertel's exophthalmometer.
  • The patient and the examiner must be at the same level, eye to eye.
  • Locate the orbital notch with patients eyes closed (the deepest points on orbital rim) on the temporal side of the orbital rim near the lateral canthus.
  • The prisms or the mirrors are slide across the bar to adjust the footplates to fit on the orbital rim. The exophthalmometer is opened so that the grooves are placed in the orbital notch.
  • The separation of the exophthalmometer (baseline reading) is very important and must always be noted and the exophthalmometer must always be set at that separation on future or repeated readings.
  • The patient is then asked to open the eyes and look straight ahead.
  • Look into the exophthalmometer, the red lines should overlap to avoid the parallax. Look into the mirrors located at each end of the exophthalmometer. Now note the millimeter mark corresponding to the corneal apex position on the scale and the readings on the cross bar for baseline reading.
Limitations: There are several limitations to Hertel's exophthalmometer. The readings are unreliable in presence of poor fixation, uncooperative patients with convergence or repeated head movements. In addition, in presence of depressed/fractured lateral orbital rim, it cannot be done.
Luedde's exophthalmometer: It is a transparent plastic ruler which is thicker than the normal ruler. It has several advantages over an ordinary plastic ruler such as, the reading starts from the apex and the apex fits the orbital notch accurately. It is more accurate than Hertel's exophthalmometer in presence of facial asymmetry.
Naugle exophthalmometer: It uses fixation points slightly above and below the superior and inferior orbital rims (cheekbones and forehead). Naugle exophthalmometer measures the difference in proptosis between the two eyes rather than absolute measure with the Hertel method. It is preferred in presence of an orbital fracture or after lateral orbitotomy.
Interpretation: The normal range is 12–21 mm. A difference of > 2 mm between the two eyes is significant. In pediatric age group the readings may vary depending upon the age; <4 years old 7(13.2 mm), 5–8 years old (14.4 mm), 9–12 years old (15.2 mm) and 13–17 years old (16.2 mm).
Conjunctiva: Examine carefully (with a slit-lamp) for following:
  • Dilated or tortuous blood vessels, chemosis, dilated lymphatics-vascular malformation, inflammation
  • Hyperemia over the insertions of the horizontal rectus muscles—one of the earliest sign of thyroid eye disease
  • Corkscrew shaped tortuous dilated episcleral vessels-high-flow AV malformations or a carotid cavernous fistula
  • Subconjunctival mass—anterior extension of a deeper orbital tumor, such as a lymphoma or intraocular melanoma with extraskeletal orbital extension (a dark subconjunctival mass)
Cornea: Look for signs of exposure keratopathy. Fluorescein staining can reveal early stage of exposure keratopathy showing multiple punctate defects. The corneal sensation must be checked in all cases of proptosis.
Sclera/Iris: Careful examination is carried out to look for signs of inflammation and any nodules
Pupil: Examination of pupil is extremely important since it often provides the first clue of a probable optic nerve involvement. The presence of relative afferent pathway defect (RAPD) must be ruled out in all cases. Any intraconal mass compressing optic nerve or any tumor intrinsic to optic nerve (optic nerve glioma) can lead to RAPD.
Intraocular pressure (IOP): If possible IOP should be recorded in different gaze (ideally in al gazes and at least in primary and superior) especially in cases of suspected thyroid ophthalmopathy (in case of thyroid ophthalmopathy variable IOP in upgaze can be an early sign due to an involvement of inferior rectus muscle). The IOP may vary due to the restricted mobility of extraocular muscles.
Lens: Lens is usually clear except the effect of aging.
Vitreous: Usually normal except in cases of intraocular tumor.
Fundus: Carefully look for signs of globe compression such as venous engorgement, choroidal folds, and papilledema or optic atrophy. Choroidal folds and opticociliary shunts may be seen in patients with meningiomas. In cases with cavernous sinus thrombosis retinal edema, exudates and engorged retinal veins can be seen due to increased venous pressure.
 
DIFFERENTIAL DIAGNOSIS
Based upon history and clinical examination the differential diagnosis has to be made. The commonly seen diseases in clinical practice includes:
  • Vascular (cavernous fistula, cavernous hemangiomas)
  • Endocrine (thyroid eye disease)
  • Inflammatory (orbital cellulitis, orbital inflammatory disease, orbital cysticercosis)
  • Neoplastic (ON glioma, ON meningiomas, lacrimal gland tumors, frontal sinus mucocele)
 
Investigations
The line of investigation depends on upon the differential diagnosis arrived. Following tests must be done routinely in a case of proptosis.
  • Thyroid function tests,
  • Complete blood count (CBC), peripheral smear (leukemia/lymphoma), erythrocyte sedimentation rate (ESR), blood sugar
 
Specific Tests
 
Imaging Technique
Noninvasive techniques
  • Plain X-rays: It is often the initial radiological examination, especially when other modalities are not available. Commonly done exposures are in the Caldwell view, the Water's view, a lateral view and the Rhese view (for optic foramina). The findings of orbital diseases in X-ray include enlargement of orbital cavity, calcification, hyperostosis and enlargement of optic foramina.
  • Ultrasonography: It is a nonradiational noninvasive, completely safe and extremely valuable initial scanning procedure for orbital lesions. In the diagnosis of orbital lesions, it is superior to computed tomography (CT) scanning in actual tissue diagnosis and can usually differentiate between solid, cystic, 8infiltrative and spongy masses. The limitations of USG are; limited ability to evaluate orbital bones, periorbital sinuses, and the orbital apex due to poor distance penetration.
  • Computed tomography scanning: It is extremely helpful in determining the location and size of an orbital mass. A combination of axial and coronal cuts enables a three-dimensional visualization. In addition to globe, extraocular muscles, and optic nerves visualization it can show areas adjacent to the orbits such as orbital walls, cranial cavity, paranasal sinuses and nasal cavity. Mass lesions in the orbit usually appear as an abnormal density within the typically low-density orbital fat. The lesion may be well defined with sharp borders (e.g. cavernous hemangioma), or infiltrative with diffuse borders (e.g. pseudotumor). CT can give information about adjacent bone erosion (suggestive of malignancy), remolding or fossa formation (encapsulated benign lacrimal gland tumor such as pleomorphic adenoma, encapsulated malignant lacrimal gland tumor, orbital dermoid), or displacement of adjacent bony orbital walls. Its main disadvantage is the inability to distinguish between pathologically soft tissue masses which are radiologically isodense. In addition, the risk of radiation exposure (in contrast to MRI) is always there. Examples of few findings on CT in cases of proptosis includes.
  • Magnetic resonance imaging (MRI): MRI is very sensitive for detecting differences between normal and abnormal tissues. Advantages of MRI over CT includes:
    • Better soft tissue visualization, especially in the region of the orbital apex, optic canal, and cavernous sinus.
    • Various fat suppression techniques allow the visualization of gadolinium-enhanced lesions that is often difficult to see the normally high signal-generating orbital fat.
    • Better tissue differentiation
    • No radiation exposure.
Invasive procedures
  • Orbital venography: Useful in orbital varix. It confirms the diagnosis and also outlines the size and extent of the lesion that helps in proper surgical planning.
  • Carotid angiography: Useful in cases of pulsatile proptosis and in those associated with a bruit or thrill. It helps to identify the location and extent of ophthalmic artery aneurysms, AV malformations. It is also helpful in identifying the feeding vessels thus, for planning surgery in cases of vascular orbital tumors.
 
Histopathology
The exact diagnosis of many orbital lesions cannot be made without the help of histopathological studies which can be accomplished by following techniques:
  • Fine-needle aspiration biopsy (FNAB): Quick, reliable, and relatively accurate. The biopsy aspirate is obtained under direct vision (when there is an obvious mass)or guided by CT/USG using a 23-gauge needle.
  • Incisional biopsy: The scope of incisional biopsy in the diagnosis of orbital masses is not clearly defined. It is often contraindicated due to the risk of tumor seeding or increased risk of malignant transformation (e.g. benign lacrimal gland tumor).
  • Excisional biopsy: It is the procedure of choice especially when the mass is well encapsulated or circumscribed. It is performed by orbitotomy.
 
CLASSIFICATION/STAGING/SCORING
It depends upon the individual disease.
 
MANAGEMENT
The management depends upon the individual diseases.
 
VIVA QUESTIONS
Q.1. Explain pseudoproptosis.
Ans. Pseudoproptosis is either the simulation of an abnormal prominence of the eye or a true asymmetry that is not caused by a mass, a vascular abnormality, or an inflammatory process.9
Causes are multiple and include:
Enlarged globe
  • Myopia
  • Trauma
  • Glaucoma
Asymmetric orbital size
  • Congenital
  • Postradiation
  • Postsurgical
Asymmetric palpebral fissure
  • Contralateral ptosis
  • Lid retraction
  • Facial nerve paralysis
  • Lid scar, ectropion, entropion
Extraocular muscle abnormalities
  • Postsurgical muscle recession
  • Paralysis or paresis
Contralateral enophthalmos
  • Contralateral orbital fracture
  • Contralateral small globe
  • Contralateral cicatricial tumor
Physiological proptosis: It is proptosis in infants due to the fact that orbital cavities do not attain their full size so rapidly as the eyes.
Q.2. Most common cause of proptosis.
Ans.
  • Unilateral proptosis: Thyroid eye disease
  • Bilateral proptosis: Thyroid eye disease
Q.3. Causes of pulsatile proptosis.
Ans. True vascular
  • Carotid cavernous fistula
  • AV fistula in the orbit between ophthalmic artery and orbital vein, in the neck—Carotid artery and jugular vein
  • Highly vascular orbital tumor
  • Orbital varies
Transmitted
  • Congenital failure in the development of roof of the orbit, e.g. encephalocele/encephalomyelocele
  • Traumatic or operative hiatus in orbit roof resulting in the formation of meningocele
Q.4. The 6 P's of the orbital history and physical examination are
Ans.Useful in the diagnostic process
  • Pain
  • Proptosis
  • Progression
  • Palpation
  • Pulsation
  • Periocular changes
Q.5. Few facts about orbit dimensions.
Ans.
  • Volume—30 cc
  • Horizontal entrance width—40 mm
  • Height at orbital rim—35 mm
  • Orbital depth (rim to the optic strut)—45–55 mm
  • Orbital segment of optic nerve—25 mm
References
  1. Albert DM, Miller JW, Azar DT. Albert & Jakobiec's Principles and Practice of Ophthalmology. 2008.
  1. Bowling B. Kanski's Clinical ophthalmology: A systematic approach, 8th edn. Edinburgh: Elsevier,  2015.
 
LID TUMORS
Sapna Raghuwanshi, Amar Pujari, Mandeep S Bajaj
 
INTRODUCTION
Lid tumors can arise from the epidermis, dermis, or adnexal structures of the eyelid. Malignant lesions are common around the eyes, because many are induced by sun exposure or develop from sun-related benign lesions. Typically, most of these are small and slowly growing. The common periocular malignancies are basal cell carcinoma (BCC), squamous cell carcinoma (SCC), sebaceous gland carcinoma (SGC), and malignant melanoma. In western literature, basal cell carcinoma constitute around 85% of the cases because of fair complexion. Whereas in India all three major variants constitute around 33% each.1,210
 
HISTORY
 
Demography
Commonly, present in 5th to 6 decade of life with site of involvement:
  • Involvement pattern in basal CC (lower eyelid, caruncle, upper eyelid, lateral canthus) (Fig. 1)
  • Involvement pattern in sebaceous CC (upper eyelid, lower eyelid) (Fig. 2)
  • Involvement pattern in squamous CC (lower eyelid, caruncle, upper eyelid) (Fig. 3).
 
Chief Complaints
A case of lid tumor can present with presents with following complaints:
  • Slow, generally painless growth on lid margins
    zoom view
    Fig. 1: Pigmented basal cell carcinoma of the left lower eyelid
    zoom view
    Fig. 2: Sebaceous gland carcinoma of the left upper eyelid
  • Ulceration of the lid margins
  • Loss of eyelashes
  • Distortion of eyelid margin
  • Ectropion/retraction (secondary due to lid growth or skin contracture)
  • Increasing pigmentation of eye lid margins
  • Palpebral preauricular lymph nodes
  • Dilated blood vessels (telangiectasias).
 
History of Present Illness
Following points must be noted for a lid tumor:
  • Onset: Common lid tumors are insidious in onset
  • Progression: Progression is usually slow
  • Any deterioration of vision
  • Diplopia or restriction of eye movements (frozen globe)
  • Any other swelling around the head and neck region (preauricular and cervical lymph node for metastasis)
  • Systemic features of metastasis.
 
Past History
Predisposing factors: Following are the predisposing factor for lid tumours:
  • Ultraviolet exposure (UV-B)
  • Ionizing radiation
  • Arsenic exposure
  • Psoralen plus ultraviolet A (PUVA)
  • Human papilloma virus
  • Genetic diseases (albinism, xeroderma pigmentosum risk factor for BCC)
    zoom view
    Fig. 3: Squamous cell carcinoma of the eyelid with extensive periorbital involvement
  • 11Recurrent Chalazion (could be sebaceous cell carcinoma)
  • Previous surgery for eyelid malignancies
  • Topical chemotherapy.
 
Important Examination Findings
  • Eyelid: Careful examination of the eyelid for signs of lid malignancy
  • Eyelid should be examined carefully for depth or plane of involvement, lid margin involvement, and medial and lateral canthal involvement.
  • Bulbar and palpebral conjunctiva: In cases of sebaceous cell carcinoma to look for “Pagetoid spread” (Intraepithelial spread of tumor cells).
  • Cornea: Again in case of sebaceous cell carcinoma
  • Look for scleral invasion and globe integrity: In cases where the tumor is extending beyond the orbital septum.
 
DIAGNOSIS
The best way to confirm the diagnosis is excision biopsy.
 
Histopathology
  • BCC: The tumor cells arise from the basal layer of the epidermis. The cells proliferate downwards thus exhibiting palisading at the periphery.
  • Squamous cell carcinoma: The tumor arises from the squamous layer of epidermis, well differentiated tumors show characteristic keratin ‘pearls’.
  • Sebaceous cell carcinoma: The tumor shows irregular lobules consisting of sheets of cells with varying degrees of sebaceous differentiation. The malignant cells show foamy, multivacuolated cytoplasm, secondary to intracytoplasmic lipid.
 
Biopsy Methods
  • Incision biopsy: Where only a part of the tumor is removed using either blade or biopsy punch.
  • Excision biopsy: Here the entire tumor is removed.
  • Impression cytology: After drying the area to be examined, nitrocellulose filter paper is applied, and then firmly presses with Goldmann tonometer head. Peel off paper with forceps and place in appropriate fixative solution for examination.
  • Fine needle aspiration cytology: In case of large tumor as a preliminary modality or in cases of lymph node involvement.
 
Imaging
Computed tomography is indicated in cases with postseptal extension to know the extent of orbital spread and bony involvement. PET and SPECT in cases to study the status of sentinel lymph nodes.
 
STAGING
Staging is required to plan the treatment strategy and prognosis. The most common staging system for carcinoma of the eyelid is the TNM system.
The following information applies for all three common eyelid carcinomas, but melanoma of the eyelid is staged in the manner as skin melanomas.
TNM stands for tumor, nodes, and metastasis. This describes the size of the primary tumor, the number and location of any regional lymph nodes metastatic foci anywhere in the body.
 
MANAGEMENT
 
Surgical Excision with Standard Frozen Section Control
This technique is performed by noting the clinical boundaries of the tumor edges and excising an additional 3 mm cuff of normal-appearing tissue followed by histopathological assessment of the excised specimen for tumor free margins.
 
Mohs’ Micrographic Surgery
This technique involves removal of the gross mass of the tumor plus a small peripheral margin of normal tissue. A thin layer of tissue, approximately 2 mm thick, is further excised from the entire base and edges of the wound. The initial specimen is divided into 4–7 µm thick portions on glass slides; the edges are marked with different colored dyes to maintain orientation. Frozen sections are 12obtained from the under surface and skin edge of each specimen. Locations of residual tumor are marked on a map and only those areas are re-excised. Surgical resection is continued until there is a microscopically proven tumor-free plane. The defect is then reconstructed by the oculoplastic surgeon.
 
Cryosurgery
It is useful for small lesions, but less effective for larger and deeply invasive tumors.
 
Contraindications of Cryosurgery
  • Involvement of the conjunctival fornix
  • Fixation of tumor to periosteum
  • Sensory or motor denervation
  • Cold intolerance—cryoglobulinemia or cold urticaria
  • Deeply pigmented skin
  • Indistinct margins
  • Diameter of lesion >10 mm
  • Sclerosing or multicentric type.
 
Complications
  • Depigmentation
  • Hyperpigmentation
  • Eyelid notching
  • Hypertrophic scar
  • Pseudoepithelial hyperplasia
  • Ectropion
  • Punctal and canalicular stenosis
  • Lash loss.
With conjunctival involvement: The adjuvant methods followed are
  • Local application of cryotherapy
  • Topical mitomycin C application.
Extensive disease: Exenteration for orbital involvement and extended exenteration with radical neck dissection for metastatic disease beyond the orbit with lymph node involvement.
Radiotherapy:
Indication of radiotherapy includes following:
  • Inoperable disease
  • Multiple medical problems
  • Elderly patients unable to tolerate surgical resection
  • Patients in whom surgery will result in extensive disfigurement with potential loss of useful ocular function.
    Complications of radiotherapy includes:
    • Skin atrophy
    • Ectropion
    • Entropion
    • Nasolacrimal duct stenosis
    • Keratitis
    • Conjunctival keratinization
    • Cataract
    • Loss of eyelashes
    • Globe perforation
  • Chemotherapy: Used in cases of systemic involvement
  • Others like photodynamic therapy and CO2 LASER treatment are rarely used.
 
Prognosis
Following are the poor prognostic factor:
  • Duration of symptoms >6 months
  • Vascular and lymphatic infiltration
  • Orbital extension
  • Poor tumor differentiation
  • Multicentric origin intraepithelial carcinomatous changes of the conjunctiva, cornea, or skin
  • Location in the upper eyelid
  • Tumor diameter >2 cm
  • Location on central face or ears
  • Long-standing presence prior to initial treatment
  • Incomplete excision
  • Aggressive subtype
  • Perineural or perivascular involvement
  • Recurrent tumor:
    • Basal cell carcinoma (BCC) of the eyelid rarely spreads to lymph nodes or other organs, so the prognosis for this type of tumor is usually very good.
    • Squamous cell carcinoma (SCC) can be more aggressive than BCC and can spread to the orbit, lymph nodes or other organs. However, the prognosis is good if SCC of the eyelid is detected early and can be completely removed.
    • The mortality rate (the number of people who die from the disease each year) for 13sebaceous gland carcinoma of the eyelid is about 5%–10%. However, sebaceous gland tumors are often not diagnosed early and have a high rate of recurrence and spread (metastasis).
 
VIVA QUESTIONS
Q.1. Key points of individual malignancies.
Ans. Basal cell carcinoma:
  • It is the common type of eyelid tumour.
  • Usually affects adult population.
  • Commonly involve the lower eyelid due to exposure to sunlight.
Inherited conditions predisposing to BCC:
  • Albinism
  • Xeroderma pigmentosum
  • Basal cell nevus syndrome or Gorlin syndrome
  • Bazex syndrome
  • Rombo syndrome
Sebaceous gland carcinoma (SGC) (Sebaceous gland carcinoma, Meibomian gland carcinoma)[also see short case]:
Important clinical points:
  • These arise from the Meibomian glands in the eyelid.
  • It is more often in elderly women than men and often present late due to less malignant course.
  • These tumors commonly arise from the upper eyelid followed by the lower eyelid the caruncle and bulbar conjunctiva. The upper eyelid is more prone due to more number of Meibomian glands in the upper eyelid (20–25) as compared to (15–20) in lower eyelid.
  • SGC can be multifocal due to peculiar pattern of spread called “Pagetoid spread” where there is an intraepithelial spread of the tumor.
  • The gross appearance resembles yellowish nodular mass
  • May resemble blepharoconjunctivitis.
  • Can involve the orbit and regional lymph nodes.
Q.2. Signs of lid malignancy
Ans. Following are the signs:
  • Destruction of lid architecture
  • Ulceration of the lid margins
  • Loss of eyelashes
  • Distortion of eyelid margin
  • Ectropion/retraction (secondary due to lid growth or skin contracture)
  • Increasing pigmentation of eye lid margins
  • Palpebral preauricular lymph nodes
  • Dilated blood vessels (telangiectasias)
Q.3. Differentiation of three lid malignancies based on clinical findings
Ans. See Text
Q.4. Sentinel lymph node biopsy
Ans. Sentinel lymph node (SLN) biopsy is used for identifying the microscopic nodal metastasis from a malignant tumor. Tumors may preferentially spread to a first draining or “sentinel” lymph node before they spread to distant sites.3
Preoperatively: 99mTc-Sulfur colloid (t1/2 6 hours) will be injected around the tumor followed by hybrid SPECT/CT is performed to locate sentinel lymph nodes (Preauricular, intraparotid and submandibular).
Intraoperatively: In the first setting based on the previous SPECT/CT images maximal radioactive counts were identified by hand held gamma probe, followed by injection of 1% isosulfan blue dye perilesionally followed by gentle massage to augment lymphatic drainage. An incision will be made over the area of highest radioactive count and lymph nodes are dissected. In second setting eyelid tumors are excised and reconstruction done.
Q.5. Map biopsy
Ans. See text
Q.6. Reconstruction of the eyelid defects.
Ans. Lid reconstruction:
Anterior lamella reconstruction:
  • Primary closure14
  • Full thickness skin grafts
  • Musculocutaneous flap
  • Advancement flap
  • Transposition flap
  • Rhombic flap.
Posterior lamella reconstruction
  • Buccal mucosa (preferred)
  • Hard palate mucosa
  • Tarsoconjunctival graft.
Full thickness lid defect—See Table 1.
Q.7. Different histological types of BCC
Ans. See Table 2
Q.8. Different lid manifestation of Malignant Melanoma
Ans. See Table 3
Table 1   Full thickness lid reconstruction
Size of the lid defect
Repair
<25%
Direct closure
25–50%
Direct closure with cantholysis
33–66%
Semicircular flap (Tenzel flap) alone or along with the periosteal flap
50–75%
Cutler Beard (Upper eyelid defect)
Hughes procedure (Lower eyelid defect)
75–100%
Lower eyelid (Tarsoconjunctival flap with skin grafting)
Upper eyelid (Median forehead flap with mucus membrane grafting
Table 2   Lid manifestation of BCC
Nodular-ulcerative
Pigmented
Morphea or sclerosing
Superficial
Fibroepithelioma
  • Most common lesion
    Pink or pearly papule or nodule
  • Overlying telangiectatic vessels present
  • Central ulceration with rolled border ‘rodent ulcer’
  • Similar to the noduloulcerative type in morphology
  • Brown or black pigmentation
  • More common in dark complexion persons
  • Least common
  • Flat, indurated, yellow-pink plaque with ill-defined borders
  • Aggressive and may invade the dermis deeply
  • Invade into the paranasal sinuses and orbit
  • Mimic blepharoconjunctivitis
  • Scaling patch with a raised pearly border
  • Arise on the trunk rather than the eyelid
  • Pedunculated or sessile smooth, pink nodule.
  • Arise on the trunk rather than the eyelid
Table 3   Lid manifestation of malignant melanoma
Lentigo maligna melanoma
Superficial spreading melanoma
Nodular melanoma
Acral lentiginous melanoma
Slowly expanding pigmented, flat, nonpalpable, tan to brown macule with irregular borders
Plaque with irregular outline, variable pigmentation
  • Blue-black nodule with normal surrounding skin
  • May be non-pigmented
Occurs on the palms, soles, and distal phalanges as well as on the mucous membranes
15
References
  1. Albert DM, Miller JW, Azar DT. Albert & Jakobiec's principles and practice of ophthalmology; 2008.
  1. Bowling B. Kanski's Clinical Ophthalmology: A Systematic Approach, 8th edn. Edinburgh: Elsevier;  2015.
  1. Lokdarshi G, Vuthaluru S, Pushker N, Kumar R, Kashyap S, Mathur S. Sentinel lymph node biopsy in malignant eyelid tumor: hybrid single photon emission computed tomography/computed tomography and dual dye technique. Am J Ophthalmol. 2016;162:199–200.
 
PTOSIS
Aditi Dubey, Amar Pujari
 
INTRODUCTION
Drooping of the eyelid is known as ptosis or blepharoptosis. It is a common pathology seen in all age group. It is given as both long and short case in the examination. The most important part in a case of ptosis is its examination.
 
HISTORY
 
Chief Complaints
A case of ptosis can present with following complaints:
  • Adult patient commonly complaints of drooping of the upper lid.
  • In children, parents often complaints of an affected eye being smaller in comparison to another eye.
  • Diminution of vision is not a common complaint and amblyopia is associated only with severe ptosis due to stimulus deprivation. The prevalence of amblyopia is around 12% to 20 % in cases with severe congenital ptosis.14
 
History of Present Illness
It is important to note about following points:
  • The age of onset and duration (congenital or acquired)
  • Progression of ptosis (chronic progressive external ophthalmoplegia)
  • Any head position (e.g. chin lift)
  • Associated symptoms can often indicate the underlying cause.
    For example:
    • Associated double vision points towards 3rd cranial nerve palsy/aberrant regene- ration.
    • Alteration in an amount of ptosis with jaw movements (jaw-winking phenomenon) can be seen in Marcus Gunn syndrome.
    • Difficulty in deglutition is seen in oculopharyngeal muscular dystrophy.
    • Limitation of ocular motility can be seen in myasthenia gravis and Kearns-Sayre syndrome.
    • Worsening of ptosis as the day progresses is seen in myasthenia gravis (Levator palpebrae superioris (LPS) weakness compensated for by Müller's muscle in myasthenia, which fatigues with progression of the day)
    • Associated recurrent allergic conjunctivitis can cause mechanical ptosis.
 
Past History
Following points must be noted in history in a ptosis case:
  • Recurrent episodes of ptosis can be seen in recurrent 3rd nerve palsy, e.g. ischemic neuropathy associated with diabetes or hypertension.
  • Contact lens wear (ill-fitting contact lens may cause blepharospasm and small palpebral aperture which may be confused with ptosis)
  • Drug intake such as neostigmine can point towards the possible diagnosis.16
  • Trauma (lid or facial trauma mat cause scarring or LPS damage) to rule out traumatic ptosis
  • Medical conditions, e.g. myasthenia gravis, myotonia, muscular dystrophies, diabetes and hypertension.
  • Spectacle use or amblyopia therapy during childhood is important when there is associated vision loss.
 
Past Medical History
Diabetes mellitus (DM), hypertension, bleeding diathesis (important for surgery), recurrent stye/chalazion/vernal keratoconjunctivitis (mechanical ptosis), any neurological disorder must be ruled out.
 
Past Surgical History
Peribulbar block for any intraocular surgery, cataract surgery can lead to ptosis. Any previous squint surgery or ptosis surgery must be recorded.
 
Birth History
Such as pregnancy, delivery, neonatal period, and early development are important in congenital ptosis. Birth history such as instrument/forceps delivery can lead to ptosis point towards the cause of ptosis.
 
Family History
Positive family history can be present in blepharophimosis syndrome.
 
EXAMINATION
Visual acuity: Visual acuity and refractive error must be assessed in all cases of congenital or childhood ptosis in order to identify and treat the child with concomitant amblyopia. Amblyopia can result from anisometropia, high astigmatism, strabismus or occlusion of a pupil. Amblyopia occurs in approximately 12–20% of patients with congenital ptosis.
Facial symmetry and orbit: Look for following:
  • Head posture: Chin elevation is seen in cases of bilateral moderate to severe ptosis.
  • Frontalis overaction: Raised eyebrows to compensate for ptosis.
  • Pseudoptosis: Ipsilateral microphthalmos or contralateral lid retraction can lead to pseudoptosis.
  • The presence of strabismus must be ruled out by Cover-Uncover test.
  • Extraocular muscle function should also be assessed in primary and secondary gazes. The presence of motility abnormalities is seen in congenital conditions such as double elevator palsy (combined superior rectus and LPS muscle maldevelopment), congenital oculomotor palsy and acquired conditions such as ocular or systemic MG, chronic progressive external ophthalmoplegia, oculopharyngeal dystrophy, and oculomotor palsy with or without aberrant regeneration.
Eyelid: Following points must be noted:
  • If ptosis is unilateral (Figs 1 and 2) or bilateral
  • Signs of previous trauma such as eyelid scar
  • Mechanical causes of ptosis such as eyelid tumors, multiple chalazia, giant papillae
  • Lagophthalmos—important for surgical planning
  • Pupillary reaction—pupillary involvement can be seen in cases of 3rd CN palsy and Horner's syndrome
  • The position of the lids should be noted in the different position of gaze. Variability of ptosis in the different position of gaze is an indication of aberrant regeneration after third-nerve palsy.
  • Note also the speed of saccades: slow saccades are indicative of myopathic muscles.
    zoom view
    Fig. 1: Simple severe congenital ptosis before surgery
    17
    zoom view
    Fig. 2: Simple severe congenital ptosis after surgery
  • Variation in the amount of ptosis with extraocular muscle or jaw muscle movements (synkinesis) should be noted. Synkinesis may be seen in Marcus Gunn jaw-winking ptosis, aberrant regeneration of the 3rd CN or the VII CN, and some types of Duane syndrome.
 
Measurement of Ptosis
  • Margin-reflex distance 1 (MRD1): It is the distance from the upper eyelid margin to the corneal light reflex in the primary position. It is the single most important measurement in describing the amount of ptosis. The MRD1 is also checked in the reading position.
  • Margin-reflex distance 2 (MRD2): The MRD2 is the distance from the corneal light reflex to the lower eyelid margin. MRD2 is a measure of lower eyelid retraction (or scleral show).
  • Palpebral aperture: The vertical interpalpebral fissure is measured at the widest point between the lower eyelid and the upper eyelid. This measurement is taken with the patient fixating on a distant object in primary gaze. Normally, the palpebral fissure height in males, is 7–10 mm, and in females, it is 8–12 mm.
    Remember the sum of the MRD1 and the MRD2 should equal the vertical interpalpebral fissure height.
  • Margin crease distance (MCD): It is the distance from the upper eyelid crease to the eyelid margin. The insertion of fibers from the LPS muscle into the skin contributes to the formation of the upper eyelid crease. High, duplicated or asymmetric creases may indicate an abnormal position of the levator aponeurosis. The upper eyelid crease is 8–9 mm in males and 9–11 mm in females. The crease is usually elevated in patients with involutional ptosis and is often shallow or absent in patients with congenital ptosis. The height of the crease on the normal side should be measured and compared to the ptotic eyelid in downgaze. In patients, when more than one lid crease is present, the most prominent one should be considered.
  • Levator Palpebrae Superioris (LPS) function
    Berke's Method (lid excursion): LPS function is estimated by measuring the upper eyelid excursion from downgaze to upgaze with frontalis muscle function negated. Fixating the brow with digital pressure minimizes contributions from accessory elevators of the eyelids such as the frontalis muscle. Failure to negate the influence of the frontalis muscle results in an overestimation of LPS function. LPS function can be graded according to Beard's classification
    • Normal: >15 mm
    • Good: 12 –14 mm
    • Fair: 5–11 mm
    • Poor: <4 mm
Putterman's method: This is carried out by the measurement of the distance between the middle of upper lid margin to the 6'o clock limbus in extreme upgaze. This is also known as the margin limbal distance (MLD). Normal is about 9.0 mm.
Assessment in children:
  • Assessment of LPS function in small children is a difficult task, as the child allows no formal evaluation. Following methods may help
  • The presence of lid fold and increase or decrease in its size on a movement of the eyelid gives us a clue to the LPS action.
  • The presence of anomalous head posture like the child throwing his head back suggests a poor LPS action.
  • Iliff test: This test can be performed in the first year of life to evaluate the levator function. The upper eyelid of the child is everted as the child looks down. If the levator action is good, lid reverts on its own.18
Lagophthalmos: The patient should be assessed for lagophthalmos and if it is present, the degree should be noted, checking head position, chin elevation, brow position, and brow action in attempted upgaze. Lagophthalmos and poor tear film quantity or quality may predispose a patient to complications of ptosis repair such as dryness and exposure keratitis.
Bell's phenomenon: it is an upward and outward movement of the eye when an attempt is made to close the eyes. Bell's phenomenon is a normal defense reflex.
  • Demonstration: Ask the patient to close the eye forcibly (as if the patient wants to sleep). The examiner then lifts the patient's upper eyelid manually. In a patient with a normal Bell's phenomenon, the globe will rotate upwards and outwards and the eyelid will cover the cornea.
  • Significance: If a patient does not have a good Bell's phenomenon, a cautious ptosis correction should be undertaken to prevent subsequent corneal exposure, especially when planning for sling surgery.
  • Bell's phenomenon is graded into three grades:
    1. Good: Less than one-third of cornea visible
    2. Fair: One-third to one-half of the cornea visible
    3. Poor: More than one-half of cornea visible
  • Inverse Bell's phenomenon: If the cornea is not in upgaze or if it moves to other position of gaze, such as downgaze on closing the eyes than it is called inverse Bell's.
 
Cornea
  • The corneal sensation must be checked in all cases. A normal corneal sensation is essential for normal blink reflex and prevention of exposure keratitis the following surgery.
  • Quantity and quality of the tear film must be documented in the initial examination. Schirmer test, tear break-up time (TBUT) and Tear meniscus must be recorded in all cases of ptosis. Dry eye syndrome is a contraindication for ptosis surgery; especially sling surgeries as it may cause corneal damage postoperatively.
Pupil: Pupillary examination is important in the evaluation of ptosis. Pupil abnormalities are present in some acquired and congenital conditions associated with ptosis (e.g. Horner syndrome, cranial nerve III palsy). Miosis that is most apparent in dim illumination is seen in Horner syndrome and mydriasis is seen in some cases of 3rd CN palsy.
Fundus: Fundus examination after mydriasis is essential for any concomitant fundus abnormality.
Rest of the findings such as iris, lens, sclera and IOP are usually within normal limits.
 
DIFFERENTIAL DIAGNOSIS
A case of ptosis must be differentiated from pseudoptosis. Pseudoptosis is apparent eyelid drooping due to ocular or adnexal diseases should be differentiated from true ptosis. On elevating the ptotic lid, the other eyelid droops slightly in true ptosis while remains at the same level in pseudoptosis. Causes of pseudoptosis includes following:
  • Unilateral:
    • Hypertropia
    • Enophthalmos
    • Microphthalmia
    • Anophthalmia
    • Phthisis bulbi
    • Superior sulcus defect
    • Dermatochalasis
  • Contralateral:
    • Upper eyelid retraction
    • Proptosis
    • Buphthalmos
 
INVESTIGATION
A case of ptosis usually does not need any special investigation other than routine tests done before surgery. Investigations like visual field may be required in special situations as discussed under Viva questions.
 
CLASSIFICATION
Based on the onset
  • Congenital ptosis: Ptosis present since birth.
    It can be further categorized into following:
    • Congenital simple ptosis
    • Complicated
      • With oculomotor abnormalities
      • With blepharophimosis syndrome19
      • Synkinetic ptosis
        • Marcus Gunn Jaw Winking
        • Misdirected third nerve ptosis
  • Acquired ptosis: True acquired ptosis is the result of some disturbance of the upper lid retractors, the levator or Müller's muscle, or both, and is best classified according to its primary cause, which includes; mechanical (Fig. 3), myogenic, neurogenic, and aponeurotic. Differentiating points between congenital and acquired ptosis are given in Table 1
Bases on pathogenesis: As shown in Table 2.
 
STAGING/SCORING
The amount of ptosis can be determined by:
  • The difference in MRD 1 of the two sides in unilateral cases
  • The difference from normal in bilateral cases gives the amount of ptosis.
zoom view
Fig. 3: Right upper eyelid mechanical ptosis
Table 1   Differences between congenital and acquired ptosis
Parameters
Congenital ptosis
Acquired ptosis
MRD 1
Mild-to-severe ptosis
Mild-to-severe ptosis
Upper eyelid crease
Weak or absent crease in normal position
Higher than normal crease
LPS function
Reduced
Near normal
Downgaze
Eyelid lag (lid lag sign)
Eyelid drop
Palpebral aperture
Greater in downgaze
Less in downgaze
Abbreviations: MRD, margin-reflex distance; LPS, levator palpabrae superioris
Table 2   Classification of ptosis based on pathogenesis
Type
Mechanism
Example
Myogenic
  • Maldevelopment of elevators
  • Myopathic conditions involving the LPS or myoneural junction.
  • Direct damage to LPS or myoneural junctions
  • Congenital simple ptosis
  • Blepharophimosis syndrome
  • Double elevator palsy
  • Congenital ocular fibrosis syndrome
  • Chronic progressive external ophthalmoplegia
  • Oculopharyngeal dystrophy
  • Muscular dystrophy
  • Traumatic
Neurologic
  • Dysfunction of the third cranial nerve
  • Dysfunction of sympathetic innervation to Müller's muscle
  • Aberrant regeneration after oculomotor nerve palsies
  • Oculomotor nerve palsy
  • Horner's syndrome
  • Myasthenia gravis
  • Marcus Gunn jaw-winking phenomenon20
Aponeurotic
  • Dehiscence in the central part of the aponeurosis
  • Disinsertion of the aponeurosis from the tarsus
  • Thinning and stretching, termed attenuation or rarefaction, of the aponeurosis attenuation or rarefaction, of the aponeurosis
  • Involutional (aponeurosis dehiscence)
  • Post-traumatic
  • Post-surgical blepharochalasis
  • Chronic, recurrent edema
  • Pregnancy
  • Chronic ocular inflammation
  • Rigid contact lens wear
Mechanical
  • Excessive weight
  • Dermatochalasis
  • Eyelid mass
  • Giant papillae/VKC
  • Multiple chalazion
  • Orbital mass
  • Scarring
 
Grading
  • Mild ptosis—2 mm or less
  • Moderate ptosis—3 mm
  • Severe ptosis—4 mm or more.
 
MANAGEMENT
The aim of the surgery is to lift the ptotic lid above the pupillary aperture when the eyes are in the primary position. The height of the two lids regardless of whether the ptosis is unilateral or bilateral should be equal. There should also be adequate mobility of the lid when blinking, a normal lid fold and no diplopia. The surgical procedures and their indications are as follows:
  • Fasanella Servat Operation
    • Mild ptosis (<2 mm or less)
    • Levator action >10 mm
    • Well defined lid fold—no excess skin
  • Levator resection
    • Mild/moderate ptosis
    • Levator action ≥ 4 mm
  • Brow suspension ptosis repair
    • Severe ptosis
    • Levator action <4 mm
    • Jaw-winking ptosis or blepharophimosis syndrome
The treatment of congenital ptosis has been described in detail in chapter congenital ptosis (short case). The treatment of acquired ptosis depends upon the cause.
 
VIVA QUESTIONS
Q.1. What are visual problems due to ptosis?
Ans.
  • Ptosis is a common cause of reversible peripheral visual loss. Although, the superior visual field is most often involved, central vision can also be affected. Patients with ptosis complain of difficulty with reading because the ptosis worsens in downgaze. Ptosis has also been shown to decrease the overall amount of light reaching the macula and, therefore, can reduce visual acuity, especially at night.
  • The restricted peripheral visual field is a contraindication for issuing of driving license and jobs requiring broader fields in western countries. Visual fields less than 10 degrees in the better eye with the best correction are considered as legal blindness.
Q.2. What is the normal position of UL?
Ans. The vertical interpalpebral fissure is measured at the widest point between the lower eyelid and the upper eyelid. Normally, the UL should cover 1/6th or 2 mm of the cornea and lower lid should just touch the limbus.21
Q.3. Classification of ptosis
Ans. Ptosis can be classified as following:
  • Based on onset: Congenital or acquired.
  • Based on etiopathogenesis: Myogenic, aponeurotic, neurogenic, mechanical and traumatic.
    The most common type of congenital ptosis results from a poorly developed levator muscle LPS (myogenic). The most common type of acquired ptosis is that caused by stretching or disinsertion of the levator aponeurosis (aponeurotic).
Q.4. Differentiate between congenital and acquired ptosis
Ans. See Table 1
Q.5. What is blepharophimosis epicanthus inversus ptosis syndrome (BPES)?
Ans. Details in short cases.
Q.6. What are Bells phenomenon and its grading?
Ans. See Examination
Q.7. The importance of Hering's law in ptosis
Ans. In cases with bilateral ptosis and with one side having marked ptosis compared to the other side, following surgical correction on the greater ptotic side, the side with minimal ptosis may droop more. This is due to the Hering's law. This is important to predict the postoperative results of ptosis surgery. Patient has to be warned that the contralateral eye may droop following correction of the greater ptotic lid to avoid any postoperative unrealistic expectation by the patient.
Q.8. What should be the sequence of surgery if ptosis and strabismus coexist?
Ans. Since correction of the strabismus may relieve the ptosis, strabismus surgery should be performed before treatment of ptosis. An exception may be made for cosmetically acceptable strabismus for which the only ptosis needs to be treated. If the patient has horizontal strabismus with ptosis, surgery for both strabismus and ptosis can be performed at the same sitting because the result of one is unlikely to influence the result of the other.
Q.9. How to demonstrate and grade Marcus Gunn Jaw winking?
Ans. Synkinesis is best demonstrated by having the patient move the jaw to the opposite side of the ptotic eye, but widely opening the mouth or moving the jaw forward would also elevate the eyelid.
Grading of Marcus Gunn jaw-winking phenomenon is based on the amplitude of lid movement:
  • Mild—2 mm or less
  • Moderate—3–6 mm
  • Severe—7 mm or more
Q.10. What are the other ancillary tests that should be done in ptosis?
Ans.
  • Visual field testing with the eyelids untaped (in the natural, ptotic state) and taped (artificially elevated) helps determine the patient's level of functional visual impairment. Comparison of the taped and untaped visual fields gives an estimate of the superior visual field improvement that can be anticipated following surgery.
  • Pharmacologic testing may be helpful in confirming the clinical diagnosis of Horner syndrome, myasthenia gravis. Fluctuating ptosis that seems to worsen with fatigue or prolonged upgaze, especially when accompanied by diplopia or other clinical signs of systemic MG.
  • Phenylephrine test: One drop of phenylephrine 2.5% is installed in the upper fornix, stimulates the alpha-receptors in Müller's muscle, causing its contraction and hence lid elevation. It is possible to demonstrate the potential outcome of surgery to a patient with a positive response to phenylephrine. One can also unmask a coexisting ptosis in the so-called normal eye, which appeared normal due to increased LPS stimulation (Hering's law).
Q.11. The significance of pupillary examination in ptosis.
Ans.
  • A small pupil (miotic) indicates Horner's syndrome22
  • A large pupil might be a sign of third-nerve palsy.
  • In aberrant third-nerve palsy, the size of a pupil may change in different position of gaze.
Q.12. What is MRD3?
Ans. MRD3 is the distance from the ocular light reflex to the central UL margin when the patient looks in extreme upgaze. In unilateral ptosis, the difference between normal and abnormal MRD3 multiplied by 3 approximately shows the amount of LPS that must be resected.
Q.13. What are the bedside tests to rule out myasthenic ptosis?
Ans.
  • Fatigability: Ask the patient to look up and down for about 1 min to induce fatigue or sustained upgaze for 1 min will achieve the same result. Progressive ptosis will ensue in a myasthenic patient. MRD1 is measured before and after these fatigue tests. Note that myasthenic patients might also develop diplopia with this test.
  • Cogan's twitch sign: The patient is first asked to look down for 15 s. A small upshoot of the eyelid is noted as a myasthenic patient then moves back to the primary position.
  • Ice-pack test: Apply an ice-filled glove to the affected eye for 10 min. In a myasthenic patient, the ptosis improves by 2 mm or more.
Tests to confirm Myasthenia:
  • Edrophonium chloride test or Tensilon test: This test is done in doubtful cases where an acquired ptosis due to Myasthenia Gravis is suspected. In adults, 2 mg of edrophonium is injected slowly in 15–30 seconds. The needle is left in situ and the remaining 8 mg is injected slowly if no untoward incident is observed within 1 minute. The effect occurs in 1–5 minutes and if myasthenia is the cause, ptosis improves after edrophonium injection.
  • Acetylcholine receptor antibody test when positive has 100% specificity.
  • Single-fiber electromyogram (EMG: 100% sensitivity) and muscle biopsy are other more invasive tests that are helpful in identifying the site of pathology in myopathic and myasthenic ptosis.
Q.14. Postsurgical ptosis?
Ans. The incidence of ptosis after cataract surgery has been reported to be as high as 13%.3 Although it can be seen following any intraocular surgery, it is often seen following cataract and vitreoretinal surgery. It can be transient/acute ptosis that resolves after surgery or chronic/persistent ptosis that persists after surgery
Etiopathogenesis: Postsurgical ptosis can be due to the following:
  • Myogenic due to the process of injecting anesthetic into the muscle or myotoxic effects of the anesthesia
  • Aponeurotic: Due to use of a bridle suture or rigid lid speculum
  • Neurogenic: Due to the prolonged effects of anesthetic on the neuromuscular junction, causes transient neurogenic ptosis
  • Mechanical: May be due to edema or hematoma formation in the eyelid
  • Traumatic: Due to blunt or sharp trauma to the levator aponeurosis
Prevention: Prevention of postsurgical ptosis is an essential part of the modern ocular surgery.
  • Topical anesthesia eliminates all problems with local anesthesia including hematoma and edema of the eyelid and myotoxic effects on the levator.
  • Use of ocular massage and compression decreases the amount of eyelid edema and hematoma formation.
  • Limit surgical time and thus eyelid complications secondary to ocular inflammation or compressive effects of prolonged use of a lid speculum.
  • Disuse of bridle sutures or a rigid speculum
  • Superior approach to surgery have a greater risk compared to a temporal approach23
Treatment: After a thorough examination in which the etiology is determined, one must decide whether to intervene. In most cases, postsurgical ptosis resolves with time, and therefore observation is the most prudent form of intervention. This form of ptosis typically improves within six months. Ptosis that does not resolve is typically secondary to aponeurotic dehiscence; this is readily repaired surgically.
Q.15. What is traumatic ptosis?
Ans. Trauma to the levator aponeurosis or the LPS muscle may also cause ptosis through myogenic, aponeurotic, neurogenic, or mechanical defects. Eyelid lacerations exposing preaponeurotic fat indicate that the orbital septum has been transected and suggest the possibility of damage to the levator aponeurosis exploration of the LPS muscle or aponeurosis is indicated in these patients if LPS function is diminished or ptosis is present. Orbital and neurosurgical procedures may also lead to traumatic ptosis. The ophthalmologist normally observes the patient for 6 months before considering surgical intervention.
Q.16. What is myogenic ptosis?
Ans. Congenital myogenic ptosis: This type of ptosis is due to dysgenesis of the levator palpebrae superioris (LPS) muscle. Congenital ptosis caused by maldevelopment of the LPS muscle is characterized by decreased LPS function, eyelid lag, and sometimes, lagophthalmos. The upper eyelid crease is often absent or poorly formed, especially in cases of more severe ptosis. Congenital myogenic ptosis associated with a poor Bell's phenomenon or with vertical strabismus may indicate concomitant maldevelopment of the superior rectus muscle (double elevator palsy, or monocular elevation deficiency).
Acquired myogenic ptosis: It is uncommon and results from localized or diffuse muscular diseases such as muscular dystrophy, chronic progressive external ophthalmoplegia, MG, or oculopharyngeal dystrophy. Surgical correction may be difficult, requiring frontalis sling procedures.
Q.17. What is aponeurotic ptosis?
Ans. It is the most common form of acquired ptosis. It results from stretching or dehiscence of the levator aponeurosis or disinsertion from its normal position. Common causes are involutional attenuation or repetitive traction on the eyelid, which may occur with frequent eye rubbing or prolonged use of rigid contact lenses. It can also occur due to intraocular surgery or eyelid surgery. The characteristic sign is a high or an absent upper eyelid crease secondary to upward displacement or loss of the insertion of LPS fibers into the skin. Thinning of the eyelid superior to the upper tarsal plate can also be there. LPS function in aponeurotic ptosis is usually normal (12–15 mm) and worsens in downgaze.
Q.18. Describe neurogenic ptosis.
Ans. Congenital neurogenic ptosis is caused by innervational defects during embryonic development. It is rare and commonly associated with congenital 3rd cranial nerve palsy, Horner syndrome or Marcus Gunn Jaw-Winking syndrome. It manifests as ptosis together with an inability to elevate, depress, or adduct the globe. The pupils may also be dilated.
  • Congenital Horner syndrome is a manifestation of an interrupted sympathetic nervous chain. It is associated with mild ptosis, miosis, anhidrosis, and decreased pigmentation of the iris on the involved side. Decreased sympathetic tone to the inferior tarsal muscle in the lower lid (the analog of the Müller's muscle), results in elevation of the lower eyelid. This phenomenon is called as lower eyelid reverse ptosis. The combined upper and lower eyelid ptosis decreases the vertical interpalpebral fissure and may confuse with enophthalmos. The pupillary miosis is most apparent in dim illumination.
  • Marcus Gunn Jaw-Winking syndrome (Figs 4 and 5) is the most common form of congenital synkinetic neurogenic ptosis.24
    zoom view
    Fig. 4: Right-sided severe complicated ptosis
  • Some forms of Duane retraction syndrome also cause elevation of a ptotic eyelid with the movement of the globe.
Acquired neurogenic ptosis results from interruption of normally developed innervation and is most often secondary to an acquired III-N palsy, to an acquired Horner syndrome, or MG. Less common causes of acquired neurogenic ptosis include myotonic dystrophy, chronic progressive external ophthalmoplegia, Guillain-Barré syndrome, and oculopharyngeal dystrophy. Botulinum toxin injection in the forehead or orbital region to ameliorate benign essential blepharospasm can also lead to this form of ptosis.
Q.19. What is mechanical ptosis?
Ans. Mechanical ptosis usually refers to the condition in which an eyelid or orbital mass weighs or pulls down the upper eyelid. It can be caused by a congenital abnormality, such as a plexiform neurofibroma or hemangioma, or by an acquired neoplasm such as a large chalazion, skin carcinoma, or orbital mass. Postsurgical or post-traumatic edema can also cause temporary mechanical ptosis.
zoom view
Fig. 5: Right-sided severe complicated ptosis with improvement in ptosis after chewing movement
References
  1. Brad Bowling. Kanski's Clinical Ophthalmology: A systematic approach, 8th edn. Edinburgh: Elsevier;  2015.
  1. Albert DM, Miller JW, Azar DT. Albert & Jakobiec's Principles and Practice of Ophthalmology; 2008.
  1. Oral Y, Ozgur OR, Akcay L, Ozbas M, Dogan OK. Congenital ptosis, and amblyopia. J Pediatr Ophthalmol Strabismus. 2010;47:101–4.
  1. Skuta GL, Cantor LB, Weiss JS. Basic and Clinical Science Course Orbit, Eyelids, and Lacrimal System American Academy of Ophthalmology. Section 7 San Francisco, 2011–2012.25
 
CONTRACTED SOCKET
Varsha Vashney, Aditi Dubey, Amar Pujari
 
INTRODUCTION
The contracted socket is defined as the shrinkage and shortening of all or a part of orbital tissues causing a decrease in depth of fornices and orbital volume ultimately leading to inability to retain prosthesis. it is characterized by extensive loss of conjunctival surface area, deep cicatrix formation, atrophy of the orbital fat, fornix contraction and Volume redistribution leading to post-enucleation syndrome.
In examinations, it can be given as a long or short case.
 
HISTORY
 
Chief Complaints
Patient is usually an adult with history previous enucleation or evisceration surgery for any cause with or without prosthesis with complaints of:
  • Poor cosmetic appearance
  • Repeated extrusion of prosthesis
  • Previously fitting prosthesis not fitting now
  • The patient sometimes may be a child with a history of the underdeveloped socket.
 
History of Present Illness
Note about following:
  • Age at onset: To differentiate between congenital and acquired anophthalmic socket
  • Preceding surgery: Type of surgery performed and its details from the record if available
  • Progression: Whether the implant or prosthesis was fitting earlier, and if the implant size has been changed over time.
 
History of Past Illness
Past history may give a clue about the probable cause, so following points must be noted in past history:
  • Ocular insult like chemical or radiation injury may be present
  • Panophthalmitis requiring evisceration
  • Severe ischemic ocular disease
  • Cicatrizing conjunctival diseases may lead to progressive forniceal shortening
  • Chronic inflammation and infection that might have led to the surgery
  • Mutilating trauma to eye in which evisceration might have done to prevent sympathetic ophthalmia.
 
Surgical History
History of prior surgery; such as enucleation/evisceration, multiple socket reconstruction and the details of the procedure can identify the reason for contracted socket.
 
History of Systemic Illness
History of hypertension, diabetes mellitus, vascular diseases and any previous cerebrovascular accident must be noted carefully. Any neurological event may indicate the intracranial spread of the disease, for which the surgery have been done, such as infection or malignancy.
 
EXAMINATION
 
Systemic Examination
Most of the time the surgery is carried out to remove an intraocular malignancy in an advanced stage, e.g. stage E retinoblastoma. Thus, a detailed systemic examination is carried out to rule out any systemic metastasis, especially the nervous system.
 
Ocular Examination
Ocular examination includes following:
  • Eyelids
    • Eyelid notches and abnormalities need to be looked out for. In longstanding cases there may be stretching and lengthening of the lower lid which would need to be tackled simultaneously.
    • Eyelid closure needs to be looked for too.
    • The lower eyelid should be evaluated for laxity. Lash position and lid margin 26position should be noted, as entropion can indicate socket contracture.
    • The superior sulcus should be checked for deepening and symmetry with the opposite side. The upper eyelid position should be noted for ptosis, and levator function should be evaluated.
  • Assessment of the socket: The following parameters are evaluated:
    • General appearance and symmetry compared to the other normal side should be noted carefully.
    • Area of the socket: The area is assessed particularly the depth of the fornices. The depth of the fornices can be calculated by inserting a blunt lacrimal probe into each of the fornices and noting down the length to which it can be inserted. The inferior fornix is the most important as it has to support the prosthesis. The other fornices also need to be adequate to ensure the prosthesis fitting.
    • The volume of the socket: The volume is assessed by noting the relative depth of the socket compared to the fellow eye. Another practical way of assessing the volume is to inject saline into the socket drop by drop till it overflows. The superior sulcus deformity and presence of ptosis are also indicators of volume loss.
    • Dry or wet socket: Look for any discharge from the socket. There should be no active discharge from the socket. Dry fibrotic conjunctiva indicates a poorly vascularized socket.
    • Movements: The movements of the muscles are looked for. In a case of dermis fat grafting, suturing the muscles to the graft ensures better survival.
    • The tone of the orbicularis and tarsal sulci.
    • Cicatricial bands and degree of contracture within the socket.
    • Associated any bony contracture must be checked for.
    • Look for signs of inflammation, excessive mucus, giant papillary conjunctivitis under the upper eyelid and pyogenic granulomas.
    • Palpation of the socket is done to for the presence or absence of an implant and the position of the implant should be noted.
    • Prosthesis: With the prosthetic in place the patient should be evaluated for fit, size and its appearance with respect to the fellow eye. The movement of the prosthetic should be evaluated compared to the other eye. The prosthetic can then be removed and evaluated for type, size, smoothness and cleann ess.
 
INVESTIGATIONS
  • Microbiological investigations to rule out any infection.
  • Radiological investigation to look for any fracture in orbit or search of buried implant.
 
GRADING
  • Gopal Krishna classification
    The soft tissue sockets were divided into five grades for the sake of convenience in the management of contracted sockets.
    • Grade-0: Socket is lined with the healthy conjunctiva and has deep and well-formed fornices.
    • Grade-I: Socket is characterized by the shallow lower fornix or shelving of the lower fornix. Here the lower fornix is converted into a downwards sloping shelf which pushes the lower lid down and out, preventing retention of an artificial eye. Common causes are physical injuries, endophthalmitis, and retinoblastoma.
    • Grade-II: Socket is characterized by the loss of the upper and lower fornices. The common causes are physical injuries, endophthalmitis, panophthalmitis, and retinoblastomas.
    • Grade-III: Socket is characterized by the loss of the upper, lower, medial and lateral fornices. Common causes are chemical injuries and panophthalmitis.
    • Grade-IV: Socket is characterized by the loss of all the fornices and reduction of a palpebral aperture in horizontal and 27vertical dimensions. Common causes are chemical injuries and panophthalmitis.
    • Grade-V: In some cases, there is the recurrence of contraction of the socket after repeated trial of reconstruction. Common causes are thermal and chemical injuries of the eye.
  • Byron Smith classification
    Socket contraction may also be graded as follows:
    • Mild: Includes grade I and II where only one fornix is involved and there is a shortening of the posterior lamella of the lids.
    • Moderate: Includes grade III where both superior and inferior fornices are involved.
    • Severe: Comprises of cases in which all fornices are involved along with phimosis of palpebral aperture.
    • Malignant contracted socket: It is the most severe variety of contracted socket and associated bony contraction, resulting from severe trauma or multiple surgeries.
  • Morphological classification: Guibor has classified clinically contracted socket into 4 morphological types as shown in Table 1.
 
TREATMENT (Table 2)
The primary aim of management is to create a socket so as to maintain a prosthesis with a good cosmetic appearance. Before commencing a definitive therapy, it is necessary to identify, classify and eliminate any precipitating factors leading to contracture.
General considerations before socket reconstruction:
Table 1   Morphological classification of contracted socket
Type
Examples
Anophthalmic contracted socket
Most common seen after enucleation and evisceration surgery
Ophthalmic contracted socket
Following chemical and irradiation injury
Microphthalmic contracted socket
In association with microphthalmos and microcornea
Hypoplastic contracted socket
Congenital under development of bony socket
  • Informed consent must be obtained
  • The prognosis and aim of surgery must be well explained
  • In cases, oral mucosa grafting is planned the patient should have mouthwashes started at least 2 weeks prior to the surgery.
  • Ensure the that the socket is free of any infection.
  • Mild contracted socket: This can usually be managed by deepening the inferior fornix with fornix formation sutures.
  • Management of moderate-to-severe contracted socket: These cases are usually managed with a graft. Grafts that can be used for socket reconstruction include mucosa, split skin, and dermis—fat grafts. The socket needs to be healthy and vascularized for the grafts to take up. For mucous membrane grafts, mucus can be taken from buccal cavity (lip or cheek), rectum or vagina. The buccal cavity is preferred as it is easy to access.
    Table 2   Mechanism of contracted socket
    Etiology
    Factors
    Etiology related
    • Alkali burns, Radiation therapy leading to severe damage to the socket and fibrosis
    Surgery related
    • Fibrosis from the initial injury
    • Poor surgical techniques: Extensive dissection of the orbital tissue.
    • No implant or undersized implant: In children the absence of the stimulus of either eyeball or implant can lead to bony contraction as well.
    • Excessive sacrifice of the conjunctiva and Tenon's capsule
    • Traumatic dissection within the socket leading to scar tissue
    • Multiple socket operations
    Site related
    • Poor vascular supply
    • Severe ischemic ocular disease in the past
    • Cicatrizing conjunctival diseases
    • Chronic inflammation and infection
    Implant and prosthesis related
    • Undersized implant
    • Implant migration
    • Implant exposure
    • Not wearing a conformer/prosthesis: Confirmer keeps the fornices stretched and prevents fornicial shallowing
    • Ill-fitting prosthesis
    28
  • Management severe contracted socket: These cases usually require both area and volume replacement thus a composite graft is required. The commonly used graft is the dermis fat graft wherein the fat provides the volume and the dermis provides the surface area of the socket. The graft is taken from the hip. Although autogenous dermis fat orbital implantation is an effective means of orbital reconstruction, there is a 30% chance of atrophy of at least half of the graft volume when it is implanted in an avascular socket. Introducing a pedicle flap into the orbit as a vascular bed for an autogenous dermis fat graft may increase the prospect of graft survival, as well as supply additional volume to fill the socket. Temporalis muscle graft is supplied by a superficial temporal artery, a branch of an external carotid artery and it can be used as a pedicle graft.
  • Treatment of moist socket: Partial-thickness mucous membrane grafts are more susceptible to shrinkage and contracture. Full thickness mucous membrane contracts less and may be obtained with minimal postoperative complications at the donor site. However, mucosal contracture and submucosal scar formation increase with the size of the oral mucous membrane harvested and mucous membrane lacks the rigidity needed for grafting the palpebral surface. A full thickness mucous membrane graft is obtained from oral mucosa of cheeks and lips (Most common), hard palate, preputial skin, the skin of labia. The graft should be 40–50% larger than anticipated to allow for subsequent contracture with healing. It is helpful to harvest the graft at the beginning of the procedure so that it can be soaked in antibiotic solution before use.
Amniotic membrane can also be used instead of the mucosa. It has less patient morbidity, faster recovery and better fitting of a prosthesis. No contracture is observed with an amniotic membrane as against mucous membrane. It is cheap and easily available and has no significant complications associated with it.
  • Treatment of dry socket: The socket is lined with a split-thickness skin graft in these cases. The skin graft is placed around an orbital mould with the epithelial surface towards it and perforations are made in the graft. The mould is sutured into the socket. After the 1-month graft is split open in the area of palpebral fissure. The mould is kept for at least 4 months after which a permanent prosthesis is placed.
  • Management of recalcitrant cases: A socket that has undergone multiple unsuccessful operations and has excessive scar tissue is unlikely to benefit from further repair. For such sockets exenteration of the eyelid and residual socket material to create a cavity into which a prosthesis is fitted, can be done. Optical methods to improve the appearance includes spectacle prosthesis or smoked lenses, plus or minus lenses to magnify a micro-ophthalmic socket to minimize buphthalmic socket, prisms to change the apparent horizontal or vertical position of malpositioned prosthesis or socket.
 
VIVA QUESTIONS
Q.1. What are the causes of contracted socket?
Ans. Causes of contracted socket can be congenital or acquired.
  • Congenital: Conditions such as microphthalmos (Fig. 1) or congenital anophthalmos (Fig. 2) usually lead to a contracted socket as the stimulus of the eyeball is essential for healthy growth of the orbit.
  • Acquired: Acquired causes are described here:
    • Enucleation without implant: A poorly done enucleation, particularly without implant can lead to a contracted socket. This is more so in children as in the absence of the stimulus of either eyeball or implant, there is a bony contraction as well. The implant needs to be carefully selected, both in terms of size and material.29
      zoom view
      Fig. 1: Unilateral microphthalmia with contracted socket
    • Delay in use of conformer: In both enucleation and evisceration procedures, conformer should be fitted immediately. This keeps the fornices stretched and prevents fornicial shallowing. The conformer should be on the correct side, adequate size and have multiple holes to allow flushing and drainage of secretions.
    • Trauma: Extensive lacerations of the lids and orbital tissue can lead to tissue loss and fibrosis resulting in socket contraction. Injuries with alkali/acid can also cause fibrosis.
    • Radiotherapy: Post-operative radiotherapy for retinoblastoma can cause fibrosis and a grossly contracted socket. These sockets are usually poorly vascularized and difficult to reconstruct.
  • Infection: Socket/implant infection can lead sloughing of the conjunctiva and shortening of the fornices.
Q.2. What precautions should be taken to prevent contracted socket ?
Ans. Socket contraction should be prevented as far as possible by taking some precautions at the primary surgery.
  • Proper dissection at the time of initial procedure
  • Preserving as much conjunctiva and Tenon's capsule as possible during enucleation
    zoom view
    Fig. 2: Bilateral anophthalmia with contracted socket
  • Secure closure of all layers over the implant without tension or superior displacement of the inferior fornix
  • Avoidance of ill-fitting or roughened prosthesis as it may cause a more rapid contracture, symblepharon formation and total abandonment of prosthesis
  • Elimination of any source of chronic infection that may arise from lid margin, socket, canaliculi, lacrimal sac, chemical or thermal injury
  • Identification of conjunctival cicatrizing diseases like pemphigoid, Stevens-Johnson syndrome
  • Avoidance of oversized prosthesis so as to prevent migration of the implant into the inferior fornix and thereby obliteration of inferior cul-de-sac.
  • Conformer: A conformer should always be placed at the end of the surgery. This is replaced by the prosthetic eye 4-6 weeks later. If the socket has undergone prior irradiation, chemical, or thermal injury, the conformer has to be left for a much longer time.
  • Radiotherapy if required, should be used with fractionation of dose.
Q.3. What are the characteristics of an ideal orbital implant?
Ans. Ideal orbital implant should be:
  • Lightweight
  • Nonantigenic
  • Inert30
  • Biocompatible
  • Affordable
  • Mimic the motility of the normal globe
  • Minimum complications like infection, extrusion and migration.
Q.4. Types of orbital implants
Ans. Presently all implants are broadly classified as:
  • Nonintegrated implants: Are the ones which are nonporous and are not integrated and have no direct muscle attachment, e.g. silicon and acrylic implants.
  • Semi-integrated implants: Allow attachment of muscle in tunnels on the anterior surface for better motility. Examples include Allens, Iowa, etc.
  • Biointegrated implants: These allow fibrovascular ingrowth in the porous channels and result in direct biological integration with orbital contents. Examples include hydroxyapatite, porous polyethylene implants, aluminum oxide, alpha sphere.
  • Biogenic implants: An autograft or allograft of natural tissue with direct biological integration with orbital structures but not prosthesis. For example, dermis fat graft, mucous membrane graft.
Porous implants are presently the material of choice as vascularization leads to the integration of implants. But porous implants are significantly more expensive than acrylic and silicon implants.
Q.5. Post-enucleation socket syndrome/volume deficient socket.
Ans. Typically seen the following enucleation and characterized by following:
  • Enophthalmos
  • An upper eyelid sulcus deformity
  • Ptosis or eyelid retraction
  • Laxity of the lower eyelid
  • A backward tilt of the ocular prosthesis
  • Unhappy with cosmetic appearance.
Bibliography
  1. Krishna G. Contracted sockets-I (Aetiology and types). Indian J Ophthalmol. 1980;28:117–20.
  1. Nesi FA, Lisman RD, Levine MR. Evaluation and current concepts in the management of anophthalmic socket. In: Smith's ophthalmic plastic and reconstructive surgery, 2nd edn. Mosby.  1998. pp. 1079–124.
 
BLOW OUT FRACTURE OF ORBIT
Aditi Dubey, Amar Pujari
 
INTRODUCTION
Orbital injury forms an important aspect of ocular trauma. The blowout fracture is the most common type of orbital fracture. The term pure orbital blowout fracture is used to describe fracture of the orbital floor, the medial wall or both, with an intact bony margin. The term impure orbital blowout fracture is used when such fractures occur in conjunction with a fracture of the orbital rim. The most common site for orbital blowout fracture is the posteromedial aspect of the orbital floor medial to the infraorbital neurovascular bundle where the maxillary bone is very thin (0.25–0.50 mm). As the lamina papyracea is also very thin, the medial orbital wall is also prone to fracture, either in isolation or in association with a fracture of the orbital floor or other facial bones.
In examinations, it can be given as a long case.
 
HISTORY
Chief complaints: It depends upon the duration following trauma, after which the patient presents. When the patient presents immediately following trauma the complaints, include:
  • Eyelid ecchymosis (Fig. 1) /periorbital hematoma: Usually present but signs may be absent as seen in the ‘white-eyed blowout fracture’.31
    zoom view
    Fig. 1: Orbital floor fracture
  • Subcutaneous emphysema: If a blowout fracture communicates with an air-filled sinus it may result in emphysema. Commonly seen in medial orbital wall blowout fractures. It may result in palpable crepitus. Patients should be advised not to blow their nose
  • Proptosis of variable degree can also be seen initially due to orbital edema and hemorrhage
  • Other complaints such as epistaxis, pain, loss of vision can be there depending on the damage to adjacent structures.
  • Diplopia can be there due to muscle entrap- ment
  • Small or inward displacement of an eyeball (e nophthalmos) can be there due to soft tissue prolapse into adjacent sinuses.
  • In cases, where the presentation is delayed, the complaints are usually of a small or shrunken eyeball, limitation of movements or diplopia.
 
History of Present Illness
A detailed history regarding mode of injury should be taken to assess the mechanism and extent of an injury. In cases of delayed presentation, progression must be noted carefully. The decision for surgical intervention largely depends upon the course of the symptoms over time.
Past history: Past history of poor vision in the affected eye, repeated trauma should be noted.
Systemic history: Past history of DM, hypertension, bleeding disorders and neurological problems must be noted. This will be helpful while planning surgery, especially under general anesthesia.
 
EXAMINATION
Systemic examination: General condition of a patient and another nonocular injury should be checked. Life-threatening injuries must be taken care of first before proceeding for ocular examination.
Ocular examination: Following points must be noted in a case of orbital fracture.
Visual acuity: Visual acuity at presentation has medico-legal importance in ocular trauma cases. Uncorrected as well as best corrected visual acuity must be noted in all cases of trauma.
 
Orbit
  • Palpate orbital rim to look for deformity and crepitus. Subcutaneous emphysema with crepitus seen in fractures communicating with air-filled sinuses. The malar eminences should be palpated and any depression noted. The patient should be asked to open and close his mouth to rule out pain or trismus that may be associated with a zygomatic complex fracture.
  • Paresthesia or sensory loss over ipsilateral lower lid, cheek and upper lip pathognomonic. Neurosensory loss occurs in the area supplied by the infraorbital nerve. This occurs because the fracture extends along the infraorbital groove or canal injuring the infraorbital nerve. These sensory defects tend to resolve spontaneously with time but may get aggravated by surgery in the area.
  • Ocular motility: Full orthoptic assessment should be performed in nine positions of gaze. Limitation of ocular motility can occur due to following causes:
    • Entrapment of orbital contents such as connective tissue, septa, extraocular muscle (inferior rectus most commonly) within the fracture.
    • Hematoma/edema in the orbital fat adjacent to the fracture.
    • Hematoma or contusion of the extraocular muscle itself.
    • Palsy of an extraocular muscle due to neuronal damage.
    • Volkmann's ischemic contracture of an entrapped extraocular muscle.32
    • Inferior rectus muscle leads to motility restriction especially in upgaze (vertical diplopia.
Eyelid: Pseudoptosis occurs due to loss of support. Look for other signs of trauma such as laceration or scar.
Conjunctiva: Subconjunctival hemorrhage and conjunctival chamois may be there.
Cornea: Trauma can lead to corneal abrasion, corneal laceration.
Iris and anterior chamber: Trauma may be associated with iritis, miosis or mydriasis, hyphema.
Pupil: Presence of relative afferent pupillary defect (RAPD) indicates optic nerve injury that needs urgent intervention in the form of pulse steroid therapy.
Lens: Trauma may be associated with subluxation, dislocation of lens.
Fundus: Look for effects of trauma such as dialysis, tear, retinal detachment or vitreous hemorrhage.
 
Special Tests
  • Hertel exophthalmometer: Exophthalmometry is done to document enophthalmos.
  • Force duction test (FDT): FDT is useful in determining whether dysmotility is restrictive or paralytic. In a blow out fracture with inferior rectus, entrapment FDT is ‘positive’ indicating a mechanical cause.
  • Force generation test (FGT): In testing force generation, the muscle insertion is grasped and the patient is asked to look into the muscle's field of action. A paretic muscle will feel weak when compared with the fellow eye.
  • Diplopia charting: With red green glass, diplopia charting with streak light shows diplopia worsening in upgaze.
  • Hess screen or Lee screen tests were done document the muscle involved.
 
INVESTIGATIONs
  • Plain X-rays: Easily available and cost-effective imaging modality. Water's view is the most useful projection for detecting an orbital floor fracture. X-ray shows bone discontinuity in the orbital floor with herniation of soft tissue in maxillary antrum seen as ‘hanging drop’ sign.
  • Computerized tomography (CT) scanning: CT gives the detailed visualization of bony and soft tissue injury where entrapment of muscle can be appreciated (Fig. 2). Coronal sections are particularly useful and can show antral soft tissue densities, such as prolapsed orbital fat, extraocular muscle or hematoma.
  • Magnetic resonance imaging (MRI): Can be utilized when there is the need for greater soft tissue evaluation. MRI is insufficient in assessing the bony structures and therefore needs to be combined with CT.
 
MANAGEMENT
Treatment includes both medical and surgical management:
  • Medical management/Observation: It consists of oral antibiotics, analgesics and a short course of oral prednisone. Oral steroid benefits the patient by reducing the edema of the orbit and muscle. This also may allow for a more thorough assessment of the relative contribution to enophthalmos or entrapment from the fracture versus that from edema. Medical management is indicated in following conditions:
    • No significant enophthalmos (<2 mm)
    • Lack of marked hypo-ophthalmos
      zoom view
      Fig. 2: Fracture of orbital floor on CT Scan
    • 33Absence of an entrapped muscle or tissue
    • Fracture less than 50% of the floor
    • No diplopia
    • The patient must be advised to avoid nose blowing, to avoid creating or worsening orbital emphysema. Nasal decongestants can be used if not contraindicated. Ice packs may be applied for initial 48 hours to reduce the pain and tissue edema.
  • Surgical treatment
    Surgical intervention is indicated in following cases:
    • Immediate intervention:
      • Diplopia present with CT evidence of an entrapped muscle or periorbital tissue and associated with a non-resolving oculocardiac reflex: bradycardia, heart block, nausea, vomiting or syncope
      • “White-eyed blow-out fracture.” Young patients (<18 years), history of periocular trauma, little ecchymosis or edema (white eye), marked extraocular motility vertical restriction, and CT examination revealing an orbital floor fracture with entrapped muscle or perimuscular soft tissue
      • Early enophthalmos/hypoglobus causing facial asymmetry.
  • Within 2 weeks: Patients with diplopia are usually observed for 2 weeks. If the diplopia resolves with a small fracture evident on CT, no surgical intervention is required. It is advisable to wait for 2–3 weeks for resolution of orbital edema/hematoma. However, surgery is indicated in the following scenario:
    • Symptomatic diplopia with positive forced ductions, evidence of an entrapped muscle or perimuscular soft tissue on CT examination, and minimal clinical improvement over time.
    • Large floor fracture causing latent enophthalmos
    • Significant hypo-ophthalmos
    • Progressive infraorbital hypoesthesia
Surgical principle: The basic steps of surgery includes assessing the orbital floor, releasing the soft tissue and muscle entrapment and strengthening the floor with use of implants.
The orbital floor can be approached through following ways:
Subciliary approach: Incision given 2–3 mm below the lash line. It has the advantage of better scar camouflage. The disadvantage is postoperative ectropion and lower lid retraction.
Subtarsal approach: Incision below tarsal plate over orbital rim. The advantage is it gives direct access to the floor with good exposure. The disadvantage is it leads cosmetically unacceptable scar.
Transconjunctival approach: Incision given in lower fornix 3 mm below the tarsal plate and can be combined with a lateral canthotomy for better exposure. The advantage is it gives no visible scar.
Transantral approach: Orbital floor reached via the maxillary sinus using Caldwell-Luc incision. It is a difficult technique and it is not a favored by an ophthalmologist.
Endoscopic approach: Trans-maxillary and trans-nasal endoscopies have been described which eliminate the need for eyelid incisions and gives improved visualization of fractures. However, it is difficult and often clumsy.
The orbital floor is reinforced with either autogenous or synthetic implant (Table 1). The surgeon should size the implant to cover the defect adequately and to prevent displacement or extrusion later. While cutting the implant, it should be tapered posteriorly to fit the orbital floor configuration.
 
VIVA QUESTIONS
Q.1. What is a pure and impure blowout fracture?
Ans. A pure orbital blowout fracture is used to describe fracture of the orbital floor, the medial wall or both, with an intact bony orbital margin. Impure orbital blowout fracture is used when such fractures occur in conjunction with a fracture of the orbital rim.
Q.2. Most common site for the blowout fracture of orbit?
Ans. The posteromedial aspect of the orbital floor medial to the infraorbital neurovascular bundle where the maxillary bone is very thin (0.25–0.50 mm) is the most common site.34
Table 1   Examples of implant materials used in orbital floor repair
Implant material
Advantage
Disadvantage
Membranous bone
Autogenous
  • Morbidity at donor site
  • Extended operation time
  • Resorption unpredictable
Cartilage
Autogenous
  • Morbidity at donor site
  • Extended operation time
  • Resorption unpredictable
Titanium mesh
Biocompatible stable
  • Foreign material that remains in the body
  • Combination with bone recommended
Porous polyethylene
(Medpore) sheets
Easy to shape and handle,
Biocompatible stable
Foreign material that remains in the body
Silicon sheet
Easy-to-handle and cheap
Extrusion rates higher
Silastic sheet (Teflon)
Easy-to-shape and handle
Foreign body reaction and extrusion common
Q.3. What is the etiopathogenesis of blowout fracture?
Ans. There are two theories to explain the possible mechanism of orbital fracture:
  • Retropulsion theory/hydraulic theory: States that the backward displacement of the globe caused by a blunt non-penetrating object raises the intraorbital pressure sufficiently to fracture the posteromedial orbital floor and/or the lamina papyracea of the ethmoid.
  • •Buckling theory/transmission theory: A transient deformation of the orbital rim transmits the force of injury directly to the orbital wall. During the course of injury, the force that is transmitted to bony walls of the orbit may also cause concussion ocular trauma leading to angle recession, hyphema, vitreous hemorrhage, commotio retinae, etc.
Q.4. Expanded orbit syndrome
Ans. Multiple fractures in and around the orbit may lead roomy orbit with extensive prolapse of orbital tissues. This expansion can be seen in orbital fracture along with midfacial fracture as in tripod or Le Fort type III. Clinically patient has gross enophthalmos, inferior displacement of the globe (hypoglobus), deep superior sulcus, eyelid asymmetry and diplopia.
Q.5. What are the features of medial wall fracture?
Ans. Blowout fracture of a medial wall is much less common than a floor and usually seen along with nasoethmoid fractures, rather than as an isolated entity. Horizontal diplopia is usually the primary complaint when medial orbital tissues are involved. However, a vertical or oblique component can also be found in such cases.
Q.6. What is ‘white-eyed’ blowout fracture?
Ans. The bones of a child's orbit are more elastic than adults. Thus, injury in children causes more anteroposterior buckling creating a fracture with overlapping segments. This leads to ‘trapdoor-type’ fracture where prolapsed orbital tissue is caught in the fracture site leading to severe motility restriction and diplopia in absence of marked congestion or ecchymosis. The condition is also called the ‘white-eyed’ blowout fracture. It is seen in orbital blowout fracture in children.35
Q.7. Boundaries of the orbital floor and contents
Ans.
  • The adult orbital floor is formed by the maxillary, zygomatic bones anteriorly and palatine bones posteriorly.
  • Orbital floor measures about 35–40 mm anteroposteriorly and it is the shortest of all the walls. It forms the roof of the maxillary sinus.
  • The floor of the orbit contains infra orbital groove that forms infraorbital foramen. Infraorbital nerve, a branch of the maxillary division of trigeminal nerve passes through the groove, providing sensory innervations to the ipsilateral orbital floor, mid face, and posterior upper gingival area. The infraorbital artery, a branch of the maxillary artery, and the infraorbital vein also are found within the infraorbital groove.
Q.8. What are common complications of floor repair surgery?
Ans.
  • Intraoperative bleeding
  • Residual or new-onset diplopia
  • Extraocular muscle dysfunction
  • Postoperative neuralgia
  • Residual enophthalmos
  • Implant extrusion
  • Possible loss of vision.
Q.9. How to treat persistent diplopia?
Ans. Some patients may have persistent diplopia even after adequate surgical repair of floor fracture. Diplopias in primary gaze and in downgaze (functional gaze) are more troublesome. Such cases will require muscle surgery. To correct diplopia in down gaze ‘Reverse Knapp procedure’ performed placing medial and lateral recti behind inferior rectus muscle. Fresnel prisms can be employed in selective cases.
Q.10. How to treat postoperatively cosmetically unacceptable enophthalmos?
Ans. A repeat surgery with adequate size orbital implant, if the downward sinking of an eye along with enophthalmos is unacceptable to patient, may have to be done. The pseudo ptosis can be corrected with mullerectomy, which will increase palpebral height.
BIBLIOGRAPHY
  1. Burnstine MA. Clinical recommendations for repair of isolated orbital floor fractures: An evidence-based analysis. Ophthalmology. 2002;109(7):1207–10.
  1. Principles and Practice of Ophthalmology: Albert and Jakobeic, Vol. 6, 2nd edition, Pgs. 5303–9.
  1. Smith B, Regan WF. Blowout fracture of the orbit (mechanism and correction of internal orbital fracture). Am J Ophthalmol. 1957;44:733–9.
  1. Yadav P, Pushker N, Bajaj M, Chandra M, Shrey D, Lohiya P. Orbital blow out fracture. DOS Times. 2008;8(14):2.
 
THYROID-ASSOCIATED OPHTHALMOPATHY
Aditi Dubey, Prafulla Kumar Maharana
 
INTRODUCTION
Thyroid associated ophthalmopathy (TAO) is the most common cause of proptosis in adults. TAO is a self-limiting autoimmune disease associated mainly with hyperthyroidism, but also with hypothyroid and euthyroid states. Although it can affect any age, most commonly presents during the fourth and fifth decades of life. It is one of the most important long case given in practical examinations. In TED, it is important to record a proper history and elicit the signs associated with it.
 
HISTORY
 
Demography
It can affect any age group. Patients with TAO are more likely to be female by a 2:1 ratio, following the usual predominance of autoimmunity in women. 36However, male Graves’ disease are at the same, if not higher, risk of TAO development, which is usually of a more severe form and occurs at a more advanced age than in their female counterparts. Asians are having a lower likelihood of developing the disease than Europeans.1,2
 
Chief Complaints
A case of TAO can present with following complain:
  • Excessive lacrimation, a gritty sensation, discomfort, and photophobia are often present in early course of the disease.
  • Bilateral upper lid retraction is most common presenting feature of the thyroid eye disease.
  • Bilateral proptosis can also be the presenting feature.
  • At times it may be detected accidentally or Referred by the endocrinologist.
  • Decrease in visual acuity can be a presenting feature when it is associated with Optic nerve compression, exposure keratopathy or induced astigmatism due to globe compression.
  • Patients may also complain of general symptoms like weight loss, sweating, heat intolerance, weakness, fatigue and palpitation along with ophthalmic complains.
Approximately 5 to 10% of Graves’ orbitopathy patients are euthyroid at presentation and some of them may not have a history of thyroid dysfunction. Around 40% of patients with TAO, the signs of the eye disease occur simultaneously with the first symptoms of hyperthyroidism.
 
History of Present Illness
The history of present illness must include all the points described in the chapter of proptosis.
  • The patient of TED present with the bilateral upper lid retraction and or exophthalmos. A careful history can reveal history of irritation, foreign body sensation, watering and recurrent lid edema in past that is often ignored by the patient. As the disease progresses the full blown picture of TED develops.
  • Cigarette smoking has been considered the strongest risk factor for developing TED. Hence, a detailed history of smoking must be asked.
  • Females are having more predominance in thyroid eye disease (2:1). However, males presents with more severe disease and usually at a later age.
  • Patient may be in hyperthyroid or hypothyroid state, but 5 to 10% cases are euthyroid at the time of presentation.
 
Past History
  • History of diabetes mellitus (DM), hypertension, asthma, thyroid abnormality should be noted.
  • In a case of diagnosed TAO, past history of steroid, radiotherapy, orbital decompression or any thyroidectomy must be recorded.
 
Personal History
Personal history of alcohol intake, smoking, tobacco chewing or any other if present should be noted, because smoking is considered as important risk factor for the TED.
 
EXAMINATION
Examination of a case of TAO is similar to that of a case of proptosis. Salient features of TAO are described here.
 
Systemic Examination
General examination may show the signs of hyperthyroidism such as tachycardia, fine hand tremor, warm and sweaty skin, pretibial myxedema, finger clubbing, alopecia and vitiligo. A detailed examination of cardiovascular system, respiratory system, gastrointestinal system and Central nervous system has to be done as these systems gets affected by Graves’ disease.
 
Ophthalmic Examination
The ocular examination is similar to a case of proptosis. Important points to note are described below:
  • Proptosis is usually axial.
  • Globes are mostly aligned.
  • The most characteristic signs are eyelid erythema and swelling, caruncular and conjunctival injection and edema (Fig. 1).37
    zoom view
    Fig. 1: Severe bilateral proptosis due to thyroid eye disease
  • Some patients may have restriction of the eye movements leading to squint. The most commonly affected muscle is inferior rectus followed by medial, superior, levator and lateral rectus (Fig. 2). The muscles affected results in ocular misalignment, diplopia. Strabismus is also common, and it often presents as hypotropia or esotropia.
  • Eyelid: Various lid signs which can be seen in thyroid ophthalmopathy (Table 1). All signs may not be present in a single patient. Lid retraction, also known as Dalrymple's sign, occurs most commonly as lid sign, in about 37–92% of patients (Fig. 3).
  • Eyelashes are usually normal.
  • Conjunctiva may show mild congestion due the dry eye caused by the excessive evaporation of the tears due to the lid retraction.
  • Inflammation along recti muscle (tendonitis) can be an early sign of the disease.
    • Cornea may have exposure keratopathy. Causes of exposure keratopathy include:
      • Inadequate eyelid closure leading to excessive moisture loss as a consequence of proptosis and eyelid dysfunction.
      • Diminished tear production resulting from lacrimal gland infiltration.
      • Lagophthalmos from proptosis.
      • Loss of Bells phenomenon from inferior rectus infiltration.
        zoom view
        Fig. 2: Thyroid eye disease with disproportionate proptosis and inferior dystopia in left eye
        zoom view
        Fig. 3: Thyroid eye disease with lid retraction
  • Pupillary reaction may or may not be normal. Presence of RAPD or APD suggests optic nerve compression.
  • Variable intra ocular pressure (IOP) can be there in different gazes due to restrictive myopathy. An IOP rise more than 8 mm Hg on upgaze is significant and warrant treatment. Glaucoma can also result from decreased episcleral venous outflow.
  • Lens do not show any specific changes.
  • Fundus examination may show signs of globe compression. Compressive optic neuropathy occurs in <5% of patients with thyroid ophthalmopathy resulting in slowly progressive visual loss. It occurs due to compression from the oversized recti and orbital fat causing compartment syndrome at the apex of orbit.38
    Table 1   Lid signs in proptosis
    Sign
    Description
    Abadie's sign
    Elevator muscle of upper eyelid is spastic
    Ballett's sign
    Paralysis of one or more extraocular muscle (EOM)
    Beck's sign
    Abnormal intense pulsation of retina's arteries
    Boston's sign
    Jerky movements of upper lid on lower gaze
    Cowen's sign
    Extensive hippus of consensual pupillary reflex
    Dalrymple's sign
    Upper eyelid retraction
    Enroth's sign
    Edema especially of the upper eyelid
    Gifford's sign
    Difficulty in eversion of upper lid
    Goldzieher's sign
    Deep injection of conjunctiva, especially temporal
    Griffith's sign
    Lower lid lag on upward gaze
    Hertoghe's sign
    Loss of eyebrows laterally
    Jellinek's sign
    Superior eyelid folds is hyperpigmented
    Joffroy's sign
    Absent creases in the forehead on upward gaze
    Jendrassik's sign
    Abduction and rotation of eyeball is limited also
    Knies’ sign
    Uneven pupillary dilatation in dim light
    Kocher's sign
    Spasmodic retraction of upper lid on fixation
    Loewi's sign
    Quick mydriasis after instillation of 1:1000 adrenaline
    Mann's sign
    Eyes seem to be situated at different levels because of tanned skin
    Means’ sign
    Increased scleral show on upgaze (globe lag)
    Moebius's sign
    Lack of convergence
    Payne/Trousseau sign
    Dislocation of globe
    Pochin's sign
    Reduced amplitude of blinking
    Rieseman's sign
    Bruit over the eyelid
    Movement's cap phenomenon
    Eyeball movements are performed difficultly, abruptly and incompletely
    Rosenbach's sign
    Eyelids are animated by thin tremors when closed
    Saiton's sign
    Frontalis contraction after cessation of levator activity
    Snellen-Rieseman's sign
    When placing the stethoscope's capsule over closed eyelids’ a systolic murmur could be heard
    Stellwag's sign
    Incomplete and infrequent blinking
    Suker's sign
    Inability to maintain fixation on extreme lateral gaze
    Tellas's sign
    Inferior eyelid might be hyperpigmented
    Topolanski's sign
    Around insertion areas of the four rectus muscles of the eyeball a vascular band network is noticed and this network joins the four insertion points.
    von Graefe's sign
    Upper lid lag on downgaze
    Wilder's sign
    Jerking of the eye on movement from abduction to adduction
    39It is characterized by decrease in vision, color vision, contrast sensitivity and relative afferent papillary defect. Visual loss may progress undetected due to insidious onset and subtlety of neuropathy. Risk factors for optic neuropathy include:
    • Older age
    • Smoking
    • Male gender
    • Significant strabismus with mild proptosis.
  • Visual field: Central and inferior arcuate scotoma and generalized constriction may be seen in advanced cases ophthalmopathy.
 
DIFFERENTIAL DIAGNOSIS
The diagnosis often straight forward. The characteristic signs and systemic signs are difficult to miss. However, in early course of the disease following diseases may mimic TAO
  • Orbital myositis
  • Nonspecific orbital inflammatory disease (NSOID)
  • Myasthenia gravis
  • Chronic progressive external ophthalmoplegia
  • Carotid-cavernous fistula
  • Specific inflammatory orbitopathy
  • Orbital tumors
  • Lid retraction, characteristic of TAO can also be seen in number of other diseases as given here.
 
Upper Lid Retraction
  • Congenital
  • Neurologic disorders: Midbrain disease, Hydrocephalus
  • Post-surgery: Ptosis surgery, lid reconstruction
  • Marcus Gunn phenomenon, Faulty regeneration of cranial nerve III
  • Parinaud's syndrome
  • Sympathomimetic drugs
  • Cirrhosis.
 
Lower Lid Retraction
  • Idiopathic senile flaccidity of lower lid
  • Post-traumatic impairment
  • Congenital abnormality
  • Postsurgical lesion—Recession of the inferior rectus muscle, Repair of a blowout fracture
  • Facial nerve lesion.
 
INVESTIGATIONs
Following investigations are carried out in a case of TAO:
  • CBC, Thyroid function test [Tri-iodothyronine (T3), Free thyroxin (T4), Serum thyroid-stimulating hormone (TSH), Thyrotrophic receptor antibodies (TRAB), ESR].
  • Ultrasonography (USG): On cross-section, there is an increase in thickness of the extraocular muscles. USG of the globe and the orbit can help in visualization of the tendinous intersections. This also helps to differentiate between active and inactive disease. By comparing the muscle thickness, ultrasound may help in confirming the diagnosis in unilateral cases. It also helps in differentiating associated diseases presenting with similar clinical features.
  • Computed tomography (CT) scan: Typical radiological features seen on CT are muscle belly enlargement that is classically described as ‘tendon sparing’, an increase in orbital fat volume, and crowding of the optic nerve at the orbital apex in severe cases. It helps in assessing the relationship between the optic nerve and muscles at the apex that helps in planning for the surgical intervention, if needed. CT is more sensitive than MRI in identifying enlarged extraocular muscles. As a standard, 2 mm cuts should be requested for, along with coronal and axial slices. Orbital fat is imaged in CT as a black, low-density area that contrasts with the higher-density image of extraocular muscles and the optic nerve. CT scans allow for better delineation of the bony orbit and therefore are invaluable in planning orbital decompression.
  • Magnetic resonance imaging: Demonstrates fusiform rectus enlargement and orbital fat expansion. It assesses water content in the muscles that correlates with the active inflammation. In the active phase, the extraocular muscles appear isointense in T1-weighted images and hyperintense in T2 weighted images; where as in the chronic phase, they appear hypointense on T2-images.40
  • Visual field testing is important for detecting early damage to the optic nerve due to apical crowding around the optic nerve. The changes on visual fields are reversible if the crowding is relieved early, either surgically or medically. Usually, the patterns of visual field loss vary, the most common being central, paracentral and/or inferior.
 
CLASSIFICATION
Different classification system have been proposed, however there is no consensus on the best way to classify TAO:
  • NO-SPECS classification
    • Proposed by Werner et al. and adopted by the American Thyroid Association (Table 2)
    • Based upon clinical presentation
    • Limitation: Relies on subjective evaluation, does not take into account the severity of manifestations, Patient may fall into more than 1 particular class, may not progress in an orderly fashion from class 1 to class 6 and is relatively insensitive to subtle changes, hence less preferred.
  • RELIEF classification of soft tissue signs and symptoms
    R – Resistance to retropulsion
    E – Edema of conjunctiva and caruncle
    L – Lacrimal gland enlargement
    I – Injection over the horizontal rectus muscle insertions
    E – Edema of the eyelids
    F – Fullness of the eyelids
 
Staging/Scoring
  • Clinical activity score (CAS):
    • It is one of the widely utilized grading system described by Mourits and colleagues
    • It attempts to identify patients with active disease who are likely to respond to medical therapy.
    • The CAS is generated by the addition of 1 point for the presence of each the following features: Chemosis, eyelid swelling, eyelid erythema, conjunctival erythema, caruncular swelling, pain in primary gaze, and pain with ocular movement. In addition, if the patient has been examined within the 3 months prior, additional points may be given for decreased visual acuity, worsened diplopia, and increased proptosis compared with that visit.
      Table 2   NO-SPECS classification of thyroid associated ophthalmopathy
      Class
      Grade
      Clinical features
      0
      N – No signs symptoms
      1
      O – Only signs
      2
      O
      A
      B
      C
      S – Soft-tissue involvement
      Absent
      Minimal
      Moderate
      Marked
      3
      O
      A
      B
      C
      P – Proptosis
      <23 mm
      23–24 mm
      25–27 mm
      ≥28 mm
      4
      O
      A
      B
      C
      E – Extraocular muscle involvement
      Absent
      Limitation of motion in extremes of gaze
      Evident restriction of movement
      Fixed eyeball
      5
      O
      A
      B
      C
      C – Corneal involvement
      Absent
      Stippling of cornea
      Ulceration
      Clouding
      6
      O
      A
      B
      C
      S – Sight loss
      Absent
      20/20 – 20/60
      20/70 – 20/200
      <20/200
    • TAO is considered active in patients with a CAS of 3 or more out of 7 (if no previous assessment is available), or 4 out of 10 on the complete scale.
    • This scale has a specificity of 86%, sensitivity of 55%, positive predictive value of 80%, and negative predicative value of 64% for predicting the activity of the disease.41
    • Limitation: It is subjective (depends on both patient and practitioner) and it fails to account for active improvement or worsening of the disease.
  • European Group on Graves’ orbitopathy (EUGOGO): It is one of the commonly used scoring systems. It recommends the following classification of patients with thyroid ophthalmopathy.3
    Mild GO: They usually present with one or more of the following signs:
    • Minor lid retraction (< 2 mm)
    • Mild soft tissue involvement
    • Exophthalmos < 3 mm (above the normal range for the race and gender)
    • Transient or no diplopia
    • Corneal exposure responsive to lubricants
    Moderate-to-severe GO: These patients usually have any one or more of the following:
    • Lid retraction >2 mm
    • Moderate or severe soft tissue involvement
    • Exophthalmos >3 mm above normal for race and gender
    • Inconstant or constant diplopia
    Sight-threatening GO:
    • Patients with dysthyroid optic neuropathy and/ or corneal breakdown.
    • Other infrequent conditions are ocular globe subluxation, severe forms of frozen eye, choroidal folds, and postural visual darkening.
    • This category warrants immediate intervention.
    • As a rule of thumb, it is considered that all patients who do not have a mild or a sight-threatening ophthalmopathy present a moderate-to-severe disease.
  • VISA scoring:
    • Developed by Dolman and Rootman and adopted with modifications by the International Thyroid Eye Disease Society (ITEDS).
    • It is based on symptoms (subjective) and signs (objective) inputs.
    • Four severity parameters are analyzed: V (vision); I (inflammation/congestion); S (strabismus/motility restriction); and A (appearance/exposure).
    • Each feature is considered and graded independently
      • Vision: 1 point
      • Inflammation/congestion: 10 points
      • Strabismus: 6 points (diplopia: 3 points plus restriction: 3 points)
      • Appearance/exposure: 3 points.
      • A global severity grade, with maximum score is 20 points, is the sum of each of the involved systems graded independently:
  • Vision (V): Evaluates the visual problems, especially due to associated dysthyroid optic neuropathy. It is assessed through visual acuity, pupillary reflexes, color vision, visual fields, optic nerve examination, and visual evoked potentials.
  • Soft tissue inflammation/congestion (I) evaluation is graded according to the worst score for the eye or the eyelid with the Inflammatory Index as shown in Table 3. Patients with moderate inflammatory index (less than 4 of 10) are managed conservatively. Patients with high scores (above 5 of 10) or with evidence of progression (as documented on subsequent visits) in the inflammation are offered a more aggressive therapy.
    Table 3   VISA inflammatory index
    Sign or symptom
    Score
    Caruncular edema
    0: Absent
    1: Present
    Chemosis
    0: Absent
    1: Conjunctiva lies behind the gray line of the lid
    2: Conjunctiva extends anterior to the gray line of the lid
    Conjunctival redness
    0: Absent
    1: Present
    Lid redness
    0: Absent
    1: Present
    Lid edema
    0: Absent
    1: Present but without redundant tissues
    2: Present and causing bulging in the palpebral skin, including lower lid festoon
    Retrobulbar ache
    • At rest
    • With Gaze
    0: Absent; 1: Present
    0: Absent; 1: Present
    Diurnal variation
    0: Absent; 1: Present
  • 42Strabismus/motility restriction (S) is documented by three aspects:
    1. Diplopia that is graded from 0 to 3 (0 = no diplopia, 1 = diplopia with horizontal or vertical gaze, 2 = intermittent diplopia in straight gaze, and 3 = constant diplopia in straight gaze).
    2. Ocular ductions are measured to the nearest 5° in four directions using the corneal light reflex technique. Any change of ≥ 12° in any direction can be considered progression.
    3. Ocular restriction can be graded from 0 to 3 based on the range of ductions (0 = duction > 45°, 1 = 30–45°, 2 = 15–30°, and 3 <15°) quantified by prism cover testing.
  • Appearance/exposure (A)
    • Symptoms include appearance concerns (such as bulging eyes, eyelid retraction, and fat pockets) and those derived from ocular exposure (such as gritting sensation, photophobia, dryness, and secondary tearing).
    • Signs include measurements of eyelid retraction (millimeters from the pupillary light reflex to the lid margin); scleral show (millimeters from the limbus to the lid margin); levator palpebrae superioris function; lagophthalmos (incomplete eyelid closure); and proptosis with the Hertel exophthalmometer. Signs of corneal exposure are best assessed with the slit-lamp microscope and may include punctate epithelial erosions, ulcerations, and, in severe cases, corneal thinning and risk of perforation.
The VISA and CAS were designed to determine the clinical activity. In comparison, the NO SPECS and EUGOGO classification assess the clinical severity. Both VISA (particularly in US) and EUGOGO (European countries) are currently used for deciding upon treatment and also monitoring response to treatment.35
 
MANAGEMENT
 
Treatment of Thyroid Gland Dysfunction
It is the most important aspect of treatment of thyroid ophthalmopathy. Frequent monitoring of thyroid status (every 4–6 weeks) is imperative in the initial phases of treatment when changes in thyroid status are expected.
 
Treatment of Ophthalmopathy
Treatment should follow the sequence of (V-I-S-A), i.e. 1st take care of visual disturbance then ISA (of VISA scoring).
 
Treatment Options
  • Supportive measures:
    • Artificial tears: Lubricant eye drops during the day and Lubricant ointments at night-time
    • Sunglasses: To avoid photophobia
    • Patients with symptomatic diplopia- Fresnel prisms or occlusion therapy
    • Botulinum toxin injection may be considered for upper lid retraction
    • Topical adrenergic blocking agents such as 5% guanethidine sulfate drops transiently improve mild eyelid retraction but not of much use
    • Cool compresses
    • Head elevation: To reduce periorbital edema.
  • Medical management:
    Corticosteroids: Systemic steroids are indicated in patients with severe inflammation or compressive optic neuropathy. Intravenous glucocorticoids is required for patients with advanced thyroid-associated orbitopathy. Intravenous glucocorticoids seem to be associated with higher success rate and better tolerability as compared to oral glucocorticoids.
    Steroid-sparing immunosuppressive drugs: Cyclosporine and Methotrexate, Intravenous administration of immunoglobulin, Tumor necrosis factor-α blockers and anti-CD20 monoclonal antibodies (rituximab) have been found useful. However, these are inferior to steroids as monotherapy and considered only when steroid is contraindicated.
  • Radiation therapy:
    Acts by a nonspecific anti-inflammatory effect. RT is effective in patients who have active eye disease with recent progression and ineffective in inactive stages of the disease. 43The Lymphocytes infiltrating the orbit have high radio sensitivity. Usually a dose of 20 Gy is given per eye fractionated over a 2-week period. However, RT can be associated with transient exacerbations of inflammation, hence simultaneous glucocorticoids must be started. Although, the evidence regarding the efficacy of radiation therapy in the management of TAO is limited, it is still one of the widely used treatment modality.
  • Surgical management
    Orbital decompression: It is indicated in cases with compressive optic neuropathy not improving with medical treatment. It enlarges the existing space of the orbit by partial removal of bony walls and periosteum. The most commonly done decompression involves the posteromedial wall followed by floor and lateral wall.
 
Treatment
It depends upon the stage and severity:
  • Mild TED: Only supportive therapy is required. Progression from mild-to-moderate-to-severe TED occurs in about 15%. The side effects of immunosuppressive treatment or radiation do not weigh against the expected beneficial effects.
  • Moderate-to-severe TED: Moderate-to-severe TED is defined as: no threat to vision but sufficient impact on daily life to justify the risks of immunosuppression. Corticosteroids are the treatment of choice with a response rates up to 80%. Intravenous prednisolone treatment is recommended because it has better results compared with high-dose oral therapy and it is associated with less side effects such as diabetes or weight gain. Prior to starting high dose steroid possible contraindications for high-dose prednisone treatment, such as gastrointestinal ulcer disease, severe osteoporosis, latent tuberculosis or hepatitis B or C positivity, uncontrolled diabetes/hypertension must be ruled out. The cumulative dose of prednisolone should not exceed 8 g in one course of therapy. However, the exact dose of prednisolone that yields satisfactory therapeutic effect without adverse events is not exactly known.
  • Very severe TED: Very sever TED should be treated with 1 g methylprednisolone IV daily for three consecutive days, repeated after one week, followed by an oral tapering dose. When there is clinical deterioration, urgent orbital decompression should be considered. Indications for surgical decompression includes:
    • Patients with active disease who have refractory or progressing corneal ulcer.
    • A stretched optic nerve
    • Prevention of further corneal damage
    • Cosmetic in acceptability.
In-orbital decompression, part of the bony walls is removed to provide more room for the extraocular muscles and orbital fat. Associated diplopia usually requires surgery of the extraocular muscles. But after the orbital decompression surgery, diplopia surgery should be postponed till effect of the previous if established. Eyelid surgery such as lengthening (in case of upper eye lid retraction) may be a final step in the rehabilitation of the patient with TED.
 
VIVA QUESTIONS
Q.1. What are the risk factors for TAO?
Ans. Genetics: The TED is considered be an autoimmune disease because of its clinical association with Graves disease, an associated condition known to be caused by anti-thyrotropin receptor antibodies (TRAb).
Tobacco smoking: Smoking is the risk factor most consistently linked to either development or deterioration of TAO. Overall, more than 40% of smokers either developed or worsened TAO, which was almost double the rate of non smokers. Cigarette smoke extract increase production by orbital fibroblasts of glycosaminoglycans, hydrophilic macromolecules that accumulate in TAO orbital tissues.
Therapy for TED with RAI: TAO HAS 15% and 39% risk for development or progression after RAI therapy for hyperthyroidism. The majority of patients developing TED after RAI treatment had mild and transient disease requiring no treatment.44
Thyroid dysfunction: Both hyper- and hypothyroidism have been shown in multiple reports to be associated with increased risk for development or deterioration of TAO.
Thyroxine and tri-iodothyronine levels: Some studies have suggested that circulating tri-iodothyronine (T3) or thyroxine (T4) may also be associated with GO.
Q.2. What is Rundles curve/natural course of TAO ?
Ans. Rundle conceptualized two distinct phases for TED, which is graphically represented in his famous ‘Rundle's curve’. Rundles curve represent the natural course of TED. It helps in understanding and managing TED. It has two stages:
  1. An initial active inflammatory phase which is associated with by periorbital erythema and edema, conjunctival chemosis, orbital inflammation and congestion, associated with upper lid retraction, proptosis, and occasionally diplopia. The inflammatory phase typically lasts for a period between 6 and 24 months
  2. This is followed by a quiet, minimally inflammatory chronic fibrotic phase which is associated with orbital fibrosis, glycosaminoglycan deposition and enlarged extraocular muscles. There are usually no active inflammatory episodes in this phase.
References
  1. Stan MN, Bahn RS. Risk Factors for Development or Deterioration of Graves’ Ophthalmopathy. THYROID. 2010;20:7.
  1. Bhatt R, Nelson CC, Douglas RS. Thyroid-associated orbitopathy: Current insights into the pathophysiology, immunology and management. Saudi Journal of Ophthalmology. 2011;25:15–20.
  1. Barrio-Barrio J, Sabater AL, Bonet-Farriol E, Velázquez-Villoria Á, Galofré JC. Graves' Ophthalmopathy: VISA versus EUGOGO Classification, Assessment, and Management. J Ophthalmol. 2015;2015:249125.
  1. Bahn RS. Graves' ophthalmopathy. The New England Journal of Medicine. 2010;362:726–38.
  1. Dolman PJ. Evaluating Graves’ orbitopathy. Best Practice and Research Clinical Endocrinology and Metabolism. 2012;26:229–48.
 
LACRIMAL GLAND TUMORS
Varsha Vashney, Amar Pujari
 
INTRODUCTION
The lacrimal gland is situated in the superotemporal orbit and it consists of 2 lobes, the orbital lobe, and the much smaller palpebral lobe. The palpebral lobe can be visualized in the superior fornix on lid eversion but not the orbital lobe. Thus, any pathology that affects the orbital lobes only may be missed for a long period of time. Lacrimal gland tumors account for about 10–15% of all orbital tumors.1 The clinician should consider the axiom: “Half and a half; then half again.” Approximately half of all lacrimal fossa masses are inflammatory, and the other half is neoplastic. Out of the neoplastic group, half are the aggressive adenoid cystic carcinoma (ADCC) variety.1,2
In exams, it can be given as a long case.
 
HISTORY
 
Demography
Lacrimal gland tumors are seen more frequently in the third to fourth decade of life (may present from childhood to old age), and the second bimodal peak is in the teenage years.45
 
Chief Complaints
The presentation varies from patients who are asymptomatic but have a slight fullness in the temporal upper lid to those who present with frank proptosis, diplopia, and an encroaching mass lesion.
 
History of Present Illness
All points as described in section proptosis must be recorded carefully while taking history. In addition following points must be noted:
  • History of a long-standing (>1–2 years), non-infiltrating lacrimal gland lesion suggests a benign tumor, such as a pleomorphic adenoma.
  • A shorter history suggests either an inflammatory or a malignant process.
  • Pain most commonly is seen with inflammatory lesions of the lacrimal gland, but adenoid cystic carcinomas and other malignancies also can present with pain secondary to perineural or bony involvement.
  • Malignant lesions characteristically present with a subacute course of proptosis and temporal sensory loss in the distribution of the lacrimal nerve in one-third of patients.
  • Limitation of eye movement, diplopia, and diminished visual acuity can be seen with Large tumors due to distortion of the globe by the firm tumor mass.
  • Benign lesions commonly present with painless inferonasal globe displacement and fullness of the superotemporal lid and orbit. Old photographs may be helpful in establishing the duration of displacement.
  • Acute onset of a painful, erythematous, indurated eyelid suggests inflammation.
  • Other symptoms that may present include facial asymmetry noted by friends, epiphora, exposure symptoms.
 
History of Past Illness
Past ocular history may uncover an episode of trauma, prior periorbital surgery, or periocular tumors that could relate to the present illness. A history of intraocular malignancy such as malignant melanoma might point to the possibility of orbital extension or metastasis.
 
Surgical History
History of surgical removal of similar mass may be there (recurrence is found in pleomorphic adenoma). Incomplete excision of pleomorphic adenoma can lead to relentless recurrences and malignant transformation (Fig. 1). Thus the previous history of biopsy (such as incisional or needle) is important in such cases.
 
History of Systemic Illness
The past general medical history may elicit important diagnostic information. For example, a history of breast cancer might suggest metastasis. A history of systemic inflammatory disease such as sarcoidosis should raise concern for a related orbital inflammatory process.
 
EXAMINATION
 
Systemic Examination
A detailed general examination is carried out to rule out any systemic metastasis. Preauricular lymphadenopathy from regional metastasis in malignant lesions must be ruled out. Signs of primary elsewhere in the body must be looked for.
 
Ocular Examination
Following points must be noted in ocular examination:
  • Visual acuity: Diminished visual acuity can be seen with rapidly progressive lesions. Patients with induced hyperopia from an orbital mass may show a significant asymmetric refractive error.
    zoom view
    Fig. 1: Adenoid cystic carcinoma of lacrimal gland
  • 46Eyeball: Displacement of the globe with or without proptosis can occur. The displacement is characteristically nonaxial with inferomedial globe displacement.
  • Ocular balance and ductions: Binocular patients should be examined for latent or manifest ocular deviations and the approximate extent of uniocular ductions in the four cardinal positions estimated. A forced duction (traction) test under topical anesthesia will assist differentiation of neurological from mechanical causes of restricted eye movements. Likewise, retraction of the globe during an active duction suggests fibrosis of the ipsilateral antagonist muscle, this being a common sign with chronic orbital myositis.
  • Lids: An S-shaped contour to the upper lid (the lateral half of the eyelid lies lower than the medial half) is characteristic for lacrimal gland lesions, but it is relatively nonspecific to the type of tumor. A firm, rubbery, nontender mass can be seen with either benign or lymphoproliferative lesions.
  • Complete examination of mass should be done (as discussed in chapter proptosis) which includes size, shape, site, margins, edges, consistency, mobility, adherence to overlying skin and underlying bone, color of skin, temperature of skin, reducibility and compressibility, increase in size with valsalva maneuver, pulsations, and trans illumination.
  • Conjunctiva, cornea: Signs of inflammation such as congestion and chemosis may be present with dacryoadenitis, tumors or infiltration of the lacrimal gland. A “salmon patch” subconjunctival lesion may be present and is characteristic of lymphoma. Signs of exposure keratopathy (punctate defects or epithelial defect with infiltrates) may be present. A reduced Schirmer's test may indicate towards an inflammatory lesion (e.g. Sjögren syndrome).
  • Pupils: Usually the pupillary reaction is normal. RAPD may be present if the mass is compressing or infiltrating the optic nerve.
  • Anterior segment: Usually normal. Raised IOP may be present due to globe compression.
  • Posterior segment: (Slit-lamp biomicroscopic examination using a 90D/78D lens and indirect ophthalmoscopy)
    • Vitreous, optic disc and macula: Are usually normal.
    • Choroidal folds may be present, resulting from globe indentation by an orbital mass.
 
INVESTIGATIONS
Following investigating modalities are used in a suspected case of lacrimal gland tumor:
 
High-resolution Computed Tomography (HR-CT)
  • Benign tumors: Pleomorphic adenomas appear as well-defined, sometimes nodular and non-homogeneous lesions that show moderate enhancement with intravenous contrast. Palpebral lobe tumors lie anterior to the orbital rim, whereas expansion of the lacrimal fossa with preservation of intact cortical bone is seen in most cases of orbital lobe adenoma. Molding of the lacrimal gland fossa on CT scan is a hallmark of benign growth. Discrete calcification may also be present in a minority of cases, and indentation of the globe is common with larger tumors.
  • Malignant tumors: These are poorly defined margins, with infiltration into surrounding tissues, and bone. Calcification occurs in about one-third of carcinomas but is diffuse compared to pleomorphic adenomas. In contrast to the hard pleomorphic adenomas that flatten the globe, rapidly growing and softer lesions (such as carcinoma and lymphoma) tend to mold to its surface.
 
Histologic Findings
Histologic examination of pleomorphic adenomas reveals evidence of both epithelial and mesenchymal differentiation. The proliferation of benign epithelial cells usually is arranged in a double layer to form lumens. Stromal differentiation can be seen in the formation of bone and cartilage.
ADCC are derived from duct cells, and they form spaces into which basement membrane—like material is deposited. This confers a cribriform or “Swiss cheese” appearance to the tissue, which is characteristic of ADCC.47
 
Immunohistochemistry
This may be helpful in distinguishing between inflammatory, benign, and malignant lymphoproliferative lesions. Immunohistochemistry is a laboratory modality that uses special markers to demonstrate the presence of specific antigens in target tissues. Benign inflammatory lesions (pseudotumor) have a polyclonal morphology, whereas the lymphoid lesions tend to be monoclonal.
 
DIFFERENTIAL DIAGNOSIS
The differential diagnosis of a lacrimal gland mass has been described in Table 1.
Key features [also See Table 2].
 
Benign Tumors
  • Pleomorphic adenoma: It has following important features:
  • Most common intrinsic lacrimal gland lesion
  • Painless, progressive, slow growing
  • Well-circumscribed mass with absence of bony destruction
  • Remove with pseudo capsule intact to decrease risk of recurrence or malignant transformation
  • Histopathologically, comprised of two cell components: Benign epithelial cells arranged in double layer forming ducts stellate spindle cells contained in loose stroma
Table 1   Differential diagnosis of lacrimal gland mass
Non neoplastic
Neoplastic
Dacryops/dacryoadenitis
Dermoid cysts
Hemangioma
Amyloid
Lymphoproliferative diseases
  • Benign lymphoid hyperplasia
  • Atypical lymphoid hyperplasia
  • Malignant lymphoma
Benign tumors
  • Pleomorphic adenoma (benign mixed tumor)
  • Benign fibrous histiocytoma
  • Oncocytoma
  • Myoepithelioma
  • Cystadenoma
Malignant tumors
  • Adenoid cystic carcinoma
  • Malignant mixed tumor (carcinoma expleomorphic adenoma)
  • Adenocarcinoma
  • Mucoepidermoid carcinoma
  • Squamous cell carcinoma
  • Acinic cell carcinoma
  • Malignant oncocytoma
  • Lung and breast metastases
 
Myoepithelioma
  • Rare tumor with biological behavior similar to that of a pleomorphic adenoma
  • Five subtypes: Spindle, plasmacytoid, epithelial, clear and mixed.
 
Oncocytoma
  • Rare tumor secondary to metaplasia of ductular cells (epithelial origin).
  • Large, eosinophilic cells rich in mitochondria “Warthin Tumor” (Cystadenolymphoma)
  • Commonly presents as an epithelial neoplasm of the salivary glands (lacrimal gland is an unusual location).
  • Epithelial columnar cells arranged in solid nests or lining cystic spaces.
 
Malignant Tumors
 
Adenoid Cystic Carcinoma
  • Bimodal distribution with peak incidences in second and fourth decades of life
  • Periocular pain (severe pain due to perineural spread), mild ptosis, proptosis, downward and inward displacement of the globe
  • Bony erosion, bone destruction, and soft tissue calcification on CT
  • High mortality rate (intra-arterial cytoreductive chemotherapy may improve survival)
  • Sheets of epithelial cells arranged in solid or cribriform pattern resembling a glandular structure is characteristic.
 
Primary Adenocarcinoma
  • Rare tumor with clinical findings similar to adenoid cystic carcinoma.
  • Pleomorphic, mitotically active cells arranged in sheets and cords.48
Table 2   Summary of major lacrimal gland tumors
Types of lesion
Clinical features
Imaging features
Histopathologic features
Treatment
Pleomorphic adenoma (benign mixed tumor)
Most common epithelial tumor
Painless, progressive, slow-growing mass on superotemporal area of upper eyelid,
Nontender, firm, well-contoured mass.
Variable proptosis,
Diplopia loss of vision rare
CT—round to oval well circumscribed mass in the lacrimal fossa, with bony expansion and excavation, no bony destruction.
USG—round to oval mass with medium to high reflectivity and regular internal structure
Two morphologic cell components: benign epithelial cells arranged in a double forming ducts and stellate spindle cells contained in a loose stroma. Epithelial cells in the stroma can undergo metaplasia with cartilaginous, fibrous or myxoid characteristics
Modified lateral orbitotomy and excision using an extraperiosteal approach for the lateral portion
For an anteriorly situated palpebral lobe tumor, isolated dacryoadenectomy via a transcutaneous or transconjunctival approach
Oncocytoma
Rare affects elderly females
caruncle most common site
Large, eosinophilic cells rich in mitochondria
Complete surgical excision
Cystadenoma (Warthin's tumor)
Rare
Clinical characteristic is similar to pleomorphic adenoma
Similar to those of a pleomorphic adenoma
Epithelial columnar cells arranged in solid nests or lining cystic spaces.
Often contains an exudative fluid component and a lymphoid infiltrate with focal follicular organization
Complete excision of the globular cystic mass with preservation of the thin capsule
Adenoid cystic carcinoma
Periocular pain, mild ptosis, proptosis, brow numbness and diplopia.
Rapid progression symptoms are typically present for 6 months, and almost always less than one year
Globular lacrimal gland mass with irregular borders, bony erosion, bone destruction and soft tissue calcification. Contiguous tumor extension to adjacent area
Sheets of epithelial cells arranged in either solid or cribriform patterns with spaces into which basement membrane like material is deposited. (Swiss-Cheese)
En bloc, excision of the orbit and its contents, including the orbital roof, the lateral wall, the lids, and the anterior portion of the temporalis muscle where the zygomatico frontal and zygomatico-temporal nerves extend.
Adjunctive postoperative radiotherapy49
Malignant mixed tumor
The average age at diagnosis is 50 years
This tumor may arise de novo, because of malignant transformation following an incomplete excision of a benign adenoma, or as malignant transformation years after diagnosis of a presumed benign adenoma
Similar to adenoid cystic carcinoma, may show a bilobed appearance
Histopathologically, the malignant component may be attached to and arise from the benign mixed aspect of the tumor, yielding a bilobed appearance
Complete surgical resection
Mortality is high
Mucoepidermoid carcinoma
Rarely
Locally aggressive
Average age at presentation of 49 years
Male: Female 2:3
Similar to adenoid cystic carcinoma
Epidermoid and mucus-secreting cells arranged in a pattern of cords and islands.
The mucus-secreting cells and cystoid spaces within the specimen stain positively with mucicarmine, alcian blue stains and Periodic acid-Schiff reaction
Excision with or without adjuvant radiotherapy. Advanced stage has a worse prognosis, require exenteration and radiotherapy
Carcinosarcoma
Carcinosarcoma may arise from a pleomorphic adenoma
Considered in the differential diagnosis of a lacrimal gland mass, if sarcomatous components are encountered on histologic examination
Management requires complete excision of the lesion
 
Pleomorphic Adenocarcinoma (Malignant Mixed Tumor)
  • May arise de novo, as consequence of malignant transformation following incomplete excision of benign adenoma or as malignant transformation of a presumed benign adenoma.
  • Well circumscribed, pseudo capsulated.
 
TREATMENT35(Table 3)
 
Pleomorphic Adenoma
It should be excised intact with a cuff of normal tissue. Palpebral lobe tumors can be resected through an upper lid skin crease incision Stallard-Wright incision (anterior orbitotomy). Orbital lobe tumors can be approached through a lateral orbitotomy approach.
It is important to avoid any intraoperative spillage during surgery. Thus, a buffer of normal tissue should always be maintained around the tumor mass. If intraoperative spillage of cells occurs through cautery and lavage of the operative field has to be done. If the periosteum is already destroyed, the breach should be treated by strict surgical isolation and Cyanoacrylate glue may be applied to minor capsular breaches during surgery. Excision of the orbital lobe alone, with preservation of palpebral lobe, reduces the incidence of dry eye and secondary corneal disease.50
Table 3   Difference between pleomorphic adenoma and adenoid cystic carcinoma
Features
Pleomorphic adenoma
Adenoid cystic carcinoma
Pain
Painless
Periocular pain often sever
Course
Slow-growing mass
Fast growing
Proptosis
Variable
Proptosis with downward and inward displacement of the globe
Associated features
Decreased vision and diplopia rare
Brow numbness is characteristic, diplopia, mild ptosis
Palpation
Non-tender, firm, well contoured mass
Firm irregular bordered mass
Histological features
Benign epithelial cells arranged in a double layer forming ducts and stellate spindle cells contained in a loose stroma
Sheets of epithelial cells arranged in either solid or cribriform patterns (Swiss-Cheese pattern) that mimic glandular structure
Radiological features
  • Round to oval well-circumscribed mass in the lacrimal fossa
  • Bony expansion and excavation but no bony destruction
  • The posterior edge of the lesion typically exhibits a curved contour that molds to the adjacent orbital bone
  • Globular lacrimal gland mass with borders that are irregular
  • Associated with bony erosion, bone destruction and soft tissue calcification
  • Tumor extension toward the medial orbit, apex and the temporalis fossa
Biopsy (other than excision) should not be attempted in any suspected case of pleomorphic adenoma. If it has been inadvertently biopsied, the biopsy tract and the tumor should be meticulously excised since recurrent pleomorphic adenoma is typically infiltrative and may need extensive tissue resection or exenteration.
 
Adenoid Cystic Carcinoma
It depends upon the extent of the tumor. For tumors that are localized to the orbit excision of the tumor and adjacent tissues should be done. Advanced tumors require surgical resection followed by external beam radiation therapy (EBRT). ADCC is often extensive and requires orbital exenteration or midfacial resection. EBRT delay the growth or recurrence of the tumor. Brachytherapy (locally implanted radioactive plaques seeds) may also give results similar to EBRT. Chemotherapy does not have a recognized role in the treatment of adenoid cystic carcinoma.
The malignant mixed tumor is treated with local excision followed by irradiation.
Metastatic tumor carries a poor prognosis. The target is to provide palliative therapy through orbital irradiation or chemotherapy.
 
VIVA QUESTIONS
Q.1. Most common lacrimal gland tumors.
Ans.
  • Pleomorphic adenomas account for almost all benign tumors of the lacrimal gland.
  • Adenoid cystic carcinoma is the most common (76%) malignant epithelial tumor.
  • Carcinoma arising in pre-existing pleomorphic adenoma (malignant mixed tumor) is the second most common malignancy of the lacrimal gland.51
  • Lymphoma accounts for about 10–14% of all lacrimal gland masses and may be part of a systemic disease.
Q.2. Where do you find a “salmon patch lesion”?
Ans. Salmon patch subconjunctival lesion is a characteristic non-tender firm reddish fleshy mass of conjunctival lymphoma. It may be an extension of orbital or intraocular lymphoma. Only 25% patients of orbital/adnexal lymphoma have conjunctival involvement. Most commonly, it is confused with chronic follicular conjunctivitis. Histologically, these are non-Hodgkin's lymphoma of low-grade B-cell variety.
Q.3. Differential diagnosis of a lacrimal fossa mass.
Ans. Refer to differential diagnosis.
Q.4. How do you differentiate pleomorphic adenoma from adenoid cystic carcinoma?
Ans. See Table 3.
Q.5. What is “Swiss cheese” pattern of ADCC?
Ans. It refers to the microscopic picture of ADCC. Closely packed small, densely stained cells aggregate around large ovoid spaces containing hyaline or mucin. This resembles the Swiss cheese pattern and hence the description.
Q.6. Prognosis of ADCC.
Ans. The prognosis is poor in cases of ADCC. The overall 5 year survival rate of all adenoid cystic carcinomas was 47%. This number reduces to 20% after 13 years and 22% after 15 years. Most cases recur within 2 years of treatment. Intracranial spread can occur due to the propensity of perineural invasion that results in the death of the majority of patients.
References
  1. Albert DM, Miller JW, Azar DT. Albert and Jakobiec's principles and practice of ophthalmology; 2008.
  1. Bowling B. Kanski's clinical ophthalmology: A systematic approach, 8th edn. Edinburgh: Elsevier,  2015.
  1. von Holstein SL, Coupland SE, Briscoe D, Le Tourneau C, Heegaard S. Epithelial tumors of the lacrimal gland: a clinical, histopathological, surgical and oncological survey. Acta Ophthalmol. 2013;91(3):195–206. doi: 10.1111/j.1755-3768.2012.02402.x. Epub 2012 Apr 4. Review.
  1. Bernardini FP, Devoto MH, Croxatto JO. Epithelial tumors of the lacrimal gland: an update. Curr Opin Ophthalmol. 2008;19(5):409–13. Review.
  1. Alkatan HM, Al-Harkan DH, Al-Mutlaq M, Maktabi A, Elkhamary SM. Epithelial lacrimal gland tumors: A comprehensive clinicopathologic review of 26 lesions with radiologic correlation. Saudi J Ophthalmol. 2014;28(1):49–57.
 
52SHORT CASES
 
CONGENITAL PTOSIS
Aditi Dubey, Amar Pujari
 
INTRODUCTION
Drooping of the eyelid is known as ptosis. It may be present at birth, or it may develop later in life. If a droopy eyelid is present at birth or within the first year of life, the condition is called congenital ptosis. In most cases of congenital ptosis, the problem is isolated and does not affect the vision.13
Congenital ptosis can be given as a short case in exams.
 
HISTORY
All pediatric patients presenting with either unilateral droopy eyelid or bilateral droopy eyelids need a thorough history and examination (kindly see the section of ptosis, long case).
 
Chief Complaint
Parents usually bring the child with the history of drooping of the eyelid (Fig. 1) or narrow palpebral fissure since birth.
 
History of Present Illness
The onset, progression, and other associated abnormalities such as deviation of eyes, nystagmus, face turn and any relation to the amount of ptosis to jaw movement.
 
Medical History
A careful medical history regarding malignancy should be obtained. Metastatic or primary orbital tumors can result in malpositioning of the eyelid.
 
Family History
A patient with a strong family history of congenital ptosis may not need an extensive work-up. Family photographs can help determine onset or variability of the ptosis.
zoom view
Fig. 1: Simple severe congenital ptosis
 
History of Drug or Allergic Reactions
A history of drug or allergic reactions may be helpful. Allergic reactions can result in eyelid edema and droopy eyelid.
 
History of Trauma
Orbital wall fractures (pseudoptosis with enophthalmos) or IIIcranial nerve palsy from trauma may result in ptosis.
 
EXAMINATION
A case of congenital ptosis must be evaluated in detail as described in the chapter of long case ptosis. In cases of congenital ptosis following should be taken care of:
  • Visual acuity: Risk of amblyopia is there in case of severe ptosis. The amblyopia can occur due to occlusion amblyopia or rarely due to astigmatism induced by the compression of the droopy eyelid.
  • Refractive error and cycloplegic refraction should be recorded in all cases.
  • In infants, make sure that the baby can fixate and follow objects with each eye individually.53
  • The patient should be evaluated for strabismus (misalignment).
  • Serial external photographs of the eyes and the face may be included in the patient's record for documentation.
  • Tear function should be evaluated.
  • Corneal sensitivity should be tested (if possible) may be a difficult test in young pediatric patients.
  • The pupillary size and the iris color differences between the eyes should be examined for Horner syndrome.
  • Palpebral fissure distance.
  • Lid position in downgaze (the ptotic lid appears higher in downgaze).
 
Levator Function
Measurement of levator function in small children is a difficult task, as the child allows no formal evaluation. The presence of lid fold and increase or decrease its size on the movement of the eyelid gives us a clue to the levator action. The presence of anomalous head posture like the child throwing his head back suggests a poor levator action.
 
Iliff Test
This test can be performed in the first year of life to evaluate the levator function. The upper eyelid of the child is everted as the child looks down. If the levator action is good lid reverts on its own.
 
INVESTIGATION
A routine case does not require any specific investigations except those required for general anesthesia if surgery planned (hemoglobin, urea, creatinine, bleeding and clotting time). Neuroimaging (MRI or CT) is indicated in following conditions:
  • If history not consistent and onset not clear
  • Other neurologic findings along with ptosis are present
  • Orbital wall fracture suspected with history of trauma
  • Visible or palpable lid mass
  • Suspected orbital tumors (e.g. lymphoma, leukemia, rhabdomyosarcoma)
  • New onset of Horner syndrome with or without other neurologic findings
  • New onset of third cranial nerve palsy with or without other neurologic findings
  • Globe displacement with either enophthalmos or proptosis.
Differential diagnosis/classification/management has been covered in detail in the long case, ptosis part.
 
MANAGEMENT
The following points are considered while performing the surgery.
 
Timing of Surgery
It is advisable to wait till 4-5 years of age for surgical correction when the tissues are mature enough to withstand the surgical trauma and a better assessment and postoperative care is possible due to improved patient co-operation.2,3 Urgent surgery is indicated in children with severe ptosis developing amblyopia. In such cases, sling surgery is done.
 
Surgical Approach
It is based on whether the:
  • Ptosis is unilateral or bilateral
  • Severity of ptosis
  • Levator action
  • Presence or absence of abnormal ocular motility, jaw-winking phenomenon or blepharophimosis syndrome.
 
Aim of Surgery
Target is to lift the ptotic lid above the papillary aperture when the eyes are in the primary position. The height of the two lids regardless of whether the ptosis is unilateral or bilateral should be equal. There should also be adequate mobility of the lid when blinking, a normal lid fold and no diplopia.
 
Surgical Procedure
The choice of surgery is given below:
  • Fasanella-Servat operation
    • Mild ptosis (<2 mm or less)
    • Levator action >10 mm
    • Well-defined lid fold-no excess skin54
      Table 1   Berke's criteria for levator resection
      Degree of ptosis
      Levator function
      Amount of levator resection
      Ideal preoperative correction
      1.5–2 mm (mild)
      Good (8 or more)
      Good (8 or more)—usual
      Small (10–13)
      Moderate (14–17)
      Under correct by
      1–3 mm
      3 mm (moderate)
      Fair (5–7)
      Poor (rare, 4 or less)
      Large (18–22)
      Maximal (23 or more)
      Match the level of normal lid and correct ptosis fully
      4 or more (severe)
      Fair (Sometimes, 5–7)
      Poor (usual, 4 or less)
      Super maximal (27 or more)
      Frontalis sling
      Over correct by 1–2 mm
  • Levator resection
    • Mild/moderate/severe ptosis
    • Levator action ≥4 mm
  • Brow suspension ptosis repair
    • Severe ptosis
    • Levator action <4 mm
    • Jaw-winking ptosis (along with LPS excision) or blepharophimosis syndrome.
In cases with bilateral congenital ptosis, simultaneous bilateral intervention in the two eyes is needed. However, in cases where gross asymmetry exists between the two eyes, the eye with a greater ptosis is operated first and the other eye is operated after 6–8 weeks when the final correction of the operated eye can be assessed.
Levator resection is the most commonly performed procedure, there are different criteria to determine the amount of levator resection required. The two important and widely used guidelines are Berkes (Table 1) and Puttermans (Table 2) method.
 
Contraindications to Surgery
Ptosis surgery is relatively contraindicated in presence of following:
  • Poor orbicularis muscle function (lagophthalmos and corneal exposure)
  • Loss of blink reflex
  • Loss of corneal sensitivity
  • Significant dry eye
  • Poor Bell's phenomena.
 
VIVA QUESTIONS
Q.1. What are the causes of congenital ptosis?
Ans.Following are important causes of congenital ptosis:
  • Idiopathic
    Table 2   Putterman's criteria for amount of levator resection
    Levator action
    Recommended lid placement
    2–4 mm
    1 mm above the limbus
    5–7 mm
    1 mm below the limbus
    8 mm or more
    2 mm below the limbus
  • Blepharophimosis syndrome (BPES): Short palpebral fissures, congenital ptosis, epicanthus inverses, and tele-canthus.
  • Third cranial nerve palsy
  • Horner syndrome: Ipsilateral findings of mild ptosis, miosis, and anhidrosis characterize this syndrome.
  • Marcus Gunn jaw-winking syndrome: The motor nerve to the external pterygoid muscle is misdirected to the ipsilateral levator muscle. Lid elevation occurs with mastication or with the movement of the jaw to the opposite side.
  • Birth trauma
  • Periorbital tumor: Neuroblastoma, plexiform neuromas, lymphomas, leukemias, rhabdomyosarcomas, neuromas, neurofibromas, or other deep orbital tumors may produce ptosis and proptosis.
  • Kearns-Sayre syndrome: Progressive external ophthalmoplegia, heart block, retinitis pigmentosa, and central nervous system manifestations. This condition begins in childhood but is rarely present at birth.
  • Myotonic dystrophy.
  • Blepharochalasis: Infiltrative processes that thicken the lids and produce ptosis.55
  • Myasthenia gravis: A defect at the neuromuscular junction.
  • Pseudotumor of the orbit: Ptosis due to inflammation and edema of the eyelid.
  • Pseudoptosis: Less tissue in the orbit (e.g. unilateral smaller eye, fat atrophy, blowout fracture) produces the appearance of ptosis secondary to the decreased volume of orbital contents.
Q.2. Describe the pathology in congenital ptosis.
Ans.Histologically, the levator muscles of patients with congenital ptosis are dystrophic. The levator muscle and aponeurosis tissues appear to be infiltrated or replaced by fat and fibrous tissue. In severe cases, little or no striated muscle can be identified at the time of surgery. This suggests that congenital ptosis is secondary to local developmental defects in muscle structure. Congenital ptosis may occur through autosomal dominant inheritance. Common familial occurrences suggest that genetic or chromosomal defects are likely.
Q.3. Complications of the sling surgery.
Ans.Complications associated with the frontalis suspension procedure for congenital ptosis repair include the following:
  • Granuloma
  • Lid asymmetry
  • Overcorrection with exposure kerato-pathy
  • Undercorrection
  • Infection.
Q.4. What is the prognosis after surgery?
Ans.The repair of congenital ptosis can produce excellent functional and cosmetic results. Careful observation and treatment, amblyopia ptosis can be treated successfully. Patients who require surgical intervention, 50% or more may require repeat surgery in 8–10 years following the initial surgery.
Q.5. Management of complicated ptosis.
Ans. The management of simple congenital ptosis associated with other anomalies is described below:
  • Ptosis with oculomotor abnormalities: In cases, with superior rectus involvement (usually associated with severe ptosis) an inferior rectus recession at times combined with superior rectus resection is carried out on the affected side as the first procedure. To correct the ptosis levator resection with bilateral brow suspension is done later. Knapp's procedure may be done for ptosis associated with double elevator palsy where lateral and medial rectus tendons are transplanted to the area of superior rectus insertion. This does not cause significant limitation of adduction or abduction. Ptosis is corrected 3 months later.
  • Blepharophimosis syndrome: The following sequence is followed:
    • Y-V plasty mustard's double “Z” plasty with transnasal wiring is done as a primary procedure. This gives a good surgical result both in terms of correction of telecanthus as well as deep placement of the medial canthus. The results are long lasting.
    • Brow suspension is carried out 6 months after the first procedure for correction of ptosis.
  • Marcus Gunn ptosis: Mild cases of jaw winking where the jaw winking is minimal can be treated satisfactorily by Fasanella-Servat operation while severe cases or cases where jaw winking is prominent, require bilateral resection of levator aponeurosis and terminal levator with fascia lata brow suspension.
  • Misdirected third nerve ptosis: In cases of misdirected third nerve ptosis where treatment is imperative levator resection with bilateral fascia lata sling is the procedure of choice. Ptosis associated with third nerve palsy is difficult to manage because of poor bell's phenomenon. A crutch glass may be prescribed or a conservative sling surgery may be performed.
References
  1. Bernardini FP, Devoto MH, Priolo E. Treatment of unilateral congenital ptosis. Ophthalmology. 2007;114(3):622–3.
  1. Shields M, Putterman A. Blepharoptosis correction. Curr Opin Otolaryngol Head Neck Surg. 2003;11(4):261–6.
  1. Yilmaz N, Hosal BM, Zilelioglu G. Congenital ptosis and associated congenital malformations. J AAPOS. 2004;8(3):293–5.56
 
ECTROPION
Aditi Dubey, Bijnya Birajita Panda
 
INTRODUCTION
Ectropion is characterized by an eversion or outward turning of the eyelid margin away from the globe. It is a commonly encountered eyelid malposition. It is characterized by rotation of lid margin outwards resulting in its fall away from the globe.1 To make things worse, the constant wiping and rubbing of eyes irritated by the epiphora further aggravates the condition. The underlying factor may vary in each case and an appropriate identification of the type of ectropion and the factor responsible for its occurrence are important in choosing the correct surgical intervention.2,3
In exams, it is usually given as a short case or spot case.
 
HISTORY
 
Chief Complaints
Watering, irritation, grittiness, foreign body sensation or chronic red eye. Symptoms are caused by ocular exposure and inadequate lubrication.
 
Past History
Facial palsy, lid trauma, ocular allergy and previous lid surgery should be taken.
 
EXAMINATION
The routine examination is carried out and the conjunctiva, cornea, and anterior chamber are examined for any signs of inflammation.
Eyelid: There is outward turning of the lid margin. There may be signs of chronic blepharitis.
Conjunctiva: Keratinization and hypertrophy.
Cornea: Changes secondary to exposure may be present.
Schirmer's test: (to rule out dry eye).
Syringing and Jones test I and II (to rule out lacrimal passage obstruction).
 
Tests for Ectropion
 
Pinch Test
To determine the amount of lid laxity. If the lid can be pulled more than 6 mm away from the globe, the lid is lax. If the medial and lateral canthal tendons are lax as well, the lid can be pulled away up to 20–25 mm.
 
Snap-back Test
Downward traction is applied to the lower lid and then released the lid should revert to its normal position, without the aid of a blink in a normal person. However, when laxity is present, the lid is not opposed to the globe.
Grading of lid laxity according to snap-back test
  • Normal: The lid returns to its position immediately on release
  • Grade 1: Approximately 2–3 sec
  • Grade 2: 4–5 sec
  • Grade 3: >5 sec but returns to position on blinking
  • Grade 4: Continues to hang down
 
Inferior Lid Retractor Laxity
Retractor weakness can be demonstrated by observing the lower lid as the patient looks down. Reduction in inferior movement on down gaze and a deep inferior fornix occurs due to laxity or loss of retractor attachment in this area.
 
Medial Canthal Tendon (MCT) Laxity
The lateral excursion of the inferior punctum is measured by pulling the lid laterally. The punctum 57lies lateral to the caruncle at rest and should not be displaced more than 1–2 mm with lateral lid traction. If pulling on the medial canthus allows the punctum to be stretched, it suggests MCT is lax. Laxity is graded as following:
  • Mild: Up to the limbus
  • Moderate: Up to the pupil
  • Severe: Beyond the temporal pupillary border.
 
Lateral Canthal Tendon (LCT) Laxity
The lateral canthal angle should be evaluated with the lid at rest. The lateral canthus should have an acute angular contour and should lie 1–2 mm medial to the lateral orbital rim. A rounded appearance of the canthus indicates laxity. The lateral part of the lid if pulled medially should not result in more than 1–2 mm movement of the lateral canthal angle, in absence of laxity.
 
Position of the Lacrimal Puncta
Punctum alone can be everted or the whole lid may be everted. In a normal lid, the inferior punctum is directed posteriorly against the globe and should not be visible without pulling the lid downward. Direction of the punctum away from the globe is the earliest sign of medial lid ectropion and can be graded as follows:
  • Mild: Puncta are not opposed to the globe on looking up.
  • Moderate: Puncta are not opposed to the globe even in primary gaze.
  • Severe: Palpebral conjunctiva and fornix are exposed.
 
Cicatricial Skin Changes
Vertical shortening of the anterior lamella—like signs of repair of lid laceration or scar of excision of the tumor should be looked for.
 
Orbicularis Muscle Weakness
The facial nerve must be examined to rule out ectropion due to paralysis of the seventh nerve. Lagophthalmos and reduced force of contraction on forced eyelid closure demonstrate orbicularis muscle weakness. Other signs of facial palsy such as brow ptosis, loss of forehead wrinkles, absent nasolabial fold and drooping of the angle of the mouth should also be looked for.
 
CLASSIFICATION
Ectropion can be classified as following:
  • Congenital ectropion: Rare, associated with congenital epiblepharon.
  • Acquired ectropion: Can be further classified as following on the basis of pathogenesis:
    • Involutional ectropion: It is the most common variety. Multiple factors are responsible for its development e.g. horizontal lid laxity, medial canthal tendon laxity, etc. which are all normal aging changes of the lid (Fig. 1).
    • Cicatricial ectropion: Lid margin is pulled away from the globe due to the shortage of skin e.g. congenital shortage (Fig. 2), trauma, burns, cicatrizing skin tumors, allergies, etc. it may be unilateral or bilateral/localized or generalized depending on the cause.
      zoom view
      Fig. 1: Senile ectropion
      zoom view
      Fig. 2: Paralytic ectropion due to tight skin in a collodion baby
    • 58Mechanical ectropion: Tumor or cyst near the lid margin mechanically pulling down the lid.
    • Paralytic ectropion: VII N palsy resulting in sagging and downward displacement of paralyzed orbicularis muscle.
 
GRADING
 
Grade of Orbito-lid Apposition
  • Grade 0: With normal lid-globe apposition
  • Grade 1: With punctal eversion
  • Grade 2: With partial lid eversion and scleral show
  • Grade 3: With conjunctival hyperemia and thickening
  • Grade 4: As for grade 3 with exposure keratitis.
 
Grades of Ectropion
  • Grade 1: Punctal eversion
  • Grade 2: Eversion of sharp posterior lid margin
  • Grade 3: Palpebral conjunctival exposure
  • Grade 4: Exposure of the fornix.
 
TREATMENT
Factors considered for selection of surgery for ectropion:
  • Basic cause of ectropion
  • Secondary mechanisms coexisting with the basic pathology
  • Grade of ectropion
  • Identify the defects in various components of lid lower
  • Excess lid skin
  • Laxity of LCT/MCT and its severity
  • Shortening of posterior lamella comprising of tarsoconjunctiva
  • Any mass lesion in lid causing ectropion
  • Any scarring whether localized or generalized
  • Systemic disease causing scarring of tarso-conjunctiva.
 
Surgical Management of Ectropion
 
Involutional Ectropion
  • Mild-to-moderate ectropion mainly affecting lateral lid: Full thickness pentagonal wedge resection of the lid.
  • Mild ectropion with an excess of skin: Modified Kuhnt-Szymanowski procedure: blepharoplasty with a base up lateral triangle and excision of full thickness wedge of lid beneath the blepharoplasty flap.
  • Moderate ectropion—generalized or affecting lateral lid:
    Lateral tarsal strip: In this, a horizontal incision is made and inferior crus of LCT is cut, triangular portion of the temporal lid is resected sparing tarsus and the tarsal strip is sutured with a mattress suture in a superotemporal direction to the periosteum. Smith and Lisman propose an alternative method where after excising the anterior lamella of the lid on temporal aspect the periosteum is exposed in a superotemporal direction and the tarsal strip is sutured to it (Fig. 3).
  • Marked ectropion: Double wedge resection with lateral tarsal strip
  • Extreme ectropion: Temporalis muscle transfer Involutional ectropion affecting only the medial aspect
  • Only punctal eversion present and no lid laxity: Medial conjunctivoplasty
  • Horizontal lid laxity is present but MCT is not lax: Lazy–T (Excision of tarsoconjunctiva combined with full thickness wedge excision of lid)
  • Horizontal lid laxity which is due to MCT laxity: MCT plication or resection depending on severity of laxity.
 
Management of Cicatricial Ectropion
Correction of a cicatricial ectropion requires lengthening of the cutaneous surface and correction of any associated factors-resection of subcutaneous cicatrix or horizontal lid lengthening.
zoom view
Fig. 3: Cicatricial ectropion (Figure 1) after surgery (lateral tarsal strip procedure)
59Following surgeries have been described:
  • Localized: Z-plasty (Elschnig's operation)
  • V- Y operation
  • Severe or generalized cicatricial ectropion: Full-thickness skin grafting.
 
Management of Paralytic Ectropion
Management involves giving support to the lower lid or strengthening of the lower lid. Support can be given medially, laterally or to the lower lid as a whole. In long standing cases associated with cheek ptosis, a cheek lift/mid-face lift may be necessary. Following surgeries have been described:
  • Only medial ectropion: Medial canthoplasty
  • Generalized lid laxity: MCT plication + lateral canthal sling
  • Medial with MCT laxity: MCT resection.
 
Management of Mechanical Ectropion
Masses near the lid margin causing the ectropion should be excised. Excision should be as vertical as possible and it is important to avoid scar formation/ skin shortening.
 
VIVA QUESTIONS
Q.1. Pathogenesis of involutional ectropion.
Ans.Ectropion involves the lower lid more commonly than the upper lid. There is decreased resilience and increased laxity of periocular tissues due to age-related microinfarction and secondary atrophy. This inadequate support and effect of gravity cause more pronounced stretching of the lower lid increasing the burden on suspensory canthal tendons and resulting in ectropion. Primary abnormality is laxity of the lateral canthal tendon. Other contributory factors include:
  • Horizontal lid laxity
  • Medial canthal tendon laxity
  • Punctal malposition
  • Vertical tightness of the skin
  • Lower lid retractors disinsertion or laxity
Q.2. Congenital ectropion.
Ans.Congenital ectropion is a rare entity. It is more often associated with blepharophimosis syndrome (BPES Type 2), Down syndrome or ichthyosis. In rare cases, congenital ectropion occurs as an isolated finding. It is caused by a vertical insufficiency of the anterior lamella of the eyelid. Complications include chronic epiphora and exposure keratitis. Management is as follows:
  • Mild congenital ectropion—no treat-ment.
  • Severe and symptomatic—horizontal tightening of the lateral canthal tendon and vertical lengthening of the anterior lamella by means of a full-thickness skin graft.
A complete eversion of the upper eyelids occasionally occurs in premature infants transiently due to orbicularis slippage/ lamellar slippage, inclusion conjunctivitis, anterior lamellar inflammation or shortage, or Down syndrome. Treatment includes topical lubrication, short-term patching of eyes, full-thickness sutures or a temporary tarsorrhaphy.
Q.3. What are the disadvantages of lid resection procedures?
Ans.
  • Does not correct the underlying physiological abnormality
  • Causes lid notching
  • Causes lid shortening
  • Causes loss of meibomian glands.
References
  1. Bosniak SL, Zilkha MC. Ectropion. In: Smith's Ophthalmic plastic and reconstructive surgery, Mosby;  2nd edn; 1998. pp. 290–307.
  1. Bergin DJ. Anatomy of the eyelids, lacrimal system, and orbit. In: McCord CD, Tanenbaum, Nunery WR (Eds). Oculoplastic surgery. 3rd edn. New York: Raven press;  1995. pp. 51–84.
  1. Robinson FO, Collin RO. Ectropion. In: Yanoff M, Duker JS (Eds). Ophthalmology, 2nd edn. India: Mosby.  2006;1:676–83.60
 
ENTROPION
Aditi Dubey, Ritu Nagpal
 
INTRODUCTION
Entropion is a condition in which eyelid turns inward. This causes the eyelashes or the eyelid margin to rub against the eyeball and results in irritation, watering, redness, keratitis and even corneal perforation. It may occur at any age but occurs primarily because of advancing age. It can be missed easily and thus one should specifically look at the eyelid to diagnose it.13
In exam, it can be given as a short case.
 
HISTORY
 
Chief Complaints
Patients with entropion commonly complain of:
  • Foreign body sensation
  • Frequent eye infections
  • Red eyes
  • Watering.
 
Past History
History of onset is important to rule out congenital component.
Age of patient: In children, congenital entropion is rare and should be definitely differentiated from epiblepharon.
History of ocular trauma, facial burn, Stevens-Johnson syndrome is important for cicatricial entropion.
History of ocular surface irregularity or any other painful ocular pathology (acute spastic entropion).
 
EXAMINATION
  • Lid margin is found in-turned (Fig. 1). Depending upon the degree of in turning it can be divided into three grades.
    • Grade I: Only the posterior lid border is in rolled
    • Grade II: Entropion includes in turning up to the intermarginal strip
    • Grade III: The whole lid margin including the anterior border is in turned.
  • Lid laxity:
    • The pinch test: To determine the amount of lid laxity. If the lid can be pulled more than 6 mm away from the globe, the lid is lax. If the medial and lateral canthal tendons are lax as well, the lid can be pulled away up to 20–25 mm.
    • There may a hump on lower eyelid due to overriding of preseptal orbicularis over the pretarsal part.
    • It is often associated with an absence of the downward excursion of the eyelid in down gaze due to the weakness of the lower lid retractors. Excursion of the lower lid in down gaze usually 3–4 mm—loss of movement indicates retractor weakness/disinsertion.
  • Snap-back test: Perform this test by pulling the lower lid away and down from the globe for several seconds. If the lid resumes position, note the time required for the lid to return to its original position without the patient blinking. The snap-back test provides a good idea of relative lower lid laxity. Lids with normal laxity immediately spring back to original position; the longer this takes, the more laxity is present.
    • Grades
      • Normal: Lid returns immediately on release
      • Grade 1: Approximately 2–3 sec
      • Grade 2: 4–5 sec
        zoom view
        Fig. 1: Entropion causing keratopathy
      • 61Grade 3: >5 sec but returns to position on blinking
      • Grade 4: Continues to hang down
  • Medial canthal laxity test: Perform this test by pulling the lower lid laterally from the medial canthus. Measure displacement of the medial punctum. Greater distance equates to more laxity. Normal displacement ranges from only 0–1 mm.
    • Grades
      • Mild—up to the limbus
      • Moderate—up to the pupil
      • Severe—beyond the temporal pupillary border.
  • Lateral canthal laxity test: Perform this test by pulling the lower lid medially from the lateral canthus. Measure displacement of the lateral canthal corner. Greater distance equates to more laxity. Normal displacement ranges from only 0–2 mm. Assign grades on a scale from 0–4 (0 = normal laxity, 4 = severe laxity).
  • Bell phenomenon: Instruct patient to attempt eye closure while the examiner holds lids open. If eyes move up, the test indicates a positive result for Bell phenomenon.
  • Orbicularis muscular tone: Ask the patient to squeeze eyes shut. Note how much worse the entropion is immediately after opening.
    • Grades of orbicularis muscle tone
      • Grade 0 = no paralysis
      • 1 = weak
      • 2 = normal
      • 3 = overactive
      • 4 = spastic
  • The digital eversion test can be done to distinguish cicatricial component: Observe directly by everting the lids. It can also be ascertained by pulling the lid superiorly if it does not reach 2 mm above lower limbus; lid is vertically deficient.
  • Slit-lamp examination: To look for corneal status and other evidence of dryness, punctuate keratopathy due to blepharitis, meibomitis, trichiasis, foreign bodies, corneal scarring (Fig. 2) and dry eyes.
  • Fluorescein staining: This test is essential when looking for signs of corneal damage. It can detect damage from lashes or lid skin rubbing on the cornea.
  • Schirmer test: To rule out other causes of dryness.
    zoom view
    Fig. 2: Entropion with eyelash rubbing cornea causing keratopathy
  • Lacrimal system patency assessment: Done by syringing, dye disappearance test and jones dye test I & II.
 
CLASSIFICATION
Entropion can be classified into congenital, involutional, cicatricial and acute spastic.13
  • Congenital entropion: Occurs due to hypertrophy of the anterior lamella. Mostly mild, resolves with time.
  • Involutional entropion: An age-related condition caused by the laxity of tarsus, its medial and lateral canthal tendons, lower lid retractors, along with the over-riding of the orbicularis oculi muscle.
  • Cicatricial entropion: Occurs due to scarring and shortening of the posterior lamella due to chemical injury, infection or Stevens-Johnson syndrome.
  • Acute spastic entropion: Follows ocular irritation or inflammation.
 
Most Common Types of Entropion
  • Lower lid—involutional entropion
  • Upper lid—cicatricial entropion.
 
MANAGEMENT
 
Nonsurgical Management
In cases before proceeding to the definitive surgery these medical management plans can be followed.
  • Eyelid taping to the malar eminence
  • Injecting Botox into the orbicularis muscle.62
 
Surgical Management
 
Senile Entropion
  • Rotational sutures
  • Lateral tarsal strip
  • Lower eyelid retractor reinsertion.
 
Spastic Entropion
First treat the underlying condition that might infection or irritation of ocular surface. Followed by either rotational sutures can be passed or Botox can be injected.
 
Cicatricial Entropion
  • Tarsal fracture
  • Transverse blepharotomy with marginal rotation
  • For severe cases, posterior lamellar lengthening using mucous membrane grafting.
 
Surgery Names
  • Weiss procedure (transverse blepharotomy with everting sutures)
  • Quickert everting sutures
  • Jones retractor plication.
 
VIVA QUESTIONS
Q.1. Congenital entropion.
Ans.
  • Extremely rare
  • Inversion of entire tarsus and lid margin
  • Epiblepharon and horizontal tarsal kink are to be differentiated
  • Surgery (Hotz procedure): Minimal ellipse of skin and orbicular is excised from the medial two-thirds of the lower lid. Skin is fixed to the lower edge of the tarsus.
Q.2. Differentiate between congenital entropion and epiblepharon?
Ans. See Table 1.
Q.3. Involutional entropion.
Ans.
  • Often seen in elderly patients, particularly women.
  • Causes great discomfort as well as problems with clear vision due to constant watering.
  • The appropriate procedure depends on the degree of entropion, keratinization, and distortion of the lid margin and eyelashes and analysis of posterior lamellar shortening and scarring.
  • For cases with mild to moderate degree of cicatrization—tarsal wedge resection or the tarsal fracture procedure.
  • In severe or recurrent cases: Posterior lamellar grafting procedure to lengthen the posterior lamella.
Q.4. Causes of cicatricial entropion.
Ans.
  • Trachoma
  • Acid and alkali burn
  • Ocular pemphigus
  • Leprosy
  • Severe membranous conjunctivitis
  • Stevens-Johnson syndrome
    Table 1   Differences between congenital entropion and epiblepharon
    Features
    Epiblepharon
    Congenital entropion
    Extrafold of skin
    Fold of skin overlapping the lid margin is present medially
    Absent
    Occurrence
    Common
    Rare
    Lid affected
    Lower lid
    Both
    Lid margin
    Not turned inwards
    Entire lid margin turned inward
    Direction of eye lashes
    Straight up and lie flat against the cornea
    Turned inward
    On pulling down the skin
    Lashes turn out but the margin of the lid remains in apposition to the globe
    Lid margin also pulls away from the globe
    Treatment
    Spontaneous resolution
    May require surgical correction
    63
Q.5. Causes of spastic entropion.
Ans.
  • Senile
  • Ocular surface disorder, e.g. dry eye
  • Enophthalmos
  • Loss of orbital fat
  • Tight bandage
  • Enucleation socket
Management of acute spastic entropion
  • Treatment of the underlying cause-break the irritation-entropion cycle.
  • Taping of the inturned eyelid to evert the margin, various suture techniques afford temporary relief for most patients.
  • Additional definitive surgical repair to correct the underlying involutional changes.
  • In selected cases, botulinum toxin injection can be used to paralyze the overriding preseptal orbicularis muscle.
Q.6. Role of botulinum toxin in management of entropion
Ans.
  • Works very well in correcting spastic entropion.
  • Also, in selected cases of involutional entropion with significant preseptal muscle override when the patient is not willing for surgery or is bedridden and not fit for surgery.
  • Dose and procedure: 2.5 units of botulinum toxin are injected at two or three places below the lower lid margin. Injected directly into the muscle by pinching it taking care not to go deep to avoid trauma to extraocular muscles (especially medially as it may cause inferior oblique muscle paresis and lead to diplopia).
  • Disadvantage: May take around 4–7 days for its effect to appear. Short-term, not a permanent solution.
Q.7. The material of choice for the spacer graft for entropion correction.
Ans.
  • Hard palate mucosal graft
  • Donor sclera
  • Buccal mucous membrane
  • Amniotic membrane.
References
  1. Richard R Tenzel. Orbit and Oculoplastics Textbook of Ophthalmology. 1993;4:3.1–3.12.
  1. Charles K Beyer, Machule, Gunter K. Atlas of Ophthalmalmic Surgery. Von Noorden. First Indian Edition. 1990;1:1.6–1.14.
  1. Albert and Jacobeic Principles and Practice of Ophthalmology. 2000;4:3491–2.
 
BLEPHAROPHIMOSIS, PTOSIS, EPICANTHUS INVERSUS SYNDROME
Amar Pujari, Aditi Dubey
 
INTRODUCTION
First described by Komoto in 1921, blepharophimosis-ptosis-epicanthus inversus syndrome (BPES) is a dominantly inherited disorder characterized by the presence of above features (i.e. blepharophimosis, ptosis, and epicanthus) at birth. The main findings of this disorder are eyelids that are abnormally narrow horizontally (blepharophimosis), a vertical fold of skin from the lower eyelid up either side of the nose (epicanthus inversus), and drooping of the upper eyelids (ptosis). It can be given as a short case in the examination.
 
HISTORY
 
Chief Complaints
The parents bring the child with anyone or a combination of the following complaints:
  • Drooping of eyelid,
  • Abnormal head posture, (chin up due to ptosis)
  • Small size of eyeball
  • Abnormal eyelid
  • Diminution of vision: Blurring of vision is related to refractive error, astigmatism.
  • Absence of eyeball
  • Bluish colored swelling (in case of crypto-phthalmos)
  • Epiphora (due to displaced tear ducts).64
 
Past History
A careful past history should be taken for any:
  • Perinatal and pregnancy history
  • Family history of congenital eyelid colobomas or other congenital anomalies, especially facial (e.g. cleft lip/palate)
  • History of other birth defects
  • Pediatric review of systems
  • Facial asymmetry
  • Hearing loss
  • Recurrent infection
  • Menstrual history (in case of late presentation).
 
Past Surgical History
The previous history of ocular surgery (attempts to correct any of the lid deformity or squint) may or may not be present.
 
EXAMINATION
 
Systemic Examination
Features frequently observed in both BPES type I and type II are a broad nasal bridge, low-set ears, and a short philtrum.
This condition is sometimes associated with ovarian failure although breast development is often normal. Secondary sexual characteristics are usually normal in both BPES type I and type II. In BPES type I, menarche is usually normal, followed by oligomenorrhea and secondary amenorrhea.
Other malformations that can be seen includes (also known as BPES Plus):
  • Contractures
  • Low nose bridge, micrognathia, microcephaly
  • Ear abnormalities: Incomplete ear development/Cupped ears, posteriorly rotated ears
  • Infertility in females, premature menopause, primary gonadal failure
  • Reduced muscle tone—only early in life
  • Mental defects, severe psychomotor retardation, growth retardation
  • Genitourinary malformations, Cryptorchidism, syndactyly.
 
Ocular Examination
Visual acuity: It is variably impaired depending on the severity of ptosis, astigmatism. Cycloplegic refraction is a must in all such cases since the significant refractive error may be seen in almost 1/3rd of these cases and if untreated can lead to amblyopia. Amblyopia can also occur due to stimulus deprivation (in a case of severe ptosis) but rare.
Head posture: To compensate for the ptosis, affected person assumes a characteristic posture with the head tilted backward, the brow furrowed, and the chin arched upward. Frontalis overaction may be there (eyebrows are increased in their vertical height and they are drawn up into a pronounced convex arch).
Eyeball: The eyeball may show microphthalmos, anophthalmos, cryptophthalmos. The palpebral fissure is reduced in both horizontal and vertical dimension. The normal horizontal fissure length in adults is 25–30 mm whereas in this syndrome it is usually 20–22 mm. Telecanthus is seen in the majority of patients. This refers to a lateral displacement of the inner canthi leading to a widening of the intercanthal distance. The interpupillary distance (IPD) is usually normal. The patient may have esotropia, divergent strabismus or nystagmus.
Eyelids: Eyelids are often covered by the smooth skin without eyelid folds and deficient amounts of skin in both eyelids may be found (Fig. 1). Frequently, the upper and lower lacrimal puncta are displaced laterally or duplication of puncta can be seen.
zoom view
Fig. 1: Blepharophimosis epicanthus inversus syndrome
65Blepharoptosis literally means a falling of the lids. The palpebral fissure is abnormally small in the vertical dimension. It is caused by the absence or impairment of the function of the levator palpebrae superioris muscle and is usually bilateral and symmetrical [Kindly see the examination part of ptosis in chapter ptosis for a detailed examination of ptosis].
Dysplastic eyelids: Eyelids are often covered by the smooth skin without eyelid folds and deficient amounts of skin in both eyelids may be found. The upper eyelid margin may show ‘S’ shaped curve. The lower lid margin usually has an abnormal concavity downwards, particularly laterally where an ectropion might occur. Trichiasis can also occur in BPES.
Epicanthus inversus: A small skin fold that arises from the lower lid and runs inwards and upwards characterizes it. Associated with this are an increased length of the medial canthal ligament and a lack of the normal depression seen at the internal canthus.
Conjunctiva: It is usually normal.
Cornea: It may show micro cornea occasionally. Corneal sensation should be checked.
Lens/Sclera/Iris/Fundus: Usually normal.
 
INVESTIGATIONS
Diagnosis of the disease is straightforward based on the clinical signs. Molecular genetic testing and specific laboratory studies are indicated in cases of associated syndromes. A pelvic ultrasound examination and measurement of bone mineral density are indicated if at ovarian insufficiency is suspected.
 
CLASSIFICATION
Two types of BPES have been described:
 
BPES Type I
It is the more common type, in which males transmit the syndrome only and affected females are infertile. It is associated with an early loss of ovarian function (primary ovarian insufficiency) in women, which causes their menstrual periods to become less frequent and eventually stop before age 40. Primary ovarian insufficiency can lead to difficulty conceiving a child (subfertility) or a complete inability to conceive (infertility).
 
BPES Type II
Both affected females and males transmit this variant. It is not associated with female infertility.
Both types are inherited as an autosomal dominant trait. There is complete penetrance (100%) in type I and slightly reduced (96.5%) penetrance in type II. Both types I and II include the eyelid malformations and other facial features.13
 
DIFFERENTIAL DIAGNOSIS
Differential diagnosis includes those conditions in which ptosis or blepharophimosis are a major feature (see Table 1 for features of these syndromes)
  • Congenital simple ptosis
  • Ptosis with external ophthalmoplegia
  • Noonan syndrome
  • Marden-Walker syndrome
  • Schwartz-Jampel syndrome
  • Dubowitz syndrome
  • Smith-Lemli-Opitz syndrome.
The characteristic combination of signs usually clinches the diagnosis.
 
MANAGEMENT
Management requires the input of specialists including a clinical geneticist, pediatric ophthalmologist, oculoplastic surgeon, (pediatric or adult) endocrinologist, reproductive endocrinologist, and gynecologist.
Management of BPES is primarily surgical if indicated. However, any refractive error must be corrected to avoid amblyopia. The indication of surgery is moderate to severe ptosis, amblyopia, trichiasis which may cause the corneal lesion, cosmesis, strabismus. Care should be given to treat associated amblyopia.
The timing of eyelid surgery is controversial; it involves weighing the balance of early surgery to prevent deprivation amblyopia and late surgery to allow for more reliable ptosis measurements, the latter of which provides a better surgical outcome. Furthermore, ptosis surgery is hampered by the dysplastic structure of the eyelids. The surgical management is traditionally performed in two stages and involves a medial canthoplasty for correction of the blepharophimosis, epicanthus inversus, and telecanthus at about the age of 4–5 years and correction of the ptosis about 9–12 months later.66
Table 1   Syndromes associated with BPES
Association
Inheritance
Features
Hereditary congenital ptosis 1 (PTOS1)
AD
Ptosis
Hereditary congenital ptosis 2 (PTOS2)
XL
Ptosis
Ohdo blepharophimosis syndrome
AD
Blepharophimosis, ptosis, mental retardation, congenital heart defects, teeth abnormality (hypoplastic teeth)
3MC syndrome 1 (Michels syndrome)
AD
Blepharophimosis, ptosis, epicanthus inversus, corneal abnormality, Cleft lip/palate, skeletal abnormalities
Ptosis with external ophthalmoplegia
AR
Ptosis, ophthalmoplegia, miosis. Decreased accommodation
Noonan syndrome
AD
Ptosis, short stature, heart defects, clotting abnormalities
Marden-Walker syndrome
AR
Ptosis, blepharophimosis, growth retardation, mental retardation
Schwartz-Jampel syndrome
AR
Intermittent ptosis, blepharophimosis, telecanthus, cataract, short stature, skeletal anomalies, muscle hypertrophy
Dubowitz syndrome
AR
Ptosis, blepharophimosis, lateral telecanthus, short stature, intellectual disability, immunodeficiencies
Smith-Lemli-Opitz syndrome
AR
Ptosis, epicanthus, cataract, growth retardation, intellectual disability, genitourinary, cardiac, gastrointestinal anomalies
KANSL1-related intellectual disability syndrome
AR
Developmental delay, intellectual disability, long face, high forehead, ptosis, blepharophimosis, large low-set ears, bulbous nasal tip, pear-shaped nose, cardiac septal defects, seizures, cryptorchidism
Abbreviations: AD, autosomal dominant; AR, autosomal recessive; XL, X-linked; BPES, blepharophimosis-ptosis-epicanthus inversus syndrome
Early surgery may be necessary for amblyopia.
Epicanthus fold and telecanthus: The various procedures for correction of epicanthus fold and telecanthus includes: double Z or Y-Z plasties (Fig. 2), Transnasal wiring of the medial canthal tendons. If the epicanthal folds are small, a Y-V canthoplasty is traditionally used; if the epicanthal folds are severe, a double Z-plasty is used. An alternate technique for medial canthoplasty has been described recently using the skin redraping method, which has a simple flap design, less scarring, and the effective repair of epicanthus inversus and telecanthus.3
zoom view
Fig. 2: Blepharophimosis, ptosis, epicanthus inversus syndrome (Figure 1 patient) after bilateral Y-V plasty and bilateral sling surgery
67Ptosis: Generally, it is corrected with brow suspension procedure. Super-maximal resection and frontalis suspension is the preferred method as it leads to a good cosmetic outcome as well as to an improved muscle function.
Although traditional management of blepharophimosis syndrome includes medial canthoplasty between the ages of 3 and 5 years, followed by ptosis correction about 6 months later, patients with severe ptosis may need early surgery to prevent amblyopia. Traditional multiple surgeries may delay the amblyopia management and influence the visual outcome. Thus, many surgeons suggest correction of ptosis first, even at a very early age, to prevent amblyopia. Soft-tissue medial canthal and lateral canthal surgery can wait until the face is grown.3 Treatment of associated abnormalities: It includes management of ovarian failure, hormone replacement therapy, and embryo cryopreservation. Management of amblyopia (i.e. with/without spectacle wear/contact lens must be continued after surgical intervention to obtain optimal results.
Patient education: Genetic consultation is highly recommended, especially for patients with associated syndromes.
 
VIVA QUESTIONS
Q.1. What is the sequence of surgeries for ptosis and epicanthus in BPES?
Ans. See management part.
Q.2. When should the BPES be repaired?
Ans. See management part.
Q.3. What are the syndromes associated with ptosis?
Ans. See Table 1.
Q.4. Classify BPES.
Ans. Already given in classification.
Q.5. Genetics of BPES.
Ans. Both types are caused by mutations in the FOXL2 gene. The FOXL2 gene provides instructions for making a protein that is active in the eyelids and ovaries. The FOXL2 protein is likely involved in the development of muscles in the eyelids.
References
  1. Alao MJ, Lalèyè A, Lalya F, Hans Ch, Abramovicz M, Morice-Picard F, Arveiler B, Lacombe D, Rooryck C. Blepharophimosis, ptosis, epicanthus inversus syndrome with translocation and deletion at chromosome 3q23 in a black African female. Eur J Med Genet. 2012;55:630–4.
  1. Batista F, Vaiman D, Dausset J, Fellous M, Veitia RA. Potential targets of FOXL2, a transcription factor involved in craniofacial and follicular development, identified by transcriptomics. Proc Natl Acad Sci USA. 2007;104:3330–5.
  1. Beckingsale PS, Sullivan TJ, Wong VA, Oley C. Blepharophimosis: a recommendation for early surgery in patients with severe ptosis. Clin Experiment Ophthalmol. 2003;31:138–42.
 
SEBACEOUS GLAND CARCINOMA
Amar Pujari, Sapna Raghuwanshi
 
INTRODUCTION
Sebaceous gland carcinoma (SGC) is third most common eyelid malignancy of the eyelids. Most sebaceous carcinomas arise from the meibomian glands, Zeiss gland, and Moll gland of the eyelid usually occurs between fifth and ninth decades of life, women's are more affected than man.13 It mostly involves the upper lid (Fig. 1).
In exams, it can be given as a long case or short case.68
zoom view
Fig. 1: Sebaceous gland carcinoma
 
HISTORY
Kindly see the section of lid tumors also.
 
Chief Complaint
Patient may present with the following complaint:
  • Slowly enlarging, firm, and painless mass or nodule at the lid margins (most common presentation)
  • Yellowish nodule at eyelid margin or caruncle
  • As chronic blepharoconjunctivitis (irritation, redness or foreign body sensation)
  • Recurrent chalazion
  • Chronic blepharitis with loss of cilia
  • Skin or conjunctival ulcer
  • Advanced cases may present as eyelid mass with a destruction of marginal cilia and lid architecture
  • Proptosis due to local invasion (anterior orbital mass or lacrimal gland tumor).
 
Past History
A careful history about the onset, progression, association with pain must be recorded (similar to that described under long case section for Lid tumors)
The risk factors of sebaceous cell carcinoma must be ruled out in history such as.
  • Recurrent chalazion
  • Previous radiotherapy
  • Chronic blepharoconjunctivitis
  • Immunosuppression
  • Asian race
  • Prolonged use of thiazide diuretics
  • Older age (5th to 9th decade, average age 60–69 years)
  • Female sex (55–57% cases are females).
 
EXAMINATION
 
Ocular Examination
Eyelids: Eyelid may have a nodule with following features:1
  • Slowly enlarging, firm, and painless mass affecting the tarsal plate or the eyelid margin(Since the meibomian gland is buried deep in the tarsus, initially the tumor will form a firm mass, and may be misdiagnosed as a chalazion. As the tumor invades more superficially, a yellowish cast may be visualized through the skin).
  • Location
    • Upper lid: 60–70%, most common site, involved two to three times more frequently than the lower lid (Fig. 1). This is due to the presence of a greater number of meibomian glands in the upper lid (50 glands in upper eyelids, 25 in lower approximately).
    • Caruncle: 5–11%
    • Eyebrow: 2%
    • Simultaneous involvement of both lids: 6–8%
  • Lesions may also exhibit varying degrees of yellow coloration/yellowish cast due to the presence of lipid within the mass.
  • Lesions originating from the Zeis glands appear as small, yellowish nodules located at the eyelid margin anterior to the gray line. At times it may form a papilloma or cutaneous horn.
  • Tumors arising from sebaceous glands of the caruncle usually appear as a subconjunctival, multilobulated, yellow mass.
  • Eyelids are diffusely thickened.
  • Skin appears indurated, usually skin is movable on the mass until late stages
  • Small telangiectasia's over the mass
  • Loss of eyelashes (Disruption of eyelid architecture and lash loss can occur as the tumor destroys lash bulbs)
  • Rarely can present as anterior orbital mass or lacrimal gland tumor.69
Enlarged lymph nodes: Submandibular, submental, preauricular and cervical.
Conjunctiva: Conjunctival inflammation, superior limbic keratoconjunctivitis can be seen.
Cornea: Superficial keratitis may be present if tumor cells invade corneal epithelium.
Other findings are usually within normal limits.
 
DIFFERENTIAL DIAGNOSIS
The following disease must be differentiated from the sebaceous cell carcinoma
  • Blepharoconjunctivitis
  • Blepharitis
  • Chalazion
  • Superior limbic keratoconjunctivitis
  • Basal cell carcinoma
  • Squamous cell carcinoma.
The characteristic features of sebaceous cell carcinoma that help in differentiating from these lesions are yellowish discoloration and telangiectatic blood vessels on the surface. At times, it is often difficult to differentiate, especially in early cases and excisional biopsy is the best way to arrive at a conclusion.
 
INVESTIGATION
A case of SGC needs following investigations.
Routine tests like: Complete blood count, renal and liver function tests.
Metastatic work-up: X-ray chest/USG abdomen/ CECT: Brain and orbit.
FNAC: lymph node (lipid stains).
Conjunctival impression biopsy (for pagetoid spread).
Map biopsy: Sebaceous gland carcinoma must be confirmed by a full-thickness wedge biopsy of the affected eyelid. Because of multicentric spread, multiple biopsy specimens should be taken from the adjacent bulbar and palpebral conjunctiva and the other ipsilateral eyelid to form a map of the extent of tumor spread across an ocular surface. After histologic confirmation of sebaceous carcinoma, the surgeon must consider the extent of possible pagetoid involvement of the bulbar conjunctiva. Map conjunctival biopsies were taken in all four quadrants.13
 
MANAGEMENT
  • Nodular sebaceous carcinoma without pagetoid involvement: Lesion can be removed by following techniques:
    • Full thickness eyelid resection with frozen section control of margins
    • Mohs’ micrographic technique: Excision of the visible tumor, with a 5-mm margin of clinically normal tissue on either side.
  • Nodular sebaceous carcinoma of one eyelid with evidence of pagetoid spread:
    • Excision of the nodular lesion with conjunctive and superficial keratectomy/cryotherapy of the lesion.
    • Mitomycin C (MMC) can be tried to control conjunctival tumor. MMC 0.4% four times per day for a week and then were medication free for 1 week; this cycle was repeated until resolution of malignancy.
    • If bulbar conjunctiva is involved extensively by tumor and reconstruction is not possible: Exenteration is recommended
    • If both upper and lower eyelids are involved by tumor without an involvement of the conjunctiva: Remove both eyelids and reconstruct the defect.
    • Concomitant involvement of eyelids and conjunctiva: It requires exenteration.
  • Orbital disease with no metastasis: Orbital exenteration
  • Ocular disease with lymphatic metastasis: Mass resection/exenteration, radical neck dissection, and postoperative radiation
  • Systemic chemotherapy may is required in the management of metastatic disease; however, there is little in the literature on the efficacy of postsurgical chemotherapy for metastatic disease.
 
Prognosis
  • Poor compared to BCC.
  • Five year mortality
    • Early disease: 15% (6–30%)13
    • Metastatic disease: 50–67%13
  • Five year recurrence rate: 9–36%1370
 
VIVA QUESTIONS
Q.1. Histopathology.
Ans.Dysplasia and anaplasia of the sebaceous lobules in the meibomian glands are seen in SGC, associated with the destruction of tarsal and adnexal tissues. As the neoplastic nodule enlarges, it may erupt toward the eyelid skin to initiate the intraepidermal growth phase, wherein the sebaceous cells spread diffusely throughout the epidermis. This ‘pagetoid’ epidermal invasion is a distinctive feature of sebaceous carcinomas.
Specific characteristics on histopathology
  • Stains with oil red O and Sudan IV
  • Cytoplasm is frothy and vacuolated
  • Cells are larger vesicular and have prominent nucleoli.
Histopathologic subtypes: Lobular (lobules of sebaceous architecture), comedocarcinoma (characterized by a central necrotic core), papillary (papillary projections and areas of sebaceous differentiation), and mixed (features of any subtype). It can also be described as well differentiated, moderately differentiated, and poorly differentiated.
Q.2. Metastasis.
Ans.An invasive, potentially lethal tumor, SGC may cause an extensive local destruction of eyelid tissues. It carries a risk of metastasis to preauricular and submandibular lymph nodes or may spread hematogenous to distant sites. It may invade locally into the globe, the orbit, the sinuses, or the brain. The frequencies of the spreads are:13
  • Direct extension: Orbit, lacrimal glands—6–17%
  • Lymphatic: Regional nodes—17–28%
  • Hematogenous: Rare, <1%, lungs, liver, skull, and brain
Pagetoid spread: Intraepithelial or intraepidermal spread of malignant cells similar to that observed in Paget's disease of the nipple or extramammary Paget's disease. Spread along skin or conjunctiva, producing individual cell clusters are characteristic. It is seen in almost 47% of the cases.
Risk factors for subclinical spread include:
  • Duration of symptoms >6 months
  • Vascular and lymphatic infiltration
  • Orbital extension
  • Poor tumor differentiation
  • Multicentric origin intraepithelial carcinomatous changes of the conjunctiva, cornea, or skin
  • Location in the upper eyelid.
Q.3. Prognostic factors.
Ans.
  • Site:
    • Lower lid tumor (better) > Upper lid tumor >Both upper and lower lids (worst)
    • Conjunctival tumor: Metastatic disease
  • Tumor size: Increased size, worse prognosis >10 mm
  • Delay in diagnosis: Delay >6 months, bad prognosis
  • Histology:
    • Vascular invasion
    • Lymphatic invasion
    • Highly infiltrative pattern
    • Poor differentiation
    • Pagetoid spread
    • Multicentric origin
    • Orbital invasion.
Q.4. What is Muir-Torre syndrome?
Ans.
  • It is an autosomal dominant condition in which there are sebaceous (oil gland) skin tumors in association with internal cancer.
  • The most common organ involved is the gastrointestinal tract, with almost one-half of the patients having colorectal cancer. The second most common site is cancer of the genitourinary tract.
  • Following skin lesions are associated with this syndrome
    • Sebaceous adenomas
    • Sebaceous epitheliomas
    • Sebaceous carcinoma
    • Keratoacanthoma
    • Squamous cell carcinoma
    • Multiple follicular cysts
  • It is now thought to be a hereditary nonpolyposis colorectal cancer syndrome 71due to mutations in the DNA mismatch repair genes MSH2 or MLH1.
Q.5. Sebaceous carcinoma in systemic disease.
Ans.Sebaceous gland carcinoma is seen in association with following:
  • Familial retinoblastoma after radio-therapy
  • Immunosuppression in HIV disease
  • Muir–Torre syndrome.
References
  1. Wali UK, Al-Mujaini A. Sebaceous gland carcinoma of the eyelid. Oman J Ophthalmol. 2010;3(3):117–21.
  1. Mulay K, Aggarwal E, White VA. Periocular sebaceous gland carcinoma: A comprehensive review. Saudi J Ophthalmol. 2013;27(3):159–65.
  1. Albert DM, Miller JW, Azar DT. Albert & Jakobiec's principles and practice of ophthalmology; 2008.
 
PYOGENIC GRANULOMA
Sapna Raghuwanshi, Bijnya Birajita Panda
 
INTRODUCTION1,2
Pyogenic granuloma is an inflammatory vascular response of the tissue that usually occurs after a previous insult, typically either inflammatory or trauma. It is the most common acquired vascular lesion to involve the eyelids. It also involves the conjunctiva. The name is a misnomer because this lesion is neither pyogenic nor granulomatous.
In exams, it can be given as a short case.
 
HISTORY
 
Chief Complaints
The patient may present with following:
  • Rapidly growing mass over eyelids and conjunctiva
  • Lesion readily bleeds with minor contact
  • Pain (associated with superficial ulceration).
 
Past History
Following points must be noted in history
  • Minor trauma
  • Surgery (Limbal surgery for pterygium, squamous cell carcinoma, phthisis, squint surgery)
  • Chalazion
  • Microbial infection
  • Pterygium
  • Chemical burns.
 
EXAMINATION
Eyelids: Raised, red, smooth surfaced lesions with a narrow base (Fig. 1).
Conjunctiva: If conjunctiva is involved conjunctival inflammation may be present (Fig. 1).
Other findings are usually within normal limits.
 
DIFFERENTIAL DIAGNOSIS
  • Kaposi's sarcoma—slow growing, in immunocompromised patient in contrast to fast growing pyogenic granuloma and associated risk factors
    zoom view
    Fig. 1: Pyogenic granuloma
  • 72Intravascular papillary endothelial hyperplasia
  • Squamous papilloma
  • Conjunctival lymphoma
  • Ocular lymphangiectasia.
 
HISTOPATHOLOGY
Lesion consists of a mass of granulation tissue, prominent capillaries, and acute and chronic inflammatory cells.
 
MANAGEMENT
  • Surgical excision
  • Topical or intralesional corticosteroids: It is better to give a short course of steroid therapy before proceeding with surgery.
References
  1. Brad Bowling. Kanski's Clinical ophthalmology: A systematic approach, 8th edn. Edinburgh: Elsevier,  2015.
  1. Albert DM, Miller JW, Azar DT. Albert & Jakobiec's principles and practice of ophthalmology; 2008.
 
LAGOPHTHALMOS
Varsha Varshney, Ritu Nagpal
 
INTRODUCTION
Lagophthalmos is a condition in which the eyelids do not close to cover the eye completely. The term lagophthalmos actually comes from the Greek word for hare (lagoos) and derives from a myth that hares sleep with their eyes open.1,2 In exams, it can be given as a short or spot case. A normal, healthy eye is covered by a film of tears that protects the surface and washes away dust and particles. Dry eyes that result from lagophthalmos are not only uncomfortable but are also subject to injury or infection from foreign objects landing in and abrading the eye surface. Left untreated, lagophthalmos can lead to permanent loss of vision.
 
HISTORY
 
Chief Complaints
Lagophthalmos patients commonly complain of inability to close lids completely (Figs 1 and 2). Associated features are:
  • Foreign body sensation
  • Increased tearing
  • Photophobia
  • Pain may be worse in the morning due to increased corneal exposure and dryness during sleep
  • Blurry vision, which results from unstable tear film
    zoom view
    Fig. 1: Normal lid opening
    zoom view
    Fig. 2: Lagophthalmos on lid closure in a case (Figure 1)
  • 73In cases of advanced keratopathy and corneal ulceration, the symptoms and presentation may be severe.
 
History of Past Illness
  • The recent history of trauma or surgery involving the head, face or eyelids should be documented with special attention to fractures to the skull base (a petrous portion of the temporal bone) or mandible that could affect facial nerve.
  • Past infections should be reviewed, with particular attention to any history of herpes zoster infection.
  • It is also important to document any past symptoms suggestive of thyroid disease or obstructive sleep apnea (Floppy eyelid syndrome).
 
History of Systemic Illness
History of systemic diseases like diabetes (diabetic neuropathy), any neurological disorder (polio, Guillain-Barré syndrome, leprosy), cerebrovascular accidents.
 
EXAMINATION
 
Systemic Examination
Complete neurological examination should be performed.
 
Ocular Examination
Visual acuity: Usually, normal. The blurring of vision may be present due to tearing or dryness.
Eyeball: Usually, normal.
Lids: Ask the patient to look down and gently close both eyes. Lagophthalmos is present when space remains between the upper and lower eyelid margins in extreme downgaze. Document the degree of lagophthalmos by measuring this space, in millimeters, with a ruler. In addition, following points must be recorded carefully.
  • Record the blink rate as well as the completeness of the blink.
  • Carefully test ocular motility and the strength of the orbicularis oculi muscle. The latter can be assessed by evaluating the force generated on attempted eyelid closure.
  • Bell's phenomenon.
  • Scar marks of previous trauma or surgery or infections may be present.
Conjunctiva: Diffuse or ciliary congestion may be there in presence of dry eye and exposure keratopathy.
Cornea: Following points must be recorded:
  • Corneal sensitivity by applying a wisp soft cotton to the unanesthetized cornea and comparing the blink reaction with that of the fellow eye.
  • Fluorescein staining with cobalt blue filter to find out the presence of punctate epithelial erosions or abrasions. Pay particular attention to the inferior cornea where lid excursion ends.
  • Tear breakup time.
  • Any epithelial defects or corneal ulcers should be carefully documented.
  • Schirmer test: The Schirmer test is used to assess tearing function. The degree of tearing can be compared between the paralyzed and normal sides.
Pupils: Pupillary reflexes are usually normal unless other cranial nerves are involved.
Anterior segment: Normal chamber depth and contents.
Posterior segment: Slit-lamp biomicroscopic examination using a 90D/78D lens and indirect ophthalmoscopy—usually normal.
 
INVESTIGATIONS
 
Blood Investigations
To rule out any systemic disease or infection following tests can be performed:
  • CBC, blood sugar
  • Thyroid function test
  • HIV ELISA/western blot
  • VDRL/RPR
The tests advised must be based on the history and clinical findings.
 
Radiological Investigations
Preferably gadolinium-enhanced MRI: To rule out any neurological causes such as fracture damaging the nerve, mass compressing the nerve, ischemic areas involving facial nerve origin (geniculate ganglion).74
 
Conduction Testing and Electromyography
The tests are most useful when performed 3–10 days after the onset of paralysis. Comparison to the contralateral side helps to demonstrate the extent of nerve injury and has prognostic implications. Nerve conduction responses are abnormal if a difference of 50% in amplitude between the paralyzed and normal side is detected; a difference of 90% between the 2 sides suggests a poorer prognosis.
 
Electroneurography
It is a physiologic test that objectively measure the difference between potentials generated by the facial musculature on both sides of the face in response to a supramaximal electrical stimulation of the facial nerve.
 
Electrodiagnostic Testing
Measures the facial nerve degeneration indirectly. If a patient does not reach 90% degeneration within the first 3 weeks of the onset of paralysis, some studies suggest that the prognosis is excellent, with over 80–100% of the patients recovering with excellent function.
 
Brainstem Auditory Evoked Response (BAER)
It may be obtained in patients with peripheral facial nerve lesions and other neurologic involvement. This test measures the transmission of response through the brainstem and is effective in detecting, notably, retrocochlear lesions.
 
Blepharokymographic Analysis
A high-speed eyelid motion-analysis system, has been used to evaluate movement of the eyelids. The computer-based analysis may prove helpful in diagnosing Bell palsy, predicting prognosis, and evaluating response to therapeutic measures such as placement of a gold weight in the affected upper eyelid (used in cases in which spontaneous recovery has been limited).
All these investigations must be conducted in consultation with a neurologist.
 
DIFFERENTIAL DIAGNOSIS
Lagophthalmos can be due to a variety of causes, careful history and neuroimaging often help in arriving at a diagnosis. Following points must be kept in mind:
  • Most common cause is Bell's palsy.
  • If another cranial nerve, motor, or sensory symptoms are present, then other neurologic diseases should be considered (e.g. stroke, Guillain-Barré syndrome, basilar meningitis, cerebellar pontine angle tumor).
  • Symptoms associated with seventh nerve neoplasm include slowly progressive paralysis, facial hyperkinesis, severe pain, recurrent palsy, and other cranial nerve involvement.
  • Cerebellopontine tumors may affect the seventh, eighth, and fifth cranial nerves simultaneously.
  • Patients with a progressive paralysis of the facial nerve lasting longer than 3 weeks should be evaluated for neoplasm.
  • Recurrent ipsilateral facial paralysis must raise the suspicion of a tumor of the facial nerve or parotid gland. Tumors in the temporal bone, such as facial nerve neuromas, meningiomas, hemangiomas, and malignant primary and metastatic lesions, should be considered as well.
  • If a patient reports the sudden onset of hearing loss and severe pain with the onset of facial paralysis, Ramsay Hunt syndrome must be considered. Typically, these patients will also have an erythematous vesicular rash involving the ear canal, auricle, and/or oropharynx.
Bilateral cases: Bilateral simultaneous Bell palsy is a rare (<1% of that of unilateral facial nerve palsy). Examples include Guillain-Barré syndrome, sarcoidosis, Lyme disease, meningitis (neoplastic or infectious), or bilateral neurofibromas (in patients with neurofibromatosis type 2).
 
TREATMENT
 
Medical Treatment and Supportive Care for Corneal Exposure
Nonpreserved artificial tears should be administered frequently (at least four times per day) in order to supplement the patient's tear film. Ointments can be applied to the cornea once at bedtime or throughout the day in cases of severe corneal exposure. Prophylactic antibiotics (preferably nonepitheliotoxic such 75as chloramphenicol 0.3%) can be added to the regimen. In addition, following measures can be used:
  • Moisture goggles also may be used.
  • Punctal plugs may be helpful if dryness of the cornea is a persistent problem.
  • Infectious corneal ulcers should be treated with appropriate antibiotic therapy.
  • Patching the eye in the night time with simple micropore or a Frost suture for temporary protection of the cornea can also be helpful.
  • Botulinum toxin can be injected transcutaneously or subconjunctivally at the upper border of the tarsus to paralyze the levator muscle to produce complete ptosis and to protect the cornea.
 
Tarsorrhaphy
A temporary tarsorrhaphy performed if recovery of the eyelid closure is expected within a few weeks. In most cases, the cornea can be protected adequately by suturing the lateral one-third of the eyelids together. Ideally, a small opening remains so that the patient can retain useful vision, the health of the cornea may be assessed and lubrication or antibiotic therapy can be applied to the eye.
A permanent tarsorrhaphy performed if a protracted clinical course is expected. If the patient regains useful function of the orbicularis oculi muscle, the adhesions can be lysed. The limitation of tarsorrhaphy is poor cosmetic appearance.
 
Gold Weight Implantation
Gold weights can be implanted into the upper eyelid to treat paralytic lagophthalmos. It enhances eyelid closure in a gravity-dependent fashion. Gold is considered an ideal substance because it is inert and it is not visible through the thin skin of the eyelid. Gold weights range from 0.6 to 1.6 g (in 0.2-g increments). The appropriate weight is chosen preoperatively by taping weights of varying sizes onto the external lid above the tarsus and observing the closing and opening of the lids. Properly chosen, the ideal weight will allow full closing and the opening of the lids while avoiding ptosis in primary gaze. Gold weight implantation is usually well-tolerated. However, astigmatic shift, as well as migration and/or extrusion of the gold weight, are its limitations. In cases of allergy to gold, platinum may be used.
 
Upper Eyelid Retraction and Levator Recession
The recession of the upper eyelid retractors (levator and Müller's muscles) is a useful procedure in patients with lagophthalmos related to upper eyelid retraction from thyroid ophthalmopathy. Also, a combination of full-thickness skin grafts, advancement flaps, tarsal-sharing procedures and release of scar bands can be performed on patients with lagophthalmos from cicatricial or postsurgical lid shortening.
 
Lower Eyelid Tightening and Elevation
Laxity of the lower eyelid may occur in conditions such as facial nerve palsy and floppy eyelid syndrome. A tightening procedure such as a lateral tarsal strip will improve apposition of the lower eyelid to the globe and decrease tearing. This is also helpful in cases where upper eyelid restructuring procedures fail.
 
Other Surgical Procedures
In cases of severe lagophthalmos various other procedures have been described such as elevation of the midface using a variety of materials such as autogenous fascia slings, temporalis muscle transposition/transfer, nerve grafts and anastomoses, palpebral springs, soft tissue repositioning and suborbicularis oculi fat lifts.
 
VIVA QUESTIONS
Q.1. What are the ocular symptoms and signs of Bell's palsy?
Ans.Bell palsy is the most common cause of unilateral facial paralysis. Ocular manifestations have been described in Table 1. Two-thirds of patients complain about epiphora which is due to punctal eversion and the reduced function of the orbicularis oculi in transporting the tears (fewer tears arrive at the lacrimal sac, and overflow occurs).76
Table 1   Ocular manifestations of Bell's palsy
Early
Late
  • Lagophthalmos
  • Paralytic ectropion of the lower lid
  • Tear overflow
  • Brow ptosis
  • Upper eyelid retraction
  • Dry eye—poor tear distribution
  • Corneal exposure, erosion, infection, and ulceration (rare)
  • Narrow palpebral fissure- generalized mass contracture of the facial muscles(after several months)
  • Aberrant regeneration of the facial nerve with motor synkinesis
  • Reversed jaw winking- twitching of the corner of the mouth or dimpling of the chin occurring simultaneously with each blink
  • Crocodile tears—tearing with chewing
Q.2. Discuss etiology of lagophthalmos.
Ans.Lagophthalmos can occur due to a pathology in the facial nerve or in the lid. The different causes are summarized in Table 2.
Table 2   Causes of lagophthalmos
Pathology
Etiology
Factors/mechanism
Facial nerve
Trauma
Fractures to the skull base (petrous portion of the temporal bone) or mandible
Neurosurgical procedures
Bell's palsy
Acute viral infection or reactivation of herpes simplex virus
Tumors
Acoustic neuromas in the cerebellopontine angle
Metastatic lesions
Cerebrovascular accidents
Blockage of anterior inferior cerebellar artery
Infectious, immune-mediated causes
Lyme disease, chickenpox, mumps, polio, Guillain-Barré syndrome, leprosy, diphtheria and botulism
Mobius’ syndrome
Characterized by cranial nerve palsies (sixth and seventh cranial nerve), motility disturbances, limb anomalies and orofacial defects
Eyelids
Cicatrices
Chemical or thermal burns, ocular cicatricial pemphigoid, Stevens-Johnson syndrome, mechanical trauma
Eyelid surgery
Excessive removal of eyelid skin or muscle; blepharoplasty, tumor excision Overcorrection in ptosis repair
Proptosis
Exophthalmos of one or both globes may inhibit eyelid closure
Enophthalmos
Posterior displacement of the eye may affect eyelid apposition and closure
Causes include orbital blowout fractures; orbital fat atrophy (trauma, infection, inflammation, aging, scleroderma, HIV-AIDS); phthisical eye; scirrhous carcinomas
Floppy eyelid syndrome
Severe laxity and flexibility of the superior and inferior tarsal plates
Q.3. Relevant anatomy of facial nerve and eyelid.
Ans.Facial nerve: The facial nerve (seventh cranial nerve) innervates both the frontalis muscle, which raises the eyebrow and the orbicularis oculi muscle, which closes the eyelids. In addition, the 7th nerve innervates the muscles of the facial expression such zygomaticus muscles, which elevate the cheeks as well as the corrugator supercilii and procerus muscles, which depress the eyebrow.
Eyelids: The upper and lower eyelids contain seven structural layers. Beginning anteriorly, these comprise (1) skin and subcutaneous tissue, (2) orbicularis oculi muscle, (3) orbital septum, (4) orbital fat, (5) muscles of retraction, (6) tarsus and (7) conjunctiva. Damage to or degeneration of any of these tissues may inhibit good eyelid closure.77
Q.4. Discuss treatment options for Bell's palsy?
Ans.Treatment options for Bell's palsy includes following:
Pharmacologic therapy: The most widely accepted treatment for Bell palsy is corticosteroid therapy. However, the use of steroids is still controversial because most patients recover without treatment. The recommended dose of prednisone for the treatment of Bell palsy is 1 mg/kg or 60 mg/day for 6 days, followed by a taper, for a total of 10 days.
Antiviral agents: Such as Acyclovir (Zovirax) and Valacyclovir (Valtrex) have shown limited benefit.
Surgical options: Surgical options for Bell palsy include the following: Facial nerve decompression, subocularis oculi fat (SOOF) lift, Implantable devices (e.g. gold weights) placed into the eyelid, tarsorrhaphy, transposition of the temporalis muscle, facial nerve grafting, direct brow lift.
References
  1. Pereira MV, Glória AL. Lagophthalmos. Semin Ophthalmol. 2010;25(3):72–8.
  1. Vásquez LM, Medel R. Lagophthalmos after facial palsy: current therapeutic options. Ophthalmic Res. 2014;52(4):165–9.
 
DERMOID CYST
Shipra Singhi, Deepali Singhal
 
INTRODUCTION
A dermoid cyst (epidermal dermoid cyst) is an epithelial-lined structure with dermal appendages in its wall and keratin and hair in its lumen. It can be found in any subcutaneous location but more than 80% are located in the region of the head, with the majority in the eyelid and orbital area, usually superotemporally near the zygomaticofrontal suture.
It can be given as a short case or spot case in exam.
 
HISTORY
 
Chief Complaint
The presenting feature depends upon the location of dermoid. It can present in following manner:
  • Painless fullness of upper eyelid or a mass lesion, most commonly at lateral orbital rim can be the presenting feature in an Anteriorly located dermoid.
  • A posteriorly located dermoid can present with Painless, progressive proptosis, and diplopia.
  • Rarely there can be associated ptosis and limitation of the movement.
  • Diminution of vision: Blurring of vision is related to size and nerve compression and the presence of complications. It is usually progressive and painless.
  • Sometimes patient may present with Symptoms consist of the rapid onset of unilateral pain, redness and watery discharge due to a ruptured cyst.
  • Intermittent increase in size during chewing indicated extension to temporalis muscle of deep dermoid. However this mode of presentation is very rare.
In adults, dermoids may become symptomatic for the first time and grow considerably over a year. Based on this fact, some conclude that these lesions may be dormant for many years or have intermittent growth.
 
History of Present Illness
The onset, progression, association with pain and any preceding events such as trauma must be noted. Dermoid cyst usually has a insidious onset, painless and progresses slowly over a period of years. Rapid onset of unilateral pain, redness and watery discharge suggests a ruptured cyst.78
 
Past History
A careful past history should be taken of any mass (tumor), nerve paresis, infection, trauma and ocular inflammatory diseases.
 
Past Surgical History
Previous history of any intraocular surgery should be enquired.
 
EXAMINATION
Ocular examination: Ocular examination should include following:
Visual acuity: Visual acuity is variably impaired depending on the site of involvement, size and compression of nerve and the presence of complications.
Eyeball
  • Proptosis is usually nonaxial. Extension into intracranial fossae is possible if the frontal or sphenoid bones are involved. Temporal fossa involvement is rare but reported; this may result in intermittent proptosis associated with chewing, as positional changes of the temporalis muscle during chewing transmit pressure to the lesion and, hence, to the orbit.
  • Mass: A mass lesion due to dermoid cysts have following characteristics:
    • Firm in consistency
    • Margins are smooth
    • Non-tender
    • Mobile preseptal masses without fixity to skin or muscle
    • Superotemporal quadrant is the most common site, less commonly, the supero-nasal quadrant is affected.
    • Many of them have variable periosteal attachment near the underlying frontozygomatic or frontoethmoidal sutures.
    • Occasionally the dermoid will pass into or through defects in the neighboring bone and may communicate intracranially.
    • A dermoid cyst can rupture spontaneously or with trauma, inciting an intense inflammatory response in the orbital soft tissues. This response may be limited to injection of the conjunctiva or may be severe and mimic orbital cellulitis.
    • Occasionally, subconjunctival droplets of fat are seen. In some cases, a secondary fistula between the cyst and the skin may allow the contents of the cyst to drain intermittently.
  • Rarely the dermoid is incompletely separated from the skin surface and presents as a chronically inflamed and discharging sinus.
  • Dermoid cyst can also be associated with motility deficits and diplopia when the extent is large.
Eyelids: When dermoid is located anteriorly it can lead to ptosis.
Anterior segment: Generally normal. Occasionally, subconjunctival droplets of fat are seen in case of rupture of cyst.
Tonometry: IOP is usually unaffected.
Fundus: There may be compressive neuropathy or choroidal folds especially in cases of posterior dermoid.
 
DIFFERENTIAL DIAGNOSIS
Differential diagnosis depends upon the location of the dermoid:
  • Lateral anterior dermoid
    • Lacrimal gland mass
    • Lipodermoid
    • Teratoma
    • Plexiform neurofibroma
  • Medial anterior dermoid
    • Mucocele
    • Encephalocele
  • Cyst with spontaneous rupture
    • Orbital cellulitis
    • Orbital pseudotumor
  • Deep dermoid with mass effect
    • Orbital tumors
    • Thyroid ophthalmopathy.
 
INVESTIGATION
Classic dermoid cysts located at the frontozygomatic suture whose posterior aspect can be palpated may be diagnosed clinically without imaging. Medial lesions require imaging to rule out an encephalocele or mucocele before surgical excision. Deep orbital lesions also require imaging for diagnostic purposes and to help with surgical planning.79
 
CT Scan
A dermoid cyst typically has a hyperdense wall and a hypodense cavity which remains non-enhancing with contrast. The central cavity may appear heterogeneous as a result of keratin and other cystic debris. CT imaging is especially useful in delineating bony changes such as smooth pressure erosion (scalloped) near the affected suture, clefts, and full-thickness bony channels, seen in as much as 85% of cases. A dumbbell cyst have a typical appearance with a component on either side of the bone and a bony communication between them.
 
MRI
The lesions are generally hypointense on T1-weighted imaging with respect to fat and are best visualized using fat-suppression techniques. It appears as a well defined, round to ovoid structure of variable size. The lesions tend to be hyperintense on T2-weighted imaging. These lesions typically do not enhance with contrast due to lack of blood vessels in the cyst. MRI has the advantage of not exposing the patient to radiation, hence extremely useful in pediatric age group.
 
Ultrasonography (USG)
Ultrasound characteristics of dermoid cysts include a smooth contour and variable echo-genicity.
 
Color Doppler imaging
Color Doppler imaging of dermoid cysts shows no intralesional blood flow, which can help differentiate them from hemangioma and rhabdo-myosarcoma.
 
MANAGEMENT
A small, asymptomatic orbital epidermal dermoid cyst requires no immediate treatment. In many cases, however, the cyst slowly enlarges or ruptures, and eventually requires treatment. The treatment of orbital dermoid cysts is surgical excision. The primary goal of excision is to remove the dermoid with the cyst wall intact without causing an iatrogenic rupture. Leakage of the cystic contents into the orbit can result in significant inflammation and recurrence, while lesions removed in their entirety rarely recur.
The surgical approach depends upon the location of the lesion. An anterior orbital epidermal dermoid cyst can be removed through anterior orbitotomy (superior eyelid crease incision). A posterior orbital epidermal dermoid cyst can be removed through lateral orbitotomy. The use of a cryo-probe can help in the delivery of the cyst intact in these cases. Deeper lesions are approached based upon their location in the orbit and relationship to adjacent structures. Intracranial extension requires a multi-disciplinary surgical approach for complete excision.
Great care should be taken to remove the cyst with the capsule intact, using meticulous dissection at the site of the attachment of the cyst to the bony sutures. If the cyst is accidentally ruptured at the time of surgery, copious irrigation and attempted removal of the cyst remnants should be done.
In the rare case of dermoid cyst at the orbital apex, an orbital deroofing procedure may be necessary. If the cyst is too large to remove intact, its contents can be aspirated in order to facilitate removal. Recurrence can develop after incomplete excision.
In ruptured cyst:
  • Systemic steroid therapy
  • Systemic NSAID therapy (aspirin, ibuprofen).
 
Classification
  1. Epidermal dermoid cyst—anterior or deep
  2. Conjuctival dermoid cyst.
 
Conjunctival Dermoid Cyst
Occasionally, an otherwise typical dermoid cyst is lined by nonkeratinizing epithelium with features of conjunctival epithelium. This is called a conjunctival dermoid cyst.
 
Incidence
A conjunctival dermoid cyst is lined by conjunctival epithelium. It accounts for about 5% of dermoid cysts that occur in the orbit, with the other 95% being of epidermal origin.80
 
Clinical Features
Conjunctival dermoid cyst is probably congenital but often it is not evident until childhood or sometimes later in life.
It occurs in the superonasal aspect of the orbit usually and presents as firm or fluctuant subcutaneous mass.
 
Investigation
With CT or MRI, a conjunctival dermoid cyst has features similar to an epidermal dermoid cyst. However, it is more likely to be situated in the orbital soft tissues in the anterior and nasal aspect of the orbit, usually without contact to bone.
 
Histopathology
Histopathologically, conjunctival dermoid cyst is lined by nonkeratinizing epithelium which contains goblet cells. Like the epidermoid dermoid cyst, it contains dermal appendages such as hair shafts, sebaceous gland, and occasional sweat glands.
 
Management
A conjunctival dermoid cyst is usually symptomatic when diagnosed and is best managed by surgical excision.
Either a conjunctival or skin incision superonasally is generally used because it is usually located superonasally in the anterior orbit,
In a skin approach, an eyelid crease incision in the upper eyelid is recommended.
 
VIVA QUESTIONS
Q.1. What is the epidemiology of dermoid?
Ans. Epidemiology:
  • Congenital choriostoma
  • Account for 3–8% of orbital tumors in children
  • The dermoid cyst becomes the most common noninflammatory space-occupying lesion of the orbit
  • In the Wills Eye Hospital pathology series, dermoid cyst accounted for 46% of childhood orbital lesions and for 89% of all cystic lesions.
Q.2. Compare epidermal and conjunctival dermoid cyst?
Ans.
Epidermal dermoid
Conjunctival dermoid
  • Age of onset: Anterior dermoids typically present in first decade
  • It accounts for about 95% of dermoid cyst that occur in the orbit
  • Age of onset: Deeper dermoids may present in adolescence or adulthood
  • It accounts for about 5% of dermoid cysts that occur in the orbit
  • Histopathology: Dermoid cyst is lined by keratinizing stratified epithelium.
  • It contains dermal appendages such as hair shafts, sebaceous gland, and occasional sweat glands
  • Histopathology
  • Histopathologically, conjunctival dermoid cyst is lined by nonkeratinizing epithelium which contains goblet cells. Like the epidermoid dermoid cyst, it contains dermal appendages such as hair shafts, sebaceous gland, and occasional sweat glands
  • Site: Superotemporal usually
  • Site: Superonasal usually
  • Imaging: On CT, a dermoid typically has a hyperdense wall and a hypodense cavity which remains non-enhancing with contrast. The central cavity may appear heterogeneous as a result of keratin and other cystic debris. An estimated 85% of dermoids are associated with such bony changes as smooth pressure erosion near the affected suture, clefts, and full-thickness bony channels
  • On MRI: Hypointense on T1-weighted imaging; hyperintense on T2-weighted imaging
  • Imaging: With CT or MRI, a conjunctival dermoid cyst has features similar to an epidermal dermoid cyst. However, it is more likely to be situated in the orbital soft tissues in the anterior and nasal aspect of the orbit, usually without contact to bone81
  • Treatment: Anterior orbitotomy (superior eyelid crease incision)
  • Treatment: Anterior orbitotomy—conjunctival or skin incision superonasally
  • In a skin approach, an eyelid crease incision in the upper eyelid is recommended
Q.3. Differentiate between anterior and deep dermoid cyst.
Ans.
Anterior dermoid
Deep dermoid
Age of onset: Anterior dermoids typically present in first decade
Age of onset: Deeper dermoids may present in adolescence or adulthood
  • Suture involved:
    • Lateral: Frontozygomatic suture
    • Medial: Frontoethmoidal or frontolacrimal sutures
Suture involved: Sphenozygomatic or sphenoethmoidal suture
Symptoms: Painless fullness of upper eyelid, most commonly at lateral orbital rim
Symptoms: Painless, progressive proptosis, diplopia
Signs: Subcutaneous, mobile nodule, most commonly located at frontozygomatic suture
Signs: Proptosis, motility deficit, inferior or superior displacement of globe
Treatment: Anterior orbitotomy (superior eyelid crease incision)
Treatment: Lateral orbitotomy
Q.4. What is choriostoma?
Ans. Choristoma is a mass of histologically normal tissue present at abnormal location.
Q.5. What is difference between choriostoma and teratoma?
Ans. While choristoma is a mass of histologically normal tissue present at abnormal location, teratoma is a mass of neoplastic tissue at abnormal location.
Q.6. What is the most common location?
Ans. Children
  • The most common location is in the superior temporal aspect of the orbit. The second most common location is in the superior nasal aspect of the orbit.
  • Lesions located superotemporally are generally smooth, firm subcutaneous masses attached to the orbital rim in the region of the zygomaticofrontal suture.
  • The mass is generally less than 1 cm in diameter, nontender, and oval in shape. Little displacement of the globe usually occurs.
  • Orbital dermoid cysts are not attached to the skin, which helps differentiate them from sebaceous cysts. The cyst usually is tethered to the periosteum of the bone near suture lines, including the sinuses or intracranial cavity.
Adults
  • The cysts are palpated less easily and have more vague borders. They are more likely to displace the globe and may erode their way into adjacent structures.
  • Dystopia: A larger dermoid cyst can cause downward and medial displacement of the globe.
  • Motility deficits
  • Diplopia
Anterior lesions: typically present in the first few years of life as smooth, well-circumscribed, subcutaneous, painless masses.
  • Site: The most common location for the anterior dermoid cyst is at the superolateral aspect of the orbit at the frontozygomatic suture, as seen in the case described here. Medial lesions occur less frequently and often arise from tissue sequestered in the frontoethmoidal or frontolacrimal sutures. If there is no orbital extension, the posterior aspect of the mass may be palpable.
  • Ptosis: Because of their anterior location, these lesions do not usually cause globe displacement, but they can cause visually significant ptosis if they grow to a large enough size.82
Deep lesions are more insidious, and
  • Site: often develop at the sphenozygomatic or sphenoethmoidal suture.
  • Proptosis: Their presence is usually declared by mass effect on surrounding structures: Patients with deep lesions may present in late adolescence or adulthood with painless, progressive proptosis,
  • Dumbbell dermoids: Dermoids may also straddle the orbital bones (most commonly the lateral orbital wall) such that they have both an anterior lobe and a deeper orbital lobe. These so-called “dumbbell” dermoids must be imaged to assess the extent of the orbital component before excision.
Rupture
  • A dermoid cyst can rupture spontaneously or with trauma, inciting an intense inflammatory response in the orbital soft tissues. This response may be limited to injection of the conjunctiva or may be severe and mimic orbital cellulitis.
  • Occasionally, subconjunctival droplets of fat are seen. In some cases, a secondary fistula between the cyst and the skin may allow the contents of the cyst to drain intermittently.
  • While this is rarely the first presenting sign for an anterior dermoid, it may be the first presenting sign of a deep dermoid.
Complications
  • Rupture of the cyst
  • Orbital cellulitis
  • Recurrent cyst
  • Compressive neuropathy
  • Amblyopia
  • Strabismus.
 
ORBITAL HEMANGIOMA
Aditi Dubey, Ritika Mukhija, Rajesh Pattebahadur
 
INTRODUCTION
Orbital hemangiomas are of two types—capillary hemangioma and cavernous hemangioma. Capillary hemangioma is the most common primary benign tumor of orbit in children (infancy). Cavernous hemangioma is more common in adults (20–30 years), usually women.1,2
 
CAPILLARY HEMANGIOMA
 
History
 
Chief Complaints
Mass over eyelid present since birth or appear in the first few weeks of birth.
 
History
Usually parents complain of bluish or pink mass over eyelid present since birth with enlarging with age. In addition, there will be a history of increases in size on crying.
 
Examination
 
Inspection
Superficial hemangiomas are confined to the dermis, pink-purple mass lesion with mulberry appearance or dimpled texture and increases on crying or Valsalva. Deep orbital lesions may present with axial/non-axial proptosis.
 
Palpation
Soft, nontender, nonpulsatile, ill-defined mass over the eyelid which may have an orbital extension.
 
Auscultation
No bruit or pulsation heard.
Deep orbital lesions may present with hyperopia, optic nerve edema (due to compression), retinal striae, raised intraocular pressure and strabismus.83
 
Classification
  • Superficial or simple: Involves the skin and appear as a bright red, soft mass with a dimpled texture.
  • Preseptal or subcutaneous: Dark blue/purple soft ill-defined non-tender mass (Fig. 1). increases on crying and Valsalva nonpulsatile no bruit.
  • Deep: Located deeper within the orbit may present merely as a progressively enlarging mass without any overlying skin change (D/D Rhabdomyosarcoma should be ruled out).
 
Management
Normally the course of capillary hemangioma is as follows:
  • Rapid growth up to 6–12 months
  • 30% spontaneous resolution by 3 years
  • 70% spontaneous resolution by 7 years
Most lesions will regress spontaneously, therefore; observation, refractive correction and amblyopia therapy are the first line of management. Treatment should be deferred until it is clear that the natural course of the lesion will not lead to the desired result.
 
Indication for Treatment
  • Amblyopia secondary to astigmatism, ptosis, and anisometropia.
  • Exposure keratopathy
  • Optic nerve compression
  • Severe disfigurement or cosmetic blemish
  • Infection.
    zoom view
    Fig. 1: Capillary hemangioma
 
Modalities of Treatment
  • Small lesion < 2 mm thickness—laser
  • Superficial or preseptal lesion—intralesional steroid [betamethasone (4 mg/mL) 1–2 mL or triamcinolone (40 mg/mL) 1–2 mL at different site repeat after 2 months]. Adverse effects of steroid injection include skin necrosis, subcutaneous fat atrophy, orbital hemorrhage and rarely central retinal artery occlusion.
  • Deep/orbital lesions—systemic steroids
  • Systemic beta-blockers (inhibit angiogenesis and acts as vasoconstrictor)
  • Surgical excision may be considered for lesions that are smaller, subcutaneous or refractory to steroids
  • Interferon-α (however, it has significant systemic adverse effects and poorly tolerated)
  • Radiation therapy has also been used, but it has the potential to cause cataract formation, bone hypoplasia, and future malignancy.
 
CAVERNOUS HEMANGIOMA
 
Chief Complaints
Cavernous hemangioma—slowly progressive proptosis (growth may accelerate during pregnancy).
 
History
Cavernous hemangiomas are usually seen in adults presenting as progressive proptosis, sometimes decreased visual acuity may be present due to compressive optic neuropathy.
 
Examination
Examination is similar to a case of proptosis due to an intraconal tumor. It usually leads to axial proptosis.
 
Course and Management of Cavernous Hemangioma
  • This lesion rarely resolve spontaneously
  • Observation: If asymptomatic
  • An indication of treatment: 1. Symptomatic lesion (lesion compromising ocular function); 2. Gradually enlarging
  • Treatment: Surgical excision.84
 
VIVA QUESTIONS
Q.1. Classify orbital hemangioma.
Ans. •Cutaneous
  • Purely preseptal
  • Preseptal with orbital involvement (extraconal)
  • Preseptal with orbital involvement (extraconal + intraconal).
Q.2. What are the risk factors for capillary hemangioma?
Ans. Premature infants and newborns whose mothers had chorionic villus sampling.
Q.3. What is the most common location of hemangioma?
Ans. Capillary hemangioma: Predilection for the superonasal quadrant of the orbit and the medial upper eyelid, may involve skin over face some patients may have cutaneous and visceral hemangiomas.
Cavernous hemangioma: Extraconal and retrobulbar.
Q.4. Histopathology and imaging in hemangioma?
Ans. Shown in Table 1.
Table 1   Differentiating features between capillary and cavernous hemangioma
Parameters
Capillary hemangioma
Cavernous hemangioma
Histopathology
Tumor composed of small anastomosing channels without true encapsulation
Lesions are well encapsulated and composed of large cavernous spaces containing red blood cells with walls of the spaces containing smooth muscle
B scan
For extention of disease and anatomical relations
Well encapsulated mass lesion with cavernous fluid (blood) filled spaces
CT scan
Homogeneous enhancing soft tissue mass ± extraconal extension with fingerlike projections
Homogeneously slowly enhancing, well-encapsulated mass
MRI scan
Fine intralesional vascular channels and high blood flow
Small intralesional vascular channels with slowly flowing blood, i.e flow voids. Chronic lesions may contain radiodense phleboliths
Q.5. What are the systemically associated syndromes with capillary hemangioma?
Ans.Kasabach Merritt syndrome: Triad of hemangioma, decrease coagulation factors and thrombocytopenia. Associated with rapidly expanding visceral hemangiomas.
  • Maffucci syndrome: multiple skin and visceral hemangiomas associated with enchondromas
  • High output heart failure associated with fast growing visceral hemangiomas.
  • PHACES syndrome: Posterior fossa malformations–hemangiomas–arterial anomalies–cardiac defects–eye abnormalities–sternal cleft and supraumbilical raphe syndrome
Q.6. What are the other vascular malformations of the orbit?
Ans.
  • Hemangiopericytoma
    • The uncommon lesion, well encapsulated, hypervascular and hypercellular.
    • Appear in middle age.
    • Resemble cavernous hemangiomas on both CT and MRl, but they appear bluish intraoperatively.
    • Histologically composed of plump pericytes that surround a rich capillary network, microscopically “benign” lesions may recur and metastasize, whereas microscopically “malignant” lesions may remain localized.
    • Treatment—complete excision because they may recur, undergo mali-gnant degeneration, or metastasize.
  • Lymphatic malformation
    • Also known as lymphangiomas.
    • Due to vascular dysgenesis.
    • Become apparent in the first decade of life.85
    • Occurs in the orbit, conjunctiva, eyelids, oropharynx, or sinuses.
    • Contain both venous and lymphatic components.
    • May enlarge during URTIs and present with sudden proptosis caused by spontaneous intralesional hemor-rhage.
    • Histology: Characterized by large, not encapsulated, serum-filled channels that are lined by flat endothelial cells.
    • MRI: Pathognomonic features multiple grapes like cystic lesions with a fluid-fluid layering of the serum and red blood cells. Venography shows no arterial or venous connection.
    • Management: Surgical intervention should be deferred unless vision is affected, due to the risk of hemorrhage. A subtotal resection is generally needed to avoid sacrificing important structures. Orbital hemorrhage is allowed to resorb spontaneously; but if optic neuropathy or corneal ulceration threatens vision, aspiration of blood through a hollow-bore needle or by open surgical exploration can be attempted.
  • Venous malformations
    • Also known as orbital varices.
    • Low-flow vascular lesions due to vascular dysgenesis.
    • Clinical features: Enophthalmos at rest (when the lesion is not engorged), proptosis when the patient's head is dependent or after a Valsalva maneuver).
    • Diagnosis: Contrast-enhanced rapid spiral CT during a Valsalva maneuver showing characteristic enlargement of the engorged veins. Phleboliths may be present on imaging.
    • Treatment: Conservative
    • Biopsy: Avoided because of the risk of hemorrhage.
    • Surgery: Reserved for the relief of significant pain or for cases in which the venous malformation causes vision-threatening compressive optic neuropathy. Complete surgical excision is difficult. Intraoperative embolization of the lesion may aid surgical removal.
  • Arteriovenous malformations
    • High-flow developmental anomalies due to vascular dysgenesis.
    • Composed of anastomosing arteries and veins without an intervening capillary bed.
    • Sign: Dilated corkscrew episcleral vessels.
    • Treatment: Selective occlusion of the feeding vessels followed by surgical excision of the malformations (complication -arterial hemorrhage).
References
  1. Albert DM, Miller JW, Azar DT. Albert & Jakobiec's Principles and Practice of Ophthalmology; 2008.
  1. Brad Bowling. Kanski's Clinical ophthalmology: A systematic approach, 8th edn. Edinburgh: Elsevier;  201586
 
COLOBOMA OF EYELID
Shipra Singhi, Amar Pujari
 
INTRODUCTION
An eyelid coloboma is a full-thickness defect of the eyelid.1,2 The word coloboma comes from the Greek word that means, “Curtailed”. Lid coloboma occurs due to a delayed fusion of mesodermal components of frontonasal and maxillary processes of the face. It is caused by the failure of fusion of the mesodermal lid folds. Although an eyelid coloboma can occur in many locations, the most common position is at the junction of the medial and middle third of the upper lid.35 In exams, it can be given as a short or spot case.
 
HISTORY
 
Chief Complaint
A case of coloboma usually presents with cosmetic issues due to the defect (notching) in the eyelid. It can be associated with following:
  • Absence of eyeball
  • Bluish colored swelling (in case of crypto-phthalmos),
  • Small size of eyeball
  • Drying of eyes
  • Diminution of vision: Blurring of vision is related to corneal opacity, exposure keratopathy, cataract and choroidal coloboma. It is usually present since birth which may progress and painless in origin.
  • Diplopia due to restriction
  • Painless mass (associated limbal dermoid)
  • Foreign body sensation/irritation.
 
Past History
A careful past history should be taken of any—
  • Perinatal and pregnancy history
  • Family history of congenital eyelid colobomas or other congenital anomalies, especially facial (e.g. cleft lip/palate)
  • History of other current birth defects
  • Pediatric review of systems, hearing loss, cardiovascular disease, facial asymmetry
  • History of progressive corneal problems.
 
Past Surgical History
Previous history of ocular surgery may or may not be present.
 
EXAMINATION
 
Systemic Examination
It may be associated with multiple systemic anomalies.
  • Cardiovascular abnormalities, facial hemi atrophy, atresia of the external auditory meatus, accessory auricles, nevus flammeus, neurofibromatosis, preauricular appendages, and pre-tragal fistulas can be there. One-third of cases associated with Goldenhar's syndrome (triad of peribulbar dermoid, preauricular appendages, and pre-tragal fistulas).
  • Facial defects that may be associated with eyelid colobomas, include a less prominent supraorbital margin, and a bifid nose.
  • Among the syndromes that may include eyelid, colobomas are Goldenhar, Treacher Collins, Delleman, Fraser and nano palpebral lipoma coloboma syndrome.
 
Ocular Examination
Visual acuity: Is variably impaired depending on associated abnormalities such as limbal dermoid.
Eyeball: Microphthalmos, anophthalmos, eury- blepharon, cryptophthalmos, lagophthalmos and esotropia can be there.
Extraocular movement: Duane's retraction syndrome may be an associated feature.
Eyebrow: Loss of eyebrow hair may be seen.
Eyelids: Eyelid colobomas have following features
  • Most commonly triangular with the base at the eyelid margin.
  • It is usually located on the medial half of the upper eyelid or lateral half of the lower eyelid.
  • They are usually unilateral, generally located at the medial one-third of the upper eyelids (90%), and may vary from a small notch to complete defects of the eyelid.87
    zoom view
    Fig. 1: Surgical eyelid coloboma
  • Upper eyelid coloboma (Fig. 1) is more common than lower lid coloboma and may be associated with Goldenhar syndrome.
  • Lower eyelid coloboma: Lower lid colobomas are more commonly associated with facial clefts. Treacher Collin syndrome is usually associated with this.
Lacrimal system: Obstruction proximal to the lacrimal sac and lacrimal stenosis can be there.
Conjunctiva: Symblepharon, absence of an upper eyelid fornix and malformation of the caruncle can be seen. Conjunctival traction bands are common (present in a third of eyelid colobomas). These bands are highly amblyogenic owing to strabismus. Forced duction testing (FDT) is often positive in such cases of restriction.
Cornea: Following anomalies can be seen:
  • Exposure keratopathy
  • Corneal opacities
  • Limbal dermoid: Yellowish-white, solid, vascularized, elevated nodules straddling the corneal limbus. Size may vary ranging from 2 to 15 mm in diameter. Corneal dermoid occur as single lesions mostly but may be multiple, and they may be unilateral or bilateral, the former being the more common.
  • Dellen formation may occur
  • Cicatrization.
Lens: Cataract (anterior polar) and subluxation of the lens may be there.
Sclera: Epibulbar dermoid tumor can be there.
Iris: Coloboma (key hole)—may be typical or atypical, complete or incomplete, partial or total.
Intraocular pressure (IOP): It is usually normal
Fundus: There may be choroidal coloboma, retinal detachment due to choroidal coloboma and hypoplastic disc.
 
CLASSIFICATION
Lid coloboma can be due to following:
  • Congenital lid coloboma (isolated or syndromic)
  • Acquired lid coloboma (traumatic or post-surgical).
Depending upon the associations, it is classified as following:
 
Isolated Coloboma
  • Coloboma associated with cornea palpebral adhesions
    • Complete: No discernable eyelid differentiation and the eyes are completely covered with skin.
    • Incomplete: A skin fold devoid of tarsus covers the medial aspect of the palpebral aperture, significant cornea palpebral adhesions, lower fornix, and lateral upper eyelids usually spared.
    • Abortive type/congenital symblepharon variant: True coloboma of variable sizes with a diverse range of cornea palpebral adhesions, lower fornix, and lateral upper eyelids usually spared.
  • Simple coloboma: Coloboma not associated with cornea palpebral adhesions.
 
Syndromic Variants
  • Fraser syndrome
  • Goldenhar syndrome
  • Rare syndromes: Manitoba oculotrichoanal syndrome, Ablepharon-macrostomia syndrome, Nasopalpebral lipoma-coloboma syndrome, Amniotic band sequence, Oculoectodermal syndrome, Neurocutaneous syndromes, CHARGE (coloboma, heart defect, atresia choanae, retarded growth and development, genital abnormality, and ear abnormality) syndrome88
 
GRADING
Lid coloboma can be graded as follows (Nouby)3
  • Grade 1: Coloboma without cryptophthalmos.
  • Grade 2: Coloboma with abortive crypto-phthalmos.
  • Grade 3: Coloboma with complete crypto-phthalmos.
  • Grade 4: Classic cryptophthalmos (absence of all eyelid structures and complete coverage of eye by skin).
  • Grade 5: Severe cryptophthalmos (with severe deformity of the nose and ectropion of the upper lip).
 
INVESTIGATION
The diagnosis of a lid coloboma requires a direct clinical examination. Specific laboratory studies are generally indicated in associated syndromes that may include following:
  • X-ray of spine—for hemivertebra or scoliosis
  • ECG, echocardiography—for cardiac defect
  • MRI of brain
  • Complete blood count
  • Renal function test
  • Audiometry for hearing assessment.
 
MANAGEMENT
 
Medical Management
Corneal protection is the primary goal in the medical treatment of eyelid colobomas. Medical therapy includes artificial tears and ointment and Bedtime patching.
 
Surgical Management
Indication of surgery includes:
  • Exposure keratitis
  • Trichiasis which may cause corneal lesion.
  • Cosmesis
  • Amblyopia
  • Strabismus.
The initial evaluation of an upper eyelid coloboma consists of measuring the size of the eyelid margin defect and comparing it with the overall length of the horizontal palpebral fissure. The surgical procedure used depends on the size and the location of the defect.
Small defects: If the defect in the upper eyelid involves less than one-third of the margin, and well managed with topical lubrication, then surgery may be delayed until later in childhood. This surgery may require a lateral canthotomy and/or superior cantholysis to rotate or advance adjacent tissue to prevent excessive tension on the wound. The edges of the defect are freshened with sharp incisions, and the precise anastomosis is performed. The lid margin is brought together using a 2-layer approximation of the tarsus and the skin. Lateral cantholysis and placement of near-far, far-near sutures may be necessary to minimize horizontal tension.
Moderately-sized defects: Larger defects, a Tenzel semicircular rotational flap may be used for defects involving approximately one-third of the eyelid margin.
Large defects: If the defect is larger than one-half of the upper eyelid, other surgical procedures should be used. The various surgeries that can be performed include a free transconjunctival graft from the contralateral upper eyelid can be taken, modified Hughes procedure (for lower lid coloboma), modified Cutler-Beard procedure (upper lid coloboma), rotational flap from cheek (Mustard's technique).
Prognosis: Prognosis is excellent to good in eyelid coloboma, depending on the size of the lesion and the speed of therapy.
Patient education: Genetic consultation is highly recommended, especially for patients with associated syndromes, such as Treacher Collins syndrome, which is autosomal dominant with variable penetrance and expressivity.
 
VIVA QUESTIONS
Q.1. When should the coloboma of eyelid be repaired?
Ans. This depends on the size of the defect and on the presence of corneal exposure. If the defect of the eyelid is small and not associated with corneal exposure, surgery can be delayed until the age of 3–4 years, when there is an increased amount of eyelid tissue is available for repair. In a case of large defect, surgery should be done as soon as possible to avoid corneal lesions.89
Q.2. What is the difference between lower and upper eyelid coloboma?
Ans. See Table 1.
Table 1   Difference between upper lid and lower lid coloboma
Upper eyelid coloboma
Lower eyelid coloboma
More common
Less common
Usually isolated
Usually syndromic association
Occur at the junction of the inner and middle thirds
Occur most frequently at the junction of the middle and lateral thirds
Tend to be full thickness
Tend to be partial thickness involving preferentially the anterior lamella
Have normal adjacent lid margins
Adjacent lid margins may be abnormal
Usually not associated with facial clefts.
Usually associated with facial clefts
Often associated with cryptophthalmos
Usually not
Q.3. What are the clinical findings in Treacher Collins syndrome?
Ans. See Table 2.
Q.4. What are the clinical findings in Goldenhar syndrome?
Ans.
  • Limbal dermoid (bilateral in 25% of cases)
  • Eyelid coloboma
  • Preauricular appendages/skin tags
  • Microtia or anotia of external ear can be associated with hearing loss with or without middle ear malformation
  • Vertebral abnormalities (butterfly vertebrae or hemivertebrae)
  • Congenital heart disease (numerous anomalies have been reported)
  • Central nervous system abnormalities (hydrocephalus, intracranial lipomas, cranial nerve dysgenesis and mental retardation have been described).
Q.5. What are the clinical findings in Fraser syndrome?
Ans. Major characteristics:
  • Cryptophthalmos
  • Syndactyly
  • Genital anomalies
  • Sibling with Fraser syndrome.
Minor characteristics:
  • Alterations of the nose
  • Alterations of the ears
  • Alterations of the larynx
  • Oral clefts (cleft lip and/or palate)
  • Umbilical hernia
  • Renal agenesis (unilateral or bilateral)
  • Skeletal anomalies.
Table 2   Treacher Collins syndrome
Structure affected
Clinical features
Eyes
  • Antimangloid slant of palpebral fissures
  • Coloboma of lower eyelid
  • Hypoplasia of lower eyelid
  • Hypertelorism
Ears
  • Microtia
  • Conductive hearing loss
  • Stenosis or complete atresia
  • External ear abnormalities
Face
  • Hypoplasia of facial bones (mandibular or zygomatic arch) or complete absence of zygomatic arch
  • Dental malocclusion
  • Microstomia
  • High arched palate, cleft palate
  • Clonal atresia
  • Nasal dorsus parrot like shape
Others
  • Malformations associated with heart, kidney, vertebral column and extremities
  • Obstructive sleep apnea.
References
  1. Casey TA. Congenital colobomata of the eyelids. Trans Ophthalmol Soc UK. 1976;96:65–8.
  1. Collin JR. Congenital upper lid coloboma. Aust NZ J Ophthalmol. 1986;14:313–7.
  1. Online Mendelian Inheritance in Man (OMIM) database, http://www.ncbi.nlm.nih.gov/omim (accessed 5 May 2013).
  1. Pearson AA. The development of the eyelids. Part I. External features. J Anat. 1980;130(1):33–42.
  1. Sevel D. A reappraisal of eyelid development. Eye. 1988;2:123–9.